Math140 2003-04

You might also like

Download as pdf or txt
Download as pdf or txt
You are on page 1of 142

McGILL UNIVERSITY

FACULTY OF SCIENCE

DEPARTMENT OF
MATHEMATICS AND STATISTICS

MATH 140 2003 09


CALCULUS I

Information for Students


(Fall Term, 2003/2004)
Pages 1 - 20 of these notes may be considered the
Course Outline for this course.
W. G. Brown and N. Sancho
October 25, 2004
Information for Students in MATH 140 2003 09

Contents 1.8.2 Use of Calculators and


Computer Algebra Sys-
1 General Information 1 tems . . . . . . . . . . . 19
1.1 Instructors and Times . . . . . 1 1.8.3 Use of the Internet . . . 20
1.2 Calendar Description . . . . . . 1 1.9 Special Review Sessions . . . . 20
1.2.1 Late transfer from MATH
150 . . . . . . . . . . . . 2 A Information Specifically for Students
1.3 Tutorials; Tutors’ Coordinates . 2 in Lecture Section 001 1001
1.4 Evaluation of Your Progress . . 3 A.1 Timetable for Lecture Section 001
1.4.1 Your final grade . . . . 3 of MATH 140 2003 09 . . . . . 1001
1.4.2 WeBWorK . . . . . . 5
1.4.3 WeBWorK Precalculus B Supplementary Notes for Lectures
Assignments . . . . . . 5 in Lecture Section 001 1003
1.4.4 Regular WeBWorK As-
signments . . . . . . . . 6 C Information Specifically for Students
1.4.5 Written Assignments . . 7 in Lecture Section 002 2001
1.4.6 Quizzes at the Tutorials; C.1 Timetable for Lecture Section 002
Submission of Written As- of MATH 140 2003 09 . . . . . 2001
signments at the Quizzes 7
D Supplementary Notes for Lectures
1.4.7 Final Examination . . . 9
in Lecture Section 002 2003
1.4.8 Supplemental Assessments 9
D.1 Supplementary Notes for the Lec-
1.4.9 Machine Scoring . . . . 9
ture of September 3rd, 2003 . . 2003
1.4.10 Plagiarism . . . . . . . . 9
D.1.1 §1.1 Four Ways to Rep-
1.5 Published Materials . . . . . . 10
resent a Function. . . . 2005
1.5.1 Required Text-Book . . 10
D.1.2 §1.2 Mathematical Mod-
1.5.2 Optional Reference Books 10
els: A Catalog of Essen-
1.5.3 Recommended Video Ma-
tial Functions. . . . . . 2008
terials . . . . . . . . . . 12
D.1.3 §1.3 New Functions from
1.5.4 Other Calculus Textbooks 13
Old Functions. . . . . . 2010
1.5.5 Website . . . . . . . . . 14
D.1.4 §1.4 Graphing Calcula-
1.6 Syllabus . . . . . . . . . . . . . 15
tors and Computers . . 2011
1.7 Preparation and Workload . . . 16
D.2 Supplementary Notes for the Lec-
1.7.1 Prerequisites. . . . . . . 16
ture of September 8th, 2003 . . 2012
1.7.2 Calculators . . . . . . . 17
D.2.1 §1.5 Exponential Functions2012
1.7.3 Self-Supervision . . . . . 17
D.2.2 §1.6 Inverse Functions and
1.7.4 Escape Routes . . . . . 18
Logarithms . . . . . . . 2013
1.7.5 Terminology . . . . . . 18
D.2.3 1 Review . . . . . . . . 2019
1.8 High and Low Technology and
D.3 Supplementary Notes for the Lec-
MATH 140 . . . . . . . . . . . 19
ture of September 10th, 2003 . 2021
1.8.1 Keep your e-mail address
D.3.1 §2.1 The Tangent and Ve-
up to date . . . . . . . . 19
locity Problems. . . . . 2021
Information for Students in MATH 140 2003 09

D.3.2 §2.2 The Limit of a Func- D.10 Supplementary Notes for the Lec-
tion. . . . . . . . . . . . 2022 ture of October 8th, 2003 . . . 2051
D.4 Supplementary Notes for the Lec- D.10.1 §3.3 Rates of Change in
ture of September 15th, 2003 . 2024 the Natural and Social
D.4.1 §2.4 The Precise Defini- Sciences. . . . . . . . . . 2051
tion of a Limit. . . . . . 2025 D.10.2 §3.4 Derivatives of Trigono-
D.4.2 §2.3 Calculating Limits metric Functions. . . . . 2052
Using the Limit Laws. . 2027 D.11 Supplementary Notes for the Lec-
D.4.3 Appendix A. Numbers, ture of October 15th, 2003 . . . 2055
Inequalities, and Absolute D.11.1 §3.5 The Chain Rule. . 2055
Values . . . . . . . . . . 2028 D.12 Supplementary Notes for the Lec-
D.4.4 Appendix B. Coordinate ture of October 20th, 2003 . . . 2058
Geometry and Lines . . 2028 D.12.1 §3.6 Implicit Differentia-
D.4.5 Appendix C. Graphs of tion. . . . . . . . . . . . 2059
Second-Degree Equations 2029 D.13 Supplementary Notes for the Lec-
D.4.6 Appendix D. Trigonom- ture of October 22nd, 2003 . . 2065
etry . . . . . . . . . . . 2029 D.13.1 §3.7 Higher Derivatives 2065
D.5 Supplementary Notes for the Lec- D.14 Supplementary Notes for the Lec-
ture of September 17th, 2003 . 2031 ture of October 27th, 2003 . . . 2071
D.5.1 §2.5 Continuity. . . . . . 2031 D.14.1 §3.8 Derivatives of Log-
D.6 Supplementary Notes for the Lec- arithmic Functions . . . 2071
ture of September 22nd, 2003 . 2035 D.15 Supplementary Notes for the Lec-
D.6.1 §2.6 Limits at Infinity; Hor- ture of October 29th, 2003 . . . 2076
izontal Asymptotes. . . 2036 D.15.1 §3.9 Hyperbolic Functions 2076
D.7 Supplementary Notes for the Lec- D.15.2 §3.10 Related Rates. . . 2079
ture of September 24th, 2003 . 2040 D.16 Supplementary Notes for the Lec-
D.7.1 §2.7 Tangents, Velocities, ture of November 3rd, 2003 . . 2084
and Other Rates of Change.2041 D.16.1 §3.10 Related Rates (con-
D.7.2 §2.8 Derivatives. . . . . 2043 tinued). . . . . . . . . . 2085
D.8 Supplementary Notes for the Lec- D.16.2 §3.11 Linear Approxima-
ture of September 29th, 2003 . 2044 tions and Differentials . 2086
D.8.1 §2.9 The Derivative as a D.16.3 3 Review . . . . . . . . 2090
Function . . . . . . . . . 2044 D.17 Supplementary Notes for the Lec-
D.8.2 2 Review . . . . . . . . 2045 ture of November 5th, 2003 . . 2091
D.9 Supplementary Notes for the Lec- D.17.1 §4.1 Maximum and Min-
ture of October 1st, 2003 . . . 2047 imum Values . . . . . . 2091
D.9.1 §3.1 Derivatives of Poly- D.18 Supplementary Notes for the Lec-
nomials and Exponential ture of November 10th, 2003 . 2096
Functions . . . . . . . . 2047 D.18.1 §4.2 The Mean Value The-
D.9.2 §3.2 The Product and Quo- orem . . . . . . . . . . . 2096
tient Rules . . . . . . . 2048 D.19 Supplementary Notes for the Lec-
ture of November 12th, 2003 . 2103
Information for Students in MATH 140 2003 09

D.19.1 §4.3 How Derivatives Af- E.3 2000/2001 Problem Assignments,


fect the Shape of a Graph 2103 with Solutions . . . . . . . . . . 3023
D.20 Supplementary Notes for the Lec- E.3.1 First 2000/2001 Problem
ture of Monday, November 17th, Assignment, with Solu-
2003 . . . . . . . . . . . . . . . 2108 tions . . . . . . . . . . . 3023
D.20.1 §4.4 Indeterminate Forms E.3.2 Second 2000/2001 Prob-
and L’Hospital’s Rule . 2108 lem Assignment, with So-
D.21 Supplementary Notes for the Lec- lutions . . . . . . . . . . 3034
ture of Wednesday, November E.3.3 Third 2000/2001 Prob-
19th, 2003 . . . . . . . . . . . . 2115 lem Assignment, with So-
D.21.1 §4.5 Summary of Curve lutions . . . . . . . . . . 3041
Sketching . . . . . . . . 2115 E.3.4 Fourth 2000/2001 Prob-
D.21.2 §4.6 Graphing with Cal- lem Assignment, with So-
culus and Calculators . 2120 lutions . . . . . . . . . . 3050
D.22 Supplementary Notes for the Lec- E.3.5 Fifth 2000/2001 Problem
ture of November 24th, 2003 . 2122 Assignment, with Solu-
D.22.1 §4.7 Optimization Prob- tions . . . . . . . . . . . 3056
lems . . . . . . . . . . . 2124 E.3.6 Sixth 2000/2001 Problem
D.22.2 §4.8 Applications to Busi- Assignment, with Solu-
ness and Economics . . 2127 tions . . . . . . . . . . . 3061
D.22.3 §4.9 Newton’s Method . 2128
D.23 Supplementary Notes for the Lec- F Some Tests and Quizzes from Pre-
ture of November 26th, 2003 . 2129 vious Years 3065
D.23.1 §4.10 Antiderivatives . . 2129 F.1 Fall 1998 Class Quiz, with So-
D.23.2 4 Review . . . . . . . . 2134 lutions . . . . . . . . . . . . . . 3065
F.2 Last Three Tutorial Quizzes in
E Assignments from Previous Years 3001 2000/2001 (many versions) . . 3068
E.1 Fall 1998 Problem Assignments 3001 F.2.1 Fourth 2000/2001 Tuto-
E.2 Fall 1999 Problem Assignments 3001 rial Quizzes . . . . . . . 3068
E.2.1 First Fall 1999 Problem F.2.2 Fifth 2000/2001 Tutor-
Assignment, with Solu- ial Quizzes . . . . . . . 3071
tions . . . . . . . . . . . 3001 F.2.3 Sixth 2000/2001 Tutor-
E.2.2 Second Fall 1999 Prob- ial Quizzes . . . . . . . 3074
lem Assignment . . . . . 3011
E.2.3 Third Fall 1999 Problem G Examinations from Previous Years 3079
Assignment, with Solu- G.1 December 1996 Final Examina-
tions . . . . . . . . . . . 3012 tion in 189-122A . . . . . . . . 3079
E.2.4 Fourth Fall 1999 Prob- G.2 December 1997 Final Examina-
lem Assignment . . . . . 3020 tion in 189-140A . . . . . . . . 3081
E.2.5 Fifth Fall 1999 Problem G.3 December 1998 Final Examina-
Assignment . . . . . . . 3021 tion in 189-140A . . . . . . . . 3083
G.4 May 1999 Supplemental Exam-
ination in 189-140A . . . . . . . 3085
Information for Students in MATH 140 2003 09

G.5 December 1999 Final Examina- H.1.7 WeBWorK provides for


tion in 189-140A . . . . . . . . 3086 different kinds of “Dis-
G.6 December 1999 Special Final Ex- play Mode”. Which should
amination in 189-140A . . . . . 3088 I use? . . . . . . . . . . 4002
G.7 December 2000 Final examina- H.1.8 WeBWorK provides for
tion in 189-140A . . . . . . . . 3089 printing assignments in
G.8 May 2001 Supplemental/Deferred “Portable Document Format”
Examination in 189-140A . . . 3090 (.pdf) or “PostScript” (.ps)
G.9 December 2001 Final Examina- form. Which should I use?4003
tion in 189-140A . . . . . . . . 3091 H.1.9 What is the relation be-
G.10 May 2002 Supplemental/Deferred tween WeBWorK and
Examination in 189-140A . . . 3093 WebCT? . . . . . . . . . 4003
G.11 December 2002 Final Examina- H.1.10 Which browser should I
tion in MATH 140 2002 09 . . 3095 use for WeBWorK? . . 4003
G.12 May 2003 Supplemental/Deferred H.1.11 What do I have to do on
Examination in MATH 140 2002 WeBWorK? . . . . . . 4004
09 . . . . . . . . . . . . . . . . 3097 H.1.12 How can I learn how to
use WeBWorK? . . . . 4004
H WeBWorK 4001 H.1.13 Where should I go if I
H.1 Frequently Asked Questions (FAQ)4001 have difficulties with WeB-
H.1.1 Where is WeBWorK? 4001 WorK ? . . . . . . . . . 4005
H.1.2 Do I need a password to H.1.14 Can the WeBWorK sys-
use WeBWorK? . . . . 4001 tem ever break down or
H.1.3 Do I have to pay an ad- degrade? . . . . . . . . . 4005
ditional fee to use WeB- H.1.15 How many attempts may
WorK? . . . . . . . . . 4002 I make to solve a partic-
H.1.4 When will assignments be ular problem on WeB-
available on WeBWorK?4002 WorK? . . . . . . . . . 4005
H.1.5 Do WeBWorK assign- H.1.16 Will all Regular WeB-
ments cover the full range WorK assignments have
of problems that I should the same length? the same
be able to solve in this value? . . . . . . . . . . 4006
course? . . . . . . . . . 4002 H.1.17 Is WeBWorK a good in-
H.1.6 May I assume that the dicator of examination per-
distribution of topics on formance? . . . . . . . . 4006
quizzes and final exami-
nations will parallel the I Contents of the DVD disks for
distribution of topics in Larson/Hostetler/Edwards 5001
the WeBWorK assign-
ments? . . . . . . . . . . 4002 J References 6001
J.1 Stewart Calculus Series . . . . 6001
J.2 Other Calculus Textbooks . . . 6002
J.2.1 R. A. Adams . . . . . . 6002
Information for Students in MATH 140 2003 09

J.2.2 Larson, Hostetler, et al. 6002


J.2.3 Edwards and Penney . . 6002
J.2.4 Others, not “Early Tran-
scendentals” . . . . . . . 6003
J.3 Other References . . . . . . . . 6003
Information for Students in MATH 140 2003 09: 1

1 General Information
Distribution Date: This version, September 11th, 2003
(all information is subject to change)
Pages 1 - 20 of these notes may be considered the Course Outline for this course.

These notes may undergo minor corrections or updates during the term:
the definitive version will be the version accessible at

http://www.math.mcgill.ca/brown/math140a.html

or on WebCT, at

http://www.mcgill.ca/webct or http://webct.mcgill.ca

1.1 Instructors and Times

INSTRUCTOR: Prof. N. Sancho Prof. W. G. Brown


(Course Coordinator)
LECTURE SECTION: 1 2
CRN: 556 559
OFFICE: BURN 1130 BURN 1224
OFFICE HOURS: MW 11:15 - 12:15 W 14:30 - 15:30
(subject to or by appointment F 10:00 - 11:00
change) or by appointment
TELEPHONE: 398–3823 398–3836
E-MAIL: SANCHO@ BROWN@
MATH.MCGILL.CA MATH.MCGILL.CA
CLASSROOM: ADAMS AUD ADAMS AUD
CLASS HOURS: MWF 8:35–9:25 h. MW 16:35–17:55 h.

Table 1: Instructors and Times

1.2 Calendar Description


MATH 1401 CALCULUS I. (3 credits. 3 hours lecture; 1 hour tutorial. Prerequisite:
High School Calculus. Not open to students who have taken MATH 120, MATH 122,
1
The previous designation for this course was 189-140, and the version given in the fall was labelled
189-140A; an earlier number for a similar course was 189-122.

UPDATED TO October 25, 2004


Information for Students in MATH 140 2003 09: 2

MATH 139 or CEGEP objective 00UN or equivalent. Not open to students who have
taken or are taking MATH 130 or MATH 131, except by permission of the Department
of Mathematics and Statistics. Each Tutorial section is enrolment limited.) Review
of functions and graphs. Limits, continuity, derivative. Differentiation of elementary
functions. Antidifferentiation. Applications.
In addition to MATH 140, the Department of Mathematics and Statistics offers two other
beginning calculus courses:

• MATH 139 Calculus (4 credits); Instructor = Prof. S. Drury. This course


(whose prerequisite is a course in functions) is intended for students who have never
had a course in calculus. Students may apply at the Department of Mathematics and
Statistics before the end of the Course Change Period for authorization to register in this
course; they must bring copies of their transcripts. This course covers approximately the
same material as MATH 140.2 MATH 139 uses the same textbook as MATH 140.

• MATH 150 Calculus A (4 credits); Instructor = Dr. A. Hundemer. This course,


together with its sequel, MATH 151 Calculus B (also 4 credits), covers approximately
the material of courses MATH 140/MATH 139, MATH 141 Calculus II, and MATH 222
Calculus III, in only two semesters. A course in Vector Geometry (e.g. MATH 133) is
pre- or corequisite.3

1.2.1 Late transfer from MATH 150


Some students from MATH 150 may be permitted to transfer into MATH 140 after the
end of the Change of Course Period. If you are in this category, please send an e-mail
message to Professor Brown as soon as your transfer has been approved.4

1.3 Tutorials; Tutors’ Coordinates


Every student must be registered in one lecture session and one tutorial for this course.
Tutorials begin in the week of September 8th, 2003. The last tutorial in Tutorial sections
##009–020 will be in the week beginning November 24th, 2003; but Tutorial sections
##003-008 — which do not meet on Monday, October 13th — will meet on Monday,
December 1st, 2003. Table 2 below gives times, locations, and the tutor’s name for
2
Authorization for registration in MATH 139 2003 09 will be available beginning late in August,
2003, and ending on Friday, September 12th, 2003. For details, see the following URL:

http://www.math.mcgill.ca/brown/incoming.htm

3
Note that this year MATH 150/151 is using a different textbook from MATH 140/141; the textbook
is by R. A. Adams [15], [13],[14],[16],[17].
4
This is to ensure that your WeBWorK account is opened, and that your date of entry to the course
is recorded.

UPDATED TO October 25, 2004


Information for Students in MATH 140 2003 09: 3

each of the tutorials; Table 3 gives the tutors’ coordinates. The information in these
tables is subject to change. We try to publicize changes but sometimes we
are not informed in advance.5

1.4 Evaluation of Your Progress


1.4.1 Your final grade
(See Table 5, p. 11) Your grade in this course will be a letter grade, based on a percentage
grade computed from the following components:
1. Assignments submitted over the Web:

(a) A WeBWorK (see §1.4.2 below) “Precalculus” Assignment (see §1.4.3) which
tests your preparation for the course. Until you pass one of these assignments
with a grade of 75%, your grade on the regular WeBWorK assignments will
count for 0. The Precalculus Assignments will be numbered #P1, . . . , #P6,
and you need pass only one of them with the required grade. The grade itself
will not be used in the computations; but, if you fail to obtain 75% on one
of P1 — P6, your “Regular” WeBWorK assignments (see §1.4.4) will count
for 0/10.
(b) Ten “Regular” WeBWorK homework assignments (cf. §1.4.4) — counting
together for 10%, provided the student has obtained a grade of 75% or better
on one precalculus assignment6 (cf. §1.4.1.1a). The Regular Assignments will
be numbered #R1, . . . #R10.

2. Materials graded by your Tutor:

• Five Written Assignments — counting together for 5%. The Written Assign-
ments will be numbered #W1, . . . , #W5.
• Five Quizzes given at the tutorials — counting together for 15%. The Quizzes
which count will be numbered #Q1, . . . , #Q5. (There will also be a quiz #Q0,
which will be graded, but the grade will not count in your term mark.)

3. The final examination — counting for 70%.


Where a student’s performance on the final examination is superior to her performance
on the tutorial quizzes, the final examination grade will replace the quiz grades in the
5
The current room for your tutorial should always be available by clicking on “Class Schedule” on
MINERVA FOR STUDENTS, http://www.mcgill.ca/minerva-students/.
6
Don’t delay completing the regular assignments until you have passed a precalculus assignment.
The regular assignments have expiration dates, and must be completed by the posted times.

UPDATED TO October 25, 2004


Information for Students in MATH 140 2003 09: 4

# CRN Day Begins Ends Room Tutor


T003 564 Mon 13:35 14:25 BURN 1214 M. Fortin-Boisvert
T004 565 Mon 14:35 15:25 BURN 1214 M. Fortin-Boisvert
T005 566 Mon 14:35 15:25 MAASS 328 Y. Gao
T006 567 Mon 15:35 16:25 BURN 1B24 Y. Gao
T007 568 Mon 14:35 15:25 MAASS 217 T. Lepage
T008 569 Mon 15:35 16:25 BURN 1B36 T. Lepage
T009 570 Wed 13:35 14:25 BURN 1214 X. Liu
T010 571 Wed 14:35 15:25 BURN 1214 X. Liu
T011 572 Wed 14:35 15:25 BURN 1B39 P. Xu
T012 573 Wed 15:35 16:25 BURN 1214 P. Xu
T013 574 Wed 14:35 15:25 ARTS 265 M. Alakhrass
T014 575 Wed 15:35 16:25 ARTS 265 M. Alakhrass
T015 576 Thurs 16:05 16:55 ARTS 145 M. Taddy
T016 577 Thurs 17:05 17:55 ARTS 145 M. Taddy
T017 578 Wed 15:35 16:25 BURN 1B24 Y. Tan
T018 579 Wed 16:35 17:25 BURN 1214 Y. Tan
T019 580 Fri 14:35 15:25 BURN 1B24 Ph. Poulin
T020 581 Fri 15:35 16:25 BURN 1B23 Ph. Poulin

Table 2: Schedule and Locations of Tutorials, as of October 25, 2004 (subject to change)

Tutor E-mail address Office Office Hours


Day Begins Ends Day Begins Ends
Alakhrass, M. alakhras@math.mcgill.ca 1115 M 09:30 12:30
Fortin-Boisvert, M. boisvert@math.mcgill.ca 1008 Th 11:00 14:00
Gao, Y. gao@math.mcgill.ca 1117 T 10:00 13:00
Lepage, T. lepage@math.mcgill.ca 1018 F 09:30 12:30
Liu, X. liu@math.mcgill.ca 1020 M 13:30 16:30
Poulin, Ph., ppoulin@math.mcgill.ca 1032 W 15:30 16:30 F 16:30 18:30
Taddy, M. taddy@math.mcgill.ca 1218 T 13:30 14:30 W 16:00 18:00
Tan, Ye tan@math.mcgill.ca 1031 W 09:30 11:30 W 13:30 14:30
Xu, P. pxu@math.mcgill.ca 1021 M 12:30 15:30

Table 3: Tutors’ Coordinates, as of October 25, 2004

UPDATED TO October 25, 2004


Information for Students in MATH 140 2003 09: 5

calculations. It is not planned to permit the examination grade to replace the grades on
WeBWorK assignments or on written assignments.

1.4.2 WeBWorK
We will be using the WeBWorK system, developed at the University of Rochester —
which is designed to expose you to a large number of drill problems, and where plagiarism
is discouraged. WeBWorK is accessible only over the Web. Details on how to sign on
to WeBWorK are contained in Appendix H to these notes, page 4001.
WeBWorK assignments carry a due date and time; only answers submitted by this
date and time will count.
Some of the conditions described below may appear to be strict. They are designed
to reduce the chance that you fail the course, not to impose unreasonably bureaucratic
rules.

1.4.3 WeBWorK Precalculus Assignments


The series of assignments called Precalculus Assignments are designed to test your prepa-
ration for the course. You must obtain a grade of at least 75% on one of the Precalculus
Assignments. If your grade on Precalculus Assignment #P1 is lower than 75%, you may
try Precalculus Assignment #P2 when it becomes available: you are required to obtain
75% on one of these assignments, but the grade itself will not be used in the computation
of your final mark; however, if you do not pass any of Precalculus Assignments #P1,
#P2,#P3,#P4,#P5,#P6, your grade on the Regular WeBWorK assignments will be
considered to be zero.7 Precalculus Assignments will remain available during the term;
as one is closed, the next will become available.
There will be no limit on the number of attempts you make on the problems on
Precalculus Assignment P1; on subsequent Precalculus Assignments there may be limited
numbers of attempts permitted — check your work carefully before you submit your
answer. Once you have passed one of the Precalculus Assignments at the 75% level,
there is no need to try any of the other Precalculus Assignments, although you are
welcome to do so. It is possible that the numbers of attempts permitted for problems
on the Precalculus Assignments will be reduced as the sequence progresses: try to pass
7
If you see no hope in being able to pass any Precalculus Assignment, you may infer that you are
at risk of failing the course, and should seriously consider dropping it. The final deadline for this is
Sunday, October 12th, 2003 — by which time there is no fee refund; the deadline for withdrawal with
partial fee refund is Sunday, September 21st, 2003. If you must drop the course, you could use the term
to repair your foundations, and register again in the Winter term.
(Dates from the standard University Calendar of Dates are provided in these notes only for conve-
nience; students should verify these dates themselves, as the accuracy is not guaranteed.)

UPDATED TO October 25, 2004


Information for Students in MATH 140 2003 09: 6

the Assignment as early as you can, since the purpose of the assignment is to help you
review precalculus concepts.

1.4.4 Regular WeBWorK Assignments


In addition to the Precalculus Assignments, one of which you are required to pass with
a grade of 75% or better, there will be ten “Regular” WeBWorK assignments. These
will be paired — each even numbered assignment (##R2, R4, R6, R8, R10) will contain
the same types of problems as on the preceding odd numbered assignment (##R1,
R3, R5, R7, R9). You will have an unlimited number of tries on the problems on
the odd numbered assignments; but there will be restricted numbers of tries on the
even numbered assignments. The intention is that you should use each odd numbered
assignment to thoroughly learn how to solve the problems, and then attempt the following
even numbered assignment. The data on the even numbered assignment will probably be
different — different numbers and/or functions, but the same concepts.8 It is expected
that the due date for Regular assignments will be on specified Mondays, at midnight.
As mentioned in the WeBWorK FAQ (cf. Appendix §H) if you leave your WeBWorK
assignment until the hours close to the due time on the due date, you should not be
surprised if the system is slow to respond. This is not a malfunction, but is simply
a reflection of the fact that other students have also been procrastinating! To benefit
from the speed that the system can deliver under normal conditions, do not delay your
WeBWorK until the last possible day! If a systems failure interferes with the due date
of an assignment, arrangements may be made to change that date, and an e-mail message
may be broadcast to all users (to the e-mail addresses on record), or a note posted in
the course announcements on WebCT; but slowness in the system just before the due
time will not normally be considered a systems failure.9

BONUS assignment (added September 13th, 2003) It is hoped that there will
be one additional WeBWorK assignment, which will be optional . The grade on this
assignment would replace the grade on any other WeBWorK assignment with a lower
grade, or replace a missed assignment. This assignment will have limits on the numbers
of attempts. Further details will be announced in the lectures or on the WebCT or
WeBWorK sites.
8
The 10 assignments will count equally in your WeBWorK grade, provided you have received a grade
of at least 75% on one Precalculus Assignment; if you have not received that grade on any Precalculus
Assignment by the end of the term, your WeBWorK grade will be 0.
9
Should you find that the system is responding slowly, do not submit your solutions more than once;
you may deplete the number of attempts that have been allowed to you for a problem: this will not be
considered a systems failure.

UPDATED TO October 25, 2004


Information for Students in MATH 140 2003 09: 7

1.4.5 Written Assignments


Written assignments will be posted at about the same time as regular WeBWorK
assignments ##R2, R4, R6, R8, R10. These assignments will contain one or more
problems for which you will be expected to write full solutions, modelled on similar
types of solution in worked examples in the textbook, or solved problems in the Student
Solutions Manual. You will be assigned an individualized version of the problems. You
should prepare these solutions and carry them to the Tutorial Quiz the following week;
you will be expected to hand in your solutions with your quiz paper, and they will be
graded by your tutor and returned to you with that graded quiz paper. Please do not
attempt any other methods for submitting your written assignments, to minimize the
risk of loss.
Written Where to Find How to Submit Date of
Assignment Your Version of the Questions Your Solutions Quiz
#W1 on WeBWorK site with Quiz #Q1 Sept. 22-26
#W2 on WeBWorK site with Quiz #Q2 Oct. 06-10
#W3 on WeBWorK site with Quiz #Q3 Oct. 20-24
#W4 on WeBWorK site with Quiz #Q4 Nov. 03-07
#W5 on WeBWorK site with Quiz #Q5 Nov. 17-24

Table 4: Timetable for Written Assignments

1.4.6 Quizzes at the Tutorials; Submission of Written Assignments at the


Quizzes
1. There will be 6 short quizzes, numbered #Q0, #Q1, #Q2, #Q3, #Q4, #Q5,
administered at the tutorials. These quizzes will be graded, and returned (together
with the written assignment that will be handed in with the quiz paper). Because
the tutorials in this course are only about 50 minutes long, the time available for
quizzes is very limited; consequently the types of quiz questions could be different
from questions typically found on final examinations; the primary purpose of a
quiz is to diagnose possible gaps in your understanding, not to drill on examination
skills.
No provision is being made for students who miss a quiz. The grading formula per-
mits the quiz component of the final grade to be replaced by the final examination
grade, if this is to a student’s advantage.
2. The grade you obtain on quiz #Q0, given during the week of September
15nd, 2003, DOES NOT COUNT IN YOUR TERM MARK. Quiz #Q1

UPDATED TO October 25, 2004


Information for Students in MATH 140 2003 09: 8

— the first quiz whose grade “counts” — will be administered during the tutorials
of the week of September 22nd, 2003. Quiz #Q0 is planned to be all or partly a
review of precalculus concepts.

3. The remaining quizzes will be based on current topics in the syllabus of the course,
most10 of which topics will have been discussed in the lectures before the quiz;
the quizzes are not based directly on WeBWorK assignments. To prepare for
a quiz you should be working exercises in the textbook based on the material
currently under discussion at the lectures, and you should have attempted any open
WeBWorK assignments. But, unlike the WeBWorK assignments — where the
emphasis is on correct answers alone — students may be expected to provide full
solutions to some or all problems on quizzes.11

4. You are expected to write the quiz in the tutorial section in which you are registered.
Should the classroom become filled while some registered students still cannot be
seated, the tutor may insist that any students who are not registered in that tutorial
to leave the room.12

5. Submission of Written Assignments at Quizzes (excluding quiz #Q0)


Individualized written assignments ##W1, W2, W3, W4, W5 will be mounted on
the WeBWorK site about a week before they are to be handed in at a tutorial
quiz; you should download or copy each written assignment from the web site,
write out your solutions at home, bring the completed assignment with you to the
tutorial, and hand it in with your quiz paper — no other submission method is
acceptable. The assignment must be ready for submission when you arrive, as you
will not be provided time in the tutorial room to complete it. You will enclose the
written assignment in your folded answer paper.

6. Your tutors will normally bring graded quizzes and graded assignments submitted
with them to the tutorial to be returned to you. University regulations do not
permit us to leave unclaimed materials bearing names and student numbers in
unsupervised locations; you may be able to recover an unclaimed quiz from the
tutor who graded it, during her/his regular office hours.
10
but possibly not all
11
In Math 140 and Math 141 the general rule is that full solutions are expected to all problems, unless
you receive explicit instructions to the contrary: ALWAYS SHOW YOUR WORK! The solutions in the
Student Solution Manual [3] to the textbook can serve as a guide to what should be included in a “full”
solution.
12
Anyone who is not registered, and who does not leave the room when so requested may forfeit the
right to write quizzes at any tutorial, and could be subject to disciplinary action through their Faculty.
We ask you to respect the prior right of students who have registered for each tutorial.

UPDATED TO October 25, 2004


Information for Students in MATH 140 2003 09: 9

1.4.7 Final Examination


A 3-hour-long final examination will be scheduled during the regular examination period
for the fall term (December 5th, 2003 through December 19th, 2003). You are advised
not to make any travel arrangements that would prevent you from being present on
campus at any time during this period.

1.4.8 Supplemental Assessments


Supplemental Examination. There will be a supplemental examination in this course.
(For information about Supplemental Examinations, see the McGill Calendar, [27, §8.1,
p. 250-251; or §8.1, p. 52-53].)

There is No Additional Work Option. “Will students with marks of D, F, or J


have the option of doing additional work to upgrade their mark?” No. (“Additional
Work” refers to an option available in certain Arts and Science courses, but not available
in this course.)

1.4.9 Machine Scoring


“Will the final examination be machine scored?” It is possible that the final examination
will contain some questions that will be machine scored. Not more than 20% of the
marks available on the examination — possibly less — will derive from problems that
are machine scored.

1.4.10 Plagiarism
While students are not discouraged from discussing methods for solving WeBWorK
assignment problems with their colleagues, all the work that you submit — whether
through WeBWorK or written assignments, or on tutorial quizzes or the final exami-
nation must be your own. It is a violation of University regulations to permit others to
solve your WeBWorK problems, or to extend such assistance to others; you could be
asked to sign a statement attesting to the originality of your work.
The Handbook on Student Rights and Responsibilities states in ¶15(a)13 that

“No student shall, with intent to deceive, represent the work of another person
as his or her own in any academic writing, essay, thesis, research report,
project or assignment submitted in a course or program of study or represent
as his or her own an entire essay or work of another, whether the material so
represented constitutes a part or the entirety of the work submitted.”
13
http://upload.mcgill.ca/secretariat/G1547E.pdf

UPDATED TO October 25, 2004


Information for Students in MATH 140 2003 09: 10

You are also referred to the following URL:

http://www.mcgill.ca/integrity/studentguide/

The preceding paragraph was prepared before all faculty were advised that the Senate
of the University requires the following message in all course outlines:

“McGill University values academic integrity. Therefore all students must under-
stand the meaning and consequences of cheating, plagiarism and other academic
offences under the Code of Student Conduct and Disciplinary Procedures. (See
http://www.mcgill.ca/integrity for more information).
“L’université McGill attache une haute importance à l’honnêteté académique. Il
incombe par conséquent à tous les étudiants de comprendre ce que l’on entend
par tricherie, plagiat et autres infractions académiques, ainsi que les conséquences
que peuvent avoir de telles actions, selon le Code de conduite de l’étudiant et des
procédures disciplinaires. (Pour de plus amples renseignements, veuillez consulter
le site http://www.mcgill.ca/integrity).”

1.5 Published Materials


1.5.1 Required Text-Book
The textbook for the course is J. Stewart, SINGLE VARIABLE CALCULUS: Early
Transcendentals, Fifth Edition, Brooks/Cole (2003), ISBN 0-534-39330-6, [1].
This book is the first half of J. Stewart, CALCULUS: Early Transcendentals,
Fifth Edition, Brooks/Cole (2003), ISBN 0-534-39321-7, [2]; this edition covers the
material for Calculus III (MATH 222) as well, but is not the text-book for that course
at the present time.

1.5.2 Optional Reference Books


It is recommended that students make use of the student solution manual:

• D. Anderson, J. A. Cole, D. Drucker, STUDENT SOLUTIONS MANUAL


FOR STEWART’S SINGLE VARIABLE CALCULUS: Early Transcen-
dentals, Fifth Edition, Brooks/Cole (2003), ISBN 0-534-39333-0, [3]. This
book is also sold “bundled” with either version of the text book; the bundles are
numbered ISBN 0-534-42976-9 [4] and ISBN 0-534-10307-3 [5].

The publishers of the textbook and Solutions Manual also produce

UPDATED TO October 25, 2004


Information for Students in MATH 140 2003 09: 11

Item # Due Date Details


Precalculus P1 22 Sept 03 May be used as a “practice” assignment for learning
Assignments WeBWorK.
(cf. §1.4.3) P2 6 Oct 03 One of assignments ## P1–P6 must be passed with
75% in order for regular WeBWorK assignments to
count.
..
.
P6 1 Dec 03 Last chance to make WeBWorK count
Regular R1 22 Sept 03 No limit to number of attempts at WeBWorK R1
WeBWorK problems
Assignments R2 29 Sept 03 Same scope as R1, but with limited numbers of at-
(cf. §1.4.4) tempts
R3 6 Oct 03
R4 14 Oct 03 Same scope as R3, but with limited numbers of at-
tempts
..
10% .
R9 17 Nov 03
R10 24 Nov 03 Same scope as R9, but with limited numbers of at-
tempts
BONUS 1 Dec 03 see page 6, limited numbers of attempts
Written W1 with Q1 Download; complete at home; hand in with Q1
Assignments W2 with Q2 Download; complete at home; hand in with Q2
(cf. §1.4.5) W3 with Q3 Download; complete at home; hand in with Q3
W4 with Q4 Download; complete at home; hand in with Q4
5% W5 with Q5 Download; complete at home; hand in with Q5
Quizzes Q0 15–19 Sept 03 Quiz Q0 does not count
(cf. §1.4.6) Q1 22–29 Sept 03
..
15% or 0% .
Q5 17–21 Nov 03
Final Exam 5–19 Dec 03 Date of exam to be announced by Faculty
70% or 85%
Supplemental 3–4 May 04 Only for students who do not obtain standing at the
Exam final. Supplemental exams count in your average like
taking the course again; exam counts for 100%.

Table 5: Summary of Course Requirements, as of October 25, 2004 (all dates are subject
to change)

• a “Study Guide”, designed to provide additional help for students who believe
they require it: R. St. Andre, STUDY GUIDE FOR STEWART’S SIN-

UPDATED TO October 25, 2004


Information for Students in MATH 140 2003 09: 12

GLE VARIABLE CALCULUS: Early Transcendentals, Fifth Edition,


Brooks/Cole (2003), ISBN 0-534-39331-4, [6]. (The “Study Guide” resembles the
Student Solution Manual in appearance: be sure you know what you are buying.)

• a “Companion” which integrates a review of pre-calculus concepts with the contents


of Math 140, including exercises with solutions: D. Ebersole, D. Schattschneider,
A. Sevilla, K. Somers, A COMPANION TO CALCULUS. Brooks/Cole (1995),
ISBN 0-534-26592-8 [26].

1.5.3 Recommended Video Materials


Use of the following materials is recommended, but is not mandatory14 .

Larson/Hostetler/Edwards DVD Disks A set of video DVD disks produced for


another calculus book, [18] Calculus Instructional DVD Program, for use with (inter
alia) Larson/Hostetler/Edwards, Calculus of a Single Variable: Early Transcendental
Functions, Third Edition [19] is produced by the Houghton Mifflin Company. A copy
has been requested to be placed on reserve in the Schulich Library, and the set of 4 disks
is for sale in the bookstore among the reference materials for this course. In Appendix
§I of these notes there are charts that indicate the contents of these disks that pertain
to MATH 140.

Videotapes for Stewart’s Calculus The publisher of Stewart’s Calculus has pro-
duced a series of videotapes, [7] Video Outline for Stewart’s Calculus (Early Transcen-
dentals), Fifth Edition. These will initially be available for reserve loan at the Schulich
Library. There may not be VCR viewing equipment in the library; the intention is that
interested students borrow a tape for viewing on their own equipment at home.

Tools for Enriching Calculus This is a CD-ROM included with new copies of Stew-
art’s Calculus. From [9, Introduction]:“Tools for Enriching Calculus (TEC) enhances a topic
that is covered in the textbook by providing both broader and deeper coverage of those aspects
for which technology is particularly useful. The basic format of most modules is a point-and-
click laboratory environment in which you can easily visualize functions and their derivatives,
experiment with suggested examples and exercises, explore your own choices of examples, and
perhaps even test some of your own conjectures. You need to be a well-prepared and active
player to reap the benefits from these approaches.
14
No one will check whether you have used any of these aids; a student can obtain a perfect grade
in the course without ever consulting any of them. No audio-visual or calculator aid can replace the
systematic use of paper and pencil as you work your way through problems. But the intelligent use of
some of these aids can deepen your understanding of the subject.

UPDATED TO October 25, 2004


Information for Students in MATH 140 2003 09: 13

“First, you need to read the textbook materials carefully to gain an understanding of the
essential ideas. Next, you need to read the introductory material for each TEC module, which
explains the basic mathematical approaches and describes how to use the module. Each module
has several examples which will familiarize you with its basic features. When you have finished
reviewing this material, and have some paper and pencil in hand, you are ready to get the most
benefit from using the module. You can improve your understanding of the topic by exploring
mathematical questions that you find puzzling, and checking your ideas for solutions using the
module. Try to work through some of the exercises in the module to gauge your understanding
of the topic. Be willing to use pencil and paper to first guess what the answer might be before
seeing an electronic graph...
“Another important TEC feature is the homework15 hints. Hints have been created for
several selected exercises in each section of your textbook to help you understand some key
points in finding solutions for these exercises. Similar to a good instructor or teaching assistant,
these hints ask you questions that will allow you to make progress toward a solution without
giving you the actual answer. You need to actively pursue each hint with pencil and paper and
fill in many of the computations and details. If you can complete the solution after reading
only one or two hints, you can feel proud of your achievements. If you still have questions after
completing all of the hints for a problem, your work should help you to better understand the
solution presented in the Student Solutions Manual.”

Interactive Video Skillbuilder CD for Stewart’s Calculus: Early Transcen-


dentals, 5th Edition [8] This CD-ROM is included with new copies of the textbook.
It contains, after an enlightening “pep-talk” by the author, a discussion of some of the
worked examples in the text-book, followed by a quiz for each section in the book. Some
students may find the animations of the examples helpful, although the examples are all
worked in the book. You might wish to try some of the quiz questions using paper and
pencil, and then check your answers with those given on the CD. It is not recommended
that you attempt to enter your answers digitally, as this is a time-consuming process,
and uses a different input method from your WeBWorK assignments, which serve the
same purpose.

1.5.4 Other Calculus Textbooks


While students may wish to consult other textbooks, instructors and teaching assistants
in Math 140 will normally refer only to the prescribed edition of the prescribed textbook
for the course. Other books can be very useful, but the onus is on you to ensure that
your book covers the syllabus to at least the required depth; where there are differences
15
Note that homework refers to odd-numbered problems in the exercises in [1], not to problems on
WeBWorK .

UPDATED TO October 25, 2004


Information for Students in MATH 140 2003 09: 14

of terminology, you are expected to be familiar with the terminology of the textbook.16
In your previous calculus course(s) you may have learned methods of solving problems
that appear to differ from those you find in the current textbook. Your instructors will
be pleased to discuss any such methods with you personally, to ascertain whether they
are appropriate to the present course. In particular, any methods that depend upon
the use of a calculator, or the plotting of multiple points, or the tabulation of function
values, or the inference of a trend from a graph should be regarded with scepticism.

1.5.5 Website
These notes, and other materials distributed to students in this course, will be accessible
at the following URL:

http://www.math.mcgill.ca/brown/math140a.html

The notes will be in “pdf” (.pdf) form, and can be read using the Adobe Acrobat reader,
which many users have on their computers. This free software may be downloaded from
the following URL:
17
http://www.adobe.com/prodindex/acrobat/readstep.html

The questions on some old examinations will also be available as an appendix to these
notes on the Web.18 It is expected that most computers in campus labs should have the
necessary software to read the posted materials.
Where revisions are made to distributed printed materials — for example these in-
formation sheets — we expect that the last version will be posted on the Web.
The notes and WeBWorK will also be available via a link from the WebCT URL:

http://webct.mcgill.ca

but other features of WebCT19 have not yet been implemented.


16
There should be multiple copies of the textbook on reserve in the Schulich library.
17
At the time of this writing the current version appears to be 6.0.
18
There is no reason to expect the distribution of problems on quizzes or in assignments and exami-
nations from previous years be related to the frequencies of any types of problems on the examination
that you will be writing at the end of the term.
19
cf. Appendix H to these notes , p. 4001

UPDATED TO October 25, 2004


Information for Students in MATH 140 2003 09: 15

1.6 Syllabus
In the following list section numbers refer to the text-book [1]. The syllabus will include
all of Chapters 1, 2, 3, 4, with omissions, as listed below.20

Chapter 0: A Preview of Calculus. This is motivational material, and may not all
be discussed in the lectures. Read it.

Chapter 1: Functions and Models. In §1.1, two of the four ways to define a function
are not useful in general: a table of values can be used to define a function only if
its domain is finite, and the value of the function is prescribed for every point in
the domain; it is normally not acceptable to define a function by a graph, unless
the nature of the graph can be described without any ambiguity. §1.2 may not be
discussed in the lectures, but you should read it — in particular the definitions of
various kinds of functions — as this terminology may be used from time to time.
Omit §1.4.

Mathematical Induction (new for 2003/2004) You are expected to understand


the Principle of Mathematical Induction [1, p. 81], which will be required in proving
theorems and solving problems that involve the positive integers (=“the natural
numbers”).

Chapter 2: Limits and Derivatives. §2.4 will be discussed in the lectures, but will
not be examination material; read it! The subsection of §2.6 called “Precise Defi-
nitions” should also be read, but will not be examination material.

Chapter 3: Differentiation Rules. Omit §3.3; but you are encouraged to read the
parts of §3.3 that pertain to your own fields of interest.

Chapter 4: Applications of Differentiation. In §4.5 you may skip the discussion of


“slant asymptotes” [1, bottom p. 322 – p. 323]. Omit §4.6, §4.8, §4.9.

Exercises that require technology Students are not expected to be able to solve
exercises that require the use of calculators or computers. You may wish to try
20
If a textbook section is listed below, you should assume that all material in that section is examina-
tion material even if the instructor has not discussed every topic in his lectures; however, the instructors
may give you information during the term concerning topics that may be considered subsidiary.
Do not assume that a topic is omitted from the syllabus if it has not been tested in a
WeBWorK assignment or a quiz, or if it has not appeared on any of the old examinations
in the course! Some topics to not lend themselves to this type of testing; others may have been
omitted simply because of lack of space, or oversight. By the same token, you need not expect every
topic in the course to be examined on the final examination.

UPDATED TO October 25, 2004


Information for Students in MATH 140 2003 09: 16

such problems, as a challenge, as some of them can be solved with clever use of
paper and pencil.

Problems Plus The exercises and other material that appear in [1, Principles of Prob-
lem Solving, pp. 80-85], and in the “Problems Plus” subsections following the later
chapters are to be omitted.
The following appendices in the textbook contain some prerequisite material for this
course:
Appendix A: Intervals, Inequalities, and Absolute Values. (see information for
Chapters 1 and 3 above)

Appendix B: Coordinate Geometry and Lines.

Appendix C: Graphs of Second-Degree Equations. You are expected to be famil-


iar with the material concerning the circle, [1, pp. A16-A17]. The remainder of the
material should be familiar to most students, but will not be assumed.

Appendix D: Trigonometry. You are assumed to be familiar with the material in [1,
pp. A24–A31].
Please do not ask the tutors to provide information as to which topics should be
emphasized. Unless you are informed otherwise by the instructors in the lecture sections
or published notes — printed, or mounted on the Web — you should assume that all
materials listed are included in the syllabus. You are not expected to be able to reproduce
proofs of the theorems in the textbook.

1.7 Preparation and Workload


1.7.1 Prerequisites.
While a course in high school calculus is the normal “prerequisite” for this course,
stronger students having only the prerequisite for MATH 139 (viz., “a course in func-
tions”) have normally been permitted to register in MATH 140. All students must have
completed such a course in functions — in particular, familiarity with trigonometric
functions is assumed; it is your responsibility as a student to verify that you have this
necessary background to benefit from this course. Some of the prerequisites are reviewed
in Appendices A, B, C, D of your textbook [1]; solutions to odd-numbered problems can
be found in the Student Solution Manual [3].
Because weakness in pre-calculus topics can contribute to failure of students in MATH
140 or MATH 141, students will be tested on prerequisites in both the Precalculus
WeBWorK assignments and quiz Q0. Students who do not believe they can remedy

UPDATED TO October 25, 2004


Information for Students in MATH 140 2003 09: 17

deficiencies on their own might wish to consider MATH 11221 . MATH 112 2003 09 is
scheduled at the same time as Section 1 of MATH 140 2003 09.

Precalculus WeBWorK Assignments and Quiz #Q0 In order to help you de-
termine whether your progress in this course could be jeopardized by poor preparation,
you are being asked to complete a Precalculus WeBWorK Assignment, and you will
be given a Quiz (#Q0) at your tutorial during the week of September 15th, 2003. The
Precalculus Assignment must be passed with a grade of 75% or better22 , and the Quiz
grade will not count at all. You are urged not to minimize the significance of these
diagnostic procedures, as poor preparation is a common cause of failure in Calculus I
and Calculus II.
These tests of preparation are concerned with “precalculus” notions, not with calculus
material: while you are expected to have taken a previous course in calculus, we do not
plan to enforce or evaluate that prerequisite.

1.7.2 Calculators
The use of calculators is not permitted in either quizzes or the examination in this course.
Students whose previous mathematics courses have been calculator-oriented would be
advised to make particular efforts to avoid the use of a calculator in solving problems
in this course, in order to develop a minimal facility in manual calculation. This means
that you are urged to do all arithmetic by hand.

1.7.3 Self-Supervision
This is not a high-school course, and McGill is not a high school. The monitoring of
your progress before the final examination is largely your own responsibility. While the
tutors and instructors are available to help you, they cannot do so unless and until you
identify the need for help. WeBWorK and quizzes are designed to assist you in doing
this.

Time Demands of your Other Courses. Be sure to budget enough time to attend
lectures and tutorials, for private study, and for the solution of many problems. Don’t be
tempted to divert calculus study time to courses which offer instant gratification. While
the significance of the tutorial quizzes in the computation of your grade is minimal, these
21
MATH 112 Fundamentals of Mathematics (3) (Fall. Not open to students who have taken CEGEP
course 201-101. Open only to those students who are deficient in a pre-calculus background.) Equations
and inequalities, graphs, relations and functions, exponential and logarithmic functions, trigonometric
functions and their use, mathematical induction, binomial theorem, complex numbers.
22
in order for your WeBWorK grades to count; otherwise your WeBWorK grade will be 0.

UPDATED TO October 25, 2004


Information for Students in MATH 140 2003 09: 18

are important learning experiences, and can assist you in gauging your progress in the
course. This is not a course that can be crammed for: you must work steadily through
the term if you wish to develop the facilities needed for a strong performance on the final
examination.

Working Problems on Your Own. An effective way to master the calculus is


through working large numbers of problems from the textbook. Your textbook was
selected partly because of the availability of an excellent Student Solutions Manual [3];
this manual has brief but complete solutions to most of the odd-numbered exercises in
the textbook. The skills you acquire in solving textbook problems could have much more
influence on your final grade than either WeBWorK or the quizzes.

The real uses of WeBWorK and the quizzes. Students often misunderstand the
true significance of WeBWorK assignments and the quizzes. While both contribute
to your grade, they help you estimate the quality of your progress in the course. Take
proper remedial action if you are obtaining low grades on quizzes23 , or if you require
many attempts before being able to solve a problem on WeBWorK. However, while
both WeBWorK and the quizzes have a role to play in learning the calculus, neither
is as important as reading your textbook, working problems yourself, and attending and
listening at lectures and tutorials.

1.7.4 Escape Routes


At any time, even after the last date for dropping the course, students who are experi-
encing medical or personal difficulties should not hesitate to consult their advisors or the
Student Affairs office of their faculty. Don’t allow yourself to be overwhelmed by such
problems; the University has resource persons who may be able to help you.

1.7.5 Terminology
Do not be surprised if your instructors and tutors use different terminology from what
you have heard in your previous calculus course, particularly if that course was at a high
school. Sometimes the differences are purely due to different traditions in the professions.

“Negative x” Your instructors and tutors will often read a formula −x as minus x,
not as negative x. To a mathematician the term negative refers to real numbers which
are not squares, i.e. which are less than 0, and −x can be positive if x itself is negative.
23
The worst action is to miss the quizzes, and thereby block out an unwelcome message.

UPDATED TO October 25, 2004


Information for Students in MATH 140 2003 09: 19

However, mathematicians will sometimes refer to the operation of changing a sign


as the replacement of x by “its negative”; this is not entirely consistent with the usual
practice, but is an “abuse of language” that has crept into the professional jargon.

Inverse trigonometric functions A formula like sin−1 x will be read as the inverse
sine of x — never sine to the minus 1 or sine to the negative 1 . However, if we write
sinn x, where n is a positive integer, it will always mean (sin x)n . These conventions
apply to any of the functions sin, cos, tan, cot, sec, csc; they also apply to the hyperbolic
functions, which we will meet in [1, §3.9]: sinh, cosh, tanh, coth.... We will usually not
write exponents on general functions, so a formula like f 2 (x) does not have an obvious
meaning, and we will avoid writing it when f is other than a trigonometric or hyperbolic
function.

Logarithms These days mathematicians rarely use logarithms to the base 10. If you
were taught to interpret log x as being the logarithm to base 10, you should now forget
that. Most often, if your instructor speaks of a logarithm, and writes log x, he will be
referring to the base e, i.e. to loge ; that is, he is referring to the function that calculus
books call ln. When a logarithm to some other base is intended, it will either be denoted
by an explicit subscript, as log2 , or some comment will be made at the beginning of
the discussion, as “all logarithms in this discussion are to the base 2”. Your instructors
try to think like mathematicians even when lecturing to their classes, and so we use the
language and terminology we use when talking to each other.

1.8 High and Low Technology and MATH 140


1.8.1 Keep your e-mail address up to date
Both WebCT and WeBWorK contain an e-mail address where we may assume you can
be reached. If you prefer to use another e-mail address, the most convenient way is to
forward your mail from your student mailbox, leaving the recorded addresses in these
two systems unchanged. You can enter or change a forwarding e-mail address by going
to http://webmail.mcgill.ca, and logging in to your student mailbox at po-box.mcgill.ca.

1.8.2 Use of Calculators and Computer Algebra Systems


Insofar as course content is concerned, we will emphasize the lowest possible use of
technology: we will avoid the use of calculators and computers. You are urged to do all
calculations manually, and to avoid the use of computer algebra systems, until you have
completed MATH 141. You should not use a calculator or computer in the solution of

UPDATED TO October 25, 2004


Information for Students in MATH 140 2003 09: 20

WeBWorK problems, as it prevents you from developing skill for detecting errors in
manual calculations — a skill that you will need for the quizzes and final examination.

1.8.3 Use of the Internet


However, insofar as course delivery is concerned, students are expected to be able to
access materials through the Internet, whenever required. Here are some of the uses that
are expected:

• To access and submit Precalculus (§1.4.3) and Regular WeBWorK (§1.4.4) as-
signments

• To access the individualized problems for a student’s Written Assignments

• To access WebCT, where course grades and announcements will be mounted

• To access the web page for the course — also available through WebCT — where
these and other notes will be available in .pdf form; the site also contains notes
and examinations from previous years.

1.9 Special Review Sessions


Tutorials not held regularly on Mondays hold their last sessions during the week 25-28
November, 2003; Tutorial sections T003–T008 will meet on Monday, December 1st, 2003
(replacing the Monday tutorial lost on Thanksgiving Day). The following review sessions
are planned at the end of the term:

(subject to revision)
Date Instructor Location Starts Ends
Dec. 3, 2003 X. Liu BURN 1204 13:35 14:25
Dec. 3, 2003 X. Liu BURN 1204 14:35 15:25
Dec. 3, 2003 X. Liu BURN 1204 15:35 16:25
Dec. 5, 2003 M. Alakhrass BURN 920 14:00 17:00

UPDATED TO October 25, 2004


Information for Students in Lecture Section 1 of MATH 140 2003 09 1001

A Information Specifically for Students in Lecture


Section 001
A.1 Timetable for Lecture Section 001 of MATH 140 2003 09
Distribution Date: (0th version) Wednesday, September 3rd, 2003
(All information is subject to change.)

MONDAY WEDNESDAY FRIDAY


SEPTEMBER
1 LABOUR DAY 3 §1.1; §1.2° R 5 §1.3; OMIT §1.4
Tutorials begin the week of September 8th
Use Textbook Appendices A, B, C, D to review prerequisites!
8 §§1.5, 1.6 10 §1.6 12 §2.1, §2.2
Course changes must be completed by midnight, September 14
15 Q0 17 §2.3Q0 19 §2.4Q0
Deadline for withdrawal with fee refund = September 21
22 §2.5Q1 R1 24 §2.5, §2.6Q1 26 §2.6, §2.7Q1
29 §2.7, §2.8R2
OCTOBER
1 §2.9 3 X
6 §3.1R3 Q2 8 §3.2Q2 10 §3.2, §3.3Q2
Deadline for withdrawal (with W) from course = Oct. 12
13 THANKSGIVING 15 §3.4R4 17 §3.5
DAY (Canada):
Tutorial Sections
003-008 moved to
December 1
20 §3.6Q3 R5 22 §3.7Q3 24 §3.7, §3.8Q3
27 §3.8, §3.9R6 29 §3.9, §3.10 31
Notation:
Rn = Regular WeBWorK Assignment #Rn due at midnight on Monday this week
°R = Read Only
Qn = Quiz #Qn planned for the tutorials this week
Hand in Written Assignment #Wn (There is no #W0.)
X = reserved for eXpansion or review
Section numbers refer to the text-book.

The next page will not be distributed until the syllabus has been revised.

UPDATED TO October 25, 2004


Information for Students in Lecture Section 1 of MATH 140 2003 09 1002

MONDAY WEDNESDAY FRIDAY


NOVEMBER
3 §3.11, XQ4 R7 5 §4.1Q4 7 §4.1Q4
10 §4.2R8 12 §4.3 14 §§4.3,4.4
17 §4.4)Q5 R9 19 §4.5 (omit “slant 21 §4.5 (omit “slant
asymptotes”)Q5 asymptotes”)Q5
24 §4.7R10 26 §4.7 28 §4.10
DECEMBER
Only Tutorial Sections 003–008, 015, 016 meet week of December 1
1 X Tutorial Sections 3 X
003-008 meet at their
regular times.
Notation:
Rn = Regular WeBWorK Assignment #Rn due at midnight on Monday this week
°R = Read Only
Qn = #nth quiz planned for the tutorials this week
Hand in Written Assignment #Wn
X = reserved for eXpansion or review
Section numbers refer to the text-book.
Timetable for Lecture Section 001, November - December, 2003

UPDATED TO October 25, 2004


Supplementary Notes for Students in Lecture Section 001 1003

B Supplementary Notes for Lectures in Lecture Sec-


tion 001
Notes Distributed to Students in Lecture Section 2 of MATH 140 2003 09) 2001

C Information Specifically for Students in Lecture


Section 002
C.1 Timetable for Lecture Section 002 of MATH 140 2003 09
Distribution Date: revised Thursday, October 9th, 2003
(All information is subject to change.)

MONDAY WEDNESDAY
SEPTEMBER
1 LABOUR DAY 3 §1.1, §1.2°, R §1.3
Tutorials begin the week of September 8th
Use Textbook Appendices A, B, C, D to review prerequisites!
8 §1.5, §1.6 10 §2.1, §2.2
Course changes must be completed by midnight, September 14
15 §2.3, §2.4Q0 17 §2.5Q0
Deadline for withdrawal with fee refund = September 21
22 §2.6R1 Q1 24 §2.7, §2.8Q1
29 §2.9R2
OCTOBER
1 §3.1, §3.2
6 NO LECTURE IN LECTURE 8 §3.3, §3.4Q2
SECTION 2 Q2 R3
Deadline for withdrawal (with W) from course = Oct. 12
13 THANKSGIVING DAY 15 §3.4, §3.5R4
(Canada): Tutorial Sections
003-008 moved to December 1
20 §3.6Q3 R5 22 §3.7Q3
27 §3.8, §3.9R6 29 §3.10
Notation:
Rn = Regular WeBWorK Assignment #Rn due at midnight on Monday this week
°R = Read Only
Qn = Quiz #Qn planned for the tutorials this week
Hand in Written Assignment #Wn (There is no #W0.)
X = reserved for eXpansion or review
Section numbers refer to the text-book.

The next page will not be distributed until the syllabus has been revised.

UPDATED TO October 25, 2004


Notes Distributed to Students in Lecture Section 2 of MATH 140 2003 09) 2002

MONDAY WEDNESDAY
NOVEMBER
3 §3.10, §3.11Q4 R7 5 §4.1Q4
10 §4.2R8 12 §4.3
17 §4.4Q5 R9 19 §4.5 — omit “slant asymptotes”,
(§4.6)Q5
24 §4.7R10 26 §4.10
DECEMBER
Only Tutorial Sections 003–008, 015, 016 meet week of December 1
1 X Tutorial Sections 003-008 meet at 3 X
their regular times.
Notation:
Rn = Regular WeBWorK Assignment #Rn due at midnight on Monday this week
°R = Read Only
Qn = #nth quiz planned for the tutorials this week
Hand in Written Assignment #Wn
X = reserved for eXpansion or review
Section numbers refer to the text-book.
Timetable for Lecture Section 002, November - December, 2003
Supplementary Notes for Students in Lecture Section 002 2003

D Supplementary Notes for Lectures in Lecture Sec-


tion 002
Release Date: Wednesday, September 3rd, 2003

D.1 Supplementary Notes for the Lecture of September 3rd,


2003
Lecture Content A timetable posted on the Web will show you approximately what
is to be discussed at each lecture. It is suggested that you look through the material in
advance. If you have time to try some of the exercises, and find some that cause you
difficulty, you are welcomed to bring them to Professor Brown’s attention; perhaps he
will be able to work some of these examples into the lecture.

What goes on the chalkboard? — Should I Take Notes? Your instructor be-
lieves strongly that students should not spend the lecture hour feverishly copying notes
for fear of missing some essential topic; in this course most of what you need to know is
contained in the textbook. You should take notes, but you should be trying to think at
the same time. The chalkboard will be used for

• statement/illustration of specific definitions and theorems

• sketching solutions to problems, or classes of problems

• a scratchpad

Some of this material will be useful to you in learning the material in the course. Even
when the material on the board is equivalent to something in your textbook, the act of
writing may help you remember it.

Graphs Several topics in the syllabus, culminating in [1, §4.5], are concerned with
sketching of curves. Our emphasis here is on qualitative and quantitative properties of
the graphs of functions, but not on the production of extremely precise graphs. You can
expect to see the instructor draw on the chalkboard sketches that are extremely crude
approximations of functions, sometimes even caricatures of the true graph. Mathemati-
cians do not base proofs on sketches of graphs — the role of a sketch is usually only
to assist the reader to visualize the verbal or symbolic reasoning which accompanies it.
Sometimes a graph is used help one discover a phenomenon, but the result would not be
acceptable to a mathematician unless it could be proved in a non-graphical way.
Information for Students in MATH 140A 2003 09 3001

E Assignments from Previous Years


E.1 Fall 1998 Problem Assignments
(For most of these problems the answer was in the back of the (then) text-book [21][23],
but the full solution was not in the Student Solution Manual [22][24].)

Assignment Due Exercise Numbers


Number Date
1 25 Sept./98 Chapter 2 Miscellaneous Problems: 2,
5, 9, 12, 17, 21, 26, 28, 32, 35, 38, 40,
50, 56, 57, 59, 61
2 9 Oct./98 §3.1: 11, 17, 20, 36, 41
§3.2: 2, 5, 9, 15, 21, 33, 45, 48, 53
§3.3: 14, 15, 17, 44, 50, 53
3 23 Oct./98 §3.4: 45, 50
§3.5: 2, 6, 9, 21, 27, 45
§3.6: 21, 26, 32
§3.7: 62, 68, 74
4 6 Nov./98 §3.8: 17, 21, 26, 29, 35, 47
(deferred to §3.9: 3, 8, 11, 23, 27, 50, 51, 59
9 Nov.) Chapter 7 Miscellaneous Problems: 3,
11, 17, 21
Chapter 8 Miscellaneous Problems: 5, 9
5 23 Nov./98 §4.2: 3, 8, 13, 20, 26, 40, 41, 42
§4.3: 15, 23, 29, 33
§4.4: 5, 11, 15, 30
§4.5: 21, 32
§4.6: 31, 49
§4.7: 2, 6, 9, 15, 29, 47

Table 6: 1998 Problem Assignments

E.2 Fall 1999 Problem Assignments


E.2.1 First Fall 1999 Problem Assignment, with Solutions
Students were advised as follows in the problem set and its solutions:

UPDATED TO October 25, 2004


Information for Students in MATH 140A 2003 09 3002

• This assignment is intended to help you determine gaps in your background. It


will not be graded, but the solutions will be made available on the Web. If you
have difficulty with the problems or the solutions which will be posted, you should
discuss these with one of the tutors.

• Do not use a calculator when solving these problems, even for simple arithmetic.
(You may, however, wish to use a calculator afterwards to verify whether your
answers are ‘reasonable’.)

• Do not attempt to approximate square roots, or π. Where possible, however,


formulæ should always be simplified .

• Unless specifically stated in degrees, all angles should be assumed to be expressed


in radians. (Remember that the straight angle, 180◦ is equal to π radians, so an
π πd
angle of 1◦ is equal to 180 radians, and d◦ is equal to 180 radians.)

• Caveat lector! As in any duplicated materials, there could be misprints or errors.

1. (a) Give a right-angled triangle in which one angle is α = π3 , and use the triangle
to determine the values of sin α, cos α, tan α.
(b) Give a right-angled triangle in which one angle is β = π4 , and use the triangle
to determine the values of sin β, cos β, cot β, sec β.
It is not sufficient to state the values: you should explain how you determine them
in terms of the lengths of the sides of your triangle.
Solution:

(a) We take the right half of an equilateral triangle, each of whose sides has length
2. In ∆DBC ∠C = π3 = α. DB bisects ∠ADC and meets AC in its midpoint,
B. Then

|BD| 3
sin α = =
|CD| 2
|BC| 1
cos α = =
|CD| 2

|BD| 3 √
tan α = = = 3
|BC| 1

(b) Here we take an isosceles ∆EF G, in which EF = EG = 1, and ∠F = 45◦ = β.

|EG| 1
sin β = =√
|F G| 2

UPDATED TO October 25, 2004


Information for Students in MATH 140A 2003 09 3003

D
·T
· T
· T
· T
· T
· T
· √ T
F @
2 · 3 T 2
· T @
· T @
· T @ √
· T 1 @
2
· T @
· T @
· T @
· 1 90◦ 1 60◦T 90◦ 45@

· T 1 @
A B C E G

|EF | 1
cos β = =√
|F G| 2
|EF |
cot β = =1
|EG|

|F G| 2 √
sec β = = = 2
|EF | 1

(Note: Some students may have been taught never to leave a surd in the
denominator, as in the fraction √12 above. This convention derives from the

difficulties, in the days before calculators, of working with numbers like 2.
While it still is useful, for hand calculations, to confine surds to the numerator,
you may work with fractions like √12 if you are happy with them.)

2. State, without proof,

(a) a formula which expresses sin(x + y) in terms of the sines and cosines of x
and y.
(b) a relationship between sin x and sin(−x), and another between cos x and
cos(−x).
(c) By applying the results of parts 2b, 2a above, determine a formula that ex-
presses sin(x − y) in terms of the sines and cosines of x and y.

UPDATED TO October 25, 2004


Information for Students in MATH 140A 2003 09 3004

The formulæ you state should be valid for all values of x and y.
Solution:

(a)
sin(x + y) = sin x cos y + cos x sin y (60)
[21, A-15 (6)]
(b) sin(−x) = − sin x; cos(−x) = − cos x [21, A-15 (4)]
(c) Replacing y by −y throughout (60), one obtains

sin(x − y) = sin x cos(−y) + cos x sin(−y)


= sin x cos y + cos x(− sin y)
= sin x cos y − cos x sin y (61)

π π
3. (a) By substituting x = α = 3
and y = β = 4
in the formula in 2c, determine
the value of sin 15◦ .
π
(b) Using the result of 3a, determine the value of cos 12 .
Solution:

(a)
µ ¶
π π
sin = sin − i4
12 3 p
π π π π
= sin cos − cos sin
à √3 ! µ 4 3
¶ µ ¶ µ
4

3 1 1 1
= · √ − · √
2 2 2 2
√ √ √
3−1 6− 2
= √ = .
2 2 4
√ √ √
( 3−1)2
(b) π
cos2 12=1− 8
= 2+4 3 ; hence cos 12
π
is one of the square roots of 2+4 3 .
Since theq
angle is in the first quadrant, the sign is positive. It follows that

π 2+ 3
cos 12 = 4
. This can be seen — you were not expected to see this —

3+1
that the square root is √
2 2
.

4. State, without proof, formulæ which express cos(x + y) and cos(x − y) in terms of
the sines and cosines of x and y.

UPDATED TO October 25, 2004


Information for Students in MATH 140A 2003 09 3005

Solution:

cos(x + y) = cos x cos y − sin x sin y (62)


cos(x − y) = cos x cos y + sin x sin y (63)

[21, A-15 (7)]

5. (a) Specialize the formula in Problem 4 (by making a suitable choice for y in
terms of x) to express cos 2x in terms of sin x and cos x.
(b) Use the identity sin2 x + cos2 x = 1 to express cos 2x in terms of cos x alone;
and in terms of sin x alone.
(c) Apply your formulæ in 5b to determine the values of cos π6 and cos π2 from the
π
sines of 12 and π4 .
Solution:

(a) Taking y := x in (62) yields

cos 2x = cos2 x − sin2 x (64)

(b)

cos 2x = cos2 x − (1 − cos2 x) = 2 cos2 x − 1 (65)


= (1 − sin2 x) − sin2 x = 1 − 2 sin2 x (66)

(c)
π ³π´ ³π´
2
cos = cos 2 = 1 − 2 sin
6 12√ 12 √ √
2
( 3 − 1) 1+3−2 3 3
= 1−2· =1−2· =
8 8 2
µ ¶2
π π 1
cos = 1 − 2 sin2 = 1 − 2 √ =0
2 4 2

6. Find an equation for each of the following lines in the plane:

(a) The line through the points (−1, 4) and (−2, 1).
(b) The line through the point (4, −1) which is perpendicular to the line

2x + y = −7 . (67)

Solution: (This problem is from the 1998 Final examination in 189-112A.)

UPDATED TO October 25, 2004


Information for Students in MATH 140A 2003 09 3006

1−4
(a) The slope of the line will be (−2)−(−1)
= 3, so an equation is y−4 = 3(x−(−1))
or y = 3x + 7.
(b) The slope of the line we seek will be the negative reciprocal of the line y =
−2x − 7, which has slope −2; thus the line will have slope 21 . One equation
will be y − (−1) = 21 (x − 4), or x − 2y = 6.
7. (a) It is claimed that the equation (2x + y + 7)2 = 0 represents the same points
as the equation 2x + y + 7 = 0. Determine whether the claim is correct.
(b) Determine what is represented by the equation (2x + y)2 = (−7)2 , i.e. by the
equation obtained by squaring both sides of equation (67).
Solution:
(a) The equation of a line is a constraint satisfied by the coordinates of its points,
and satisfied by the coordinates of no other points. As (2x + y + 7)2 may
be zero if and only if 2x + y + 7 = 0, this is another equation for the same
line. (Some authors would, however, say that this is “the equation of two
coincident lines”.)
(b) (The symbol ⇔ means that the statements which it connects are logically
equivalent.)
(2x + y)2 = (−7)2 ⇔ (2x + y)2 − (−7)2 = 0
⇔ (2x + y − 7)(2x − y + 7) = 0
⇔ 2x + y − 7 = 0 or 2x + y + 7 = 0
Thus the given equation is satisfied by the coordinates of the points on either
of the parallel lines 2x + y − 7 = 0 and 2x + y + 7 = 0; the equation represents
the union of the sets of points on the two lines.
8. Determine the centre and radius of the circle x2 + y 2 + 4x − 6y + 3 = 0.
Solution: (This problem is from the 1998 Final examination in 189-112A.) We
complete the squares separately.
x2 + y 2 + 4x − 6y + 3 = 0
⇔ (x2 + 4x) + (y 2 − 6y) + 3 = 0
à µ ¶2 ! µ ¶2 à µ ¶2 ! µ ¶2
4 4 −6 −6
⇔ x2 + 4x + − + y 2 − 6y + − +3=0
2 2 2 2
⇔ (x + 2)2 + (y − 3)2 = 4 + 9 − 3 = 10

⇔ (x − (−2))2 + (y − 3)2 = 4 + 9 − 3 = ( 10)2

Hence the centre of the circle is (−2, 3), and its radius is 10.

UPDATED TO October 25, 2004


Information for Students in MATH 140A 2003 09 3007

9. Simplify completely the following


à √ formula,
!−1 leaving your answer free of negative
√6
3
a3 b2
exponents or radicals: b· √ .
a6 b3
Solution: (This problem is from the 1998 Final examination in 189-112A.)
Ã√ !−1 √

6
3
a3 b2 √
6 a6 b3
b· √ = b· √
3
a6 b3 a3 b2
1 3 1 3 5
b 6 a3 b 2 a3 b 6 + 2 a3 b 3
= 2 = 2 = 2
a1 b 3 ab 3 ab 3
5 2
= a3−1 b 3 − 3 = a2 b

10. Solve the equation


1 1 4
+ = . (68)
x x−2 3
Solution: (This problem is from the 1998 Final examination in 189-112A.) If we
multiply both sides of the equation61 by 3x(x − 2), we obtain the polynomial
equation 3(x − 2) + 3x = 4x(x − 2), which reduces to 4x2 − 14x + 6 = 0, then to
2x2 − 7x + 3 = 0, which is equivalent to (2x − 1)(x − 3) = 0. The product on the
left can be zero only with at least one of the factors is zero, i.e. when either x = 21
or x = 3. These are the only solutions to (68).
11. Determine the “natural” (i.e. largest possible) domain of definition of each of the
following functions. Explain your results.

x
(a) f (x) = .
2 − sin x
1
(b) .
log3 (x + 1)
Solution: (This problem is from the 1998 Final examination in 189-112A.)
61
We should verify, when we have found the alleged solutions to the equation, that this operation of
multiplying both sides by the same quantity did not entail the introduction of any values which were
not solutions. For example, if we had multiplied by 3x2 (x − 2), the subsequent computations would have
produced a 3rd degree equation whose solutions would be 0, 12 and 3 — but x = 0 is evidently not a
solution, since the left side is not even defined there. Similarly, if we had multiplied by 3x(x − 2)(x + 11),
we would have obtained x = 11 as one of the solutions of the polynomial equation, but it is certainly
not a solution of the original equation. While the procedure could be made more rigorous, the safest
policy is to verify that all claimed solutions are indeed solutions of the original equation — particularly
when, in the course of your solution, you have multiplied both sides of an equation by an expression
that could be 0.

UPDATED TO October 25, 2004


Information for Students in MATH 140A 2003 09 3008

(a) The denominator is defined for all x. The numerator is defined for all non-
negative x. The ratio function is defined only for non-zero denominators,
so we must ensure that 2 − sin x is not zero; but this can never happen, as
| sin x| ≤ 1. Thus the largest possible domain for this function is the set of all
non-negative real numbers.
(b) Logarithms are defined only for positive numbers, so we must require that x +
1 > 0, i.e. that x > −1. But the logarithm here appears in the denominator,
which can assume any value except 0. log3 (x+1) = 0 precisely when x+1 = 1,
i.e. x = 0. Thus the largest possible domain for this function is the union of
the set −1 < x < 0 with the set x > 0 of positive real numbers.

12. Factorize the following polynomial completely:

(x2 + 4xy + 4y 2 ) − (3x + 6y)

Solution: (This problem is from the 1998 Final examination in 189-112A.) Students
were expected to observe that the first summand is a perfect square.

(x2 + 4xy + 4y 2 ) − (3x + 6y) = (x + 2y)2 − 3(x + 2y)


= (x + 2y)(x + 2y − 3)

is the desired factorization.

13. Solve the system of equations:

3x − 7y = 1
4x + 3y = 5

Solution: (This problem is from the 1998 Final examination in 189-112A.) Students
will study the systematic solution of systems of linear equations in courses on
linear algebra and matrices. This problem is simply to detect whether you are
able to solve a routine small system, even if your methods are not systematic.
Subtracting 3 times the second equation from 4 times the first equation yields
11
−37y = −11, implying that, if there is a solution, y = 37 ; this, substituted into
38
the first equation, yields x = 37 . (This solution can be verified by substituting
in the second equation. While that is hardly necessary in the present problem,
substitution should be undertaken in large linear systems, unless care has been
taken to avoid the possibility that some constraint has been lost. This is beyond
the present course, but will be studied in courses in linear algebra. What is at
issue is the possibility that a system of equations may have no solutions at all, or
may have infinitely many solutions.)

UPDATED TO October 25, 2004


Information for Students in MATH 140A 2003 09 3009

14. Find all solutions to the equation

log2 (3x + 2) + log2 (x + 1) = 2 (69)

Solution: (This problem is from the 1998 Final examination in 189-112A.)

log2 (3x + 2) + log2 (x + 1) = 2


⇒ log2 ((3x + 2)(x + 1)) = 2 = log2 22
⇔ 3x2 + 5x + 2 = 4
⇔ 3x2 + 5x − 2 = 0
⇔ (3x − 1)(x + 2) = 0
1
⇔ x= or x = −2
3
But the value x = −2 is extraneous, as the logarithm is not defined at −2 + 1 =
−1 < 0, or at 3 · (−2) + 2 = −4 < 0. We conclude that the only possible
solution to (69) is x = 31 , and verify that it does indeed satisfy the equation, as
log2 3 + log2 34 = log2 4 = log2 22 = 2.

15. Determine all real numbers x such that

(a) 5 tan2 x − sec2 x = 11


5
(b) cos x + sec x = 2

Solution:

(a)

5 tan2 x − sec2 x = 11 ⇔ 5 tan2 x − (tan2 x + 1) = 11


⇔ 4 tan2 x = 12

⇔ tan x = ± 3

The smallest solutions in absolute value are x = ± π3 . From the period-


icity
© of the tangent
ª function we conclude that the set of all solutions is
1
(n ± 3 )π : n ∈ Z where Z is the set of all integers.
(b)
5 1 5
cos x + sec x = ⇔ cos x + =
2 cos x 2
2
⇔ 2 cos x − 5 cos x + 2 = 0
⇔ (2 cos x − 1)(cos x − 2) = 0

UPDATED TO October 25, 2004


Information for Students in MATH 140A 2003 09 3010

1
⇔ cos x = or cos x = 2
2
1
⇒ cos x = as | cos x| ≤ 1
µ 2 ¶
1
⇔ x = 2n ± π
3

16. Show that


1 x2 − x + 1
≤ 2 ≤3 (70)
3 x +x+1
for any real number x.
Solution: Let us first prove the inequalities
x2 + x + 1
≤ x2 − x + 1 ≤ 3(x2 + x + 1)
3
i.e. equivalently, the two inequalities

x2 + x + 1 ≤ 3(x2 − x + 1)
x2 − x + 1 ≤ 3(x2 + x + 1)

The first of these is equivalent to 2(x − 1)2 ≥ 0, which is true because the left side
is a square, hence non-negative; the second is equivalent to 2(x + 1)2 ≥ 0, which is
non-negative for the same reason. In order to pass from these two inequalities to
(70) we need to divide by x2 + x + 1. This is a quadratic polynomial, having no
real roots; its sign is that of the leading coefficient x2 , i.e. it is positive; this can
be shown by observing that x2 + x + 1 = (x + 12 )2 + 34 ≥ 0 + 34 > 0. So dividing
the members of an inequality by this positive quantity preserves the inequalities,
thereby yielding the desired pair of inequalities.
2
A more elegant solution can be found if we define z = xx2 +x+1
−x+1
, and transform this
equation to yield a quadratic equation for x in terms of z:

x2 − x + 1 − z(x2 + x + 1) = 0
⇔ (1 − z)x2 − (1 + z)x + (1 − z) = 0

For this equation to admit a solution for every x, the discriminant must be non-
negative, i.e.
(1 + z)2 − 4(1 − z)2 ≥ 0
This is equivalent to

[(1 + z) − 2(1 − z)][(1 + z) + 2(1 − z)] ≥ 0

UPDATED TO October 25, 2004


Information for Students in MATH 140A 2003 09 3011

¡ ¢ ¡ ¢
which is equivalent to − z − 13 (z − 3) ≥ 0. For the product z − 31 (z − 3) to
be non-positive, z must lie between the roots, i.e. z must be such as to make one
of the factors negative and the other positive. This can be achieved only with
1
3
≤ z ≤ 3, as required.

E.2.2 Second Fall 1999 Problem Assignment


1. Determine if lim f (x), lim f (x), and/or lim f (x) exist for each of the following
x→1+ x→1− x→1
functions, and evaluate those that do exist.

(a) f (x) = x2 + x − 2
½ 2
x − 3 for x ≤ 1
(b) f (x) =
2 − x for x > 1
½ √
2 − x for x < −3
(c) f (x) = 3
x − x for 2 > x ≥ −3
2. In each of the following cases, evaluate the limit, or show that it does not exist.
µ ¶
1 1
(a) lim −
x→3 x2 − x − 6 x − 3
µ ¶
5 1
(b) lim −
x→3 x2 − x − 6 x − 3
62
3. In each of the following cases, evaluate the limit, or show that it does not exist.

3x + 2 x
(a) lim
x→∞ 1−x
|2x − 3|
(b) lim
x→−∞ x + 2

1
x+
4. Evaluate lim x2
x→0 2
− 3x2
x2
63
5. Find a value for a so that
µ ¶
x2 + 1
lim 3x + 1 − a
x→∞ x+1
exists as a finite limit, and evaluate that limit.
62
Added 21.09.99: This problem should be omitted. It is based upon [21, §§4.5,4.7], which material
may not have been discussed in the lectures before the due date of the assignment.
63
See footnote 62.

UPDATED TO October 25, 2004


Information for Students in MATH 140A 2003 09 3012


 2 when x ≤ −2
6. Sketch the graph of f (x) = x + 4 when −2 < x < 4 .

20 − x2 when 4≤x
Find the values of x for which f (x) is not continuous.
7. Find values of a and b which will make the following function continuous:
 2
 x + 2a when 0 < x < 1

f (x) = bx + a when 1 ≤ x < 2 .
 2b − 3 when

2≤x
x
x2 − 2x − 3
8. Let f (x) = . Determine where f (x) is not continuous, and where
4 − x2
x2 − 2x − 3
f (x) = 0. Then solve the inequality ≥ 0. [Hint: One method to solve
4 − x2
this problem uses the Intermediate Value Theorem.]
3 1 − 4x
9. Solve the inequality ≤ . [Hint: Bring everything to one side of the
x+1 x−1
equation and simplify first.]

E.2.3 Third Fall 1999 Problem Assignment, with Solutions


Caveat lector! There could be misprints and other errors in these draft solutions.
1. (a) Apply the
r definition of the derivative as a limit to determine the derivative of
7x − 1
f (x) = for any x in the domain.
3
Solution:
q q
7(x+h)−1 7x−1
3
− 3
0
f (x) = lim
h→0
q h q
q q
7(x+h)−1 7x−1 7(x+h)−1 7x−1
3
− 3 3
+ 3
= lim ·q q
h→0 h 7(x+h)−1
3
+ 7x−13
7(x+h)−1 7x−1
3
− 3 1
= lim ·q q
h→0 h 7(x+h)−1 7x−1
3
+ 3
7h
3 1
= lim ·q q
h→0 h 7(x+h)−1 7x−1
3
+ 3

UPDATED TO October 25, 2004


Information for Students in MATH 140A 2003 09 3013

7 1
= lim ·q q
h→0 3 7(x+h)−1 7x−1
3
+ 3
7 1
= lim q q
3 h→0 7(x+h)−1 7x−1
3
+ 3
7 1
= q
6 7x−1
3

(b) Determine the value of f 0 (7) from the result of 1a.


Solution: √749−1 = 7
24
6 3

2. You will find below the definitions for a number of functions. If the domain of
definition is not stated, then you are to assume the domain to be as large as
possible.

• Determine the derivative of each function wherever it is defined in the domain.


• List any points in the domain where the function does not possess a derivative,
and explain precisely why there fails to be a derivative.
• If possible, determine, for those functions where requested, the equation of
the tangent or the normal to the graph of the function at the given point.
x
(a) f1 (x) = ; tangent at (1, f1 (1)).
5 − 8x
Solution: The domain of f1 is R − { 85 }, the set of all non-zero real numbers x
for which the denominator is non-zero; the function is continuous and differ-
entiable at every point of the domain. f10 (x) = 1(5−8x)−x(−8)
(5−8x)2
5
= (5−8x) 2 , at every
5
point x in the domain, i.e. at all real numbers except x = 8 . As f10 (1) = 59 ,
the equation of the tangent at (1, − 13 ) is y − (− 13 ) = 59 (x − 1), i.e. 5x − 9y = 8.
(b) f2 (x) = (5 + 6x2 + x4 ) − (1 + 2x + x2 )2 + 4x(1 + x); normal at (5, f2 (5)).
Solution: As f2 is a polynomial, its domain is the entire real line; it is dif-
ferentiable (hence continuous) at all points of R also. f20 (x) = (12x + 4x3 ) −
2(1 + 2x + x2 )(2 + 2x) + (4 + 8x) = −12x2 + 8x; hence f20 (5) = −260, and the
1
slope of the normal at the point (x, y) = (5, f2 (5)) is 260 . The equation of the
1
normal is y − (−396) = 260 (x − 5).
(c) f3 (x) = (1 − x)2 .
Solution: f3 is a polynomial, and is both defined and differentiable at all real
numbers. f30 (x) = −2(1 − x).

UPDATED TO October 25, 2004


Information for Students in MATH 140A 2003 09 3014


(d) f4 (x) = − 10x − x2 ; tangent at (1, −3).
Solution: The domain of f4 consists of all real numbers x such that 10x−x2 ≥
0, i.e. where the product x(x−10) is non-positive: that is, all x in the interval
0 ≤ x ≤ 10. The derivative will be defined at every point in the domain except
at the end-points of the interval, at one of which the tangent approaches the
1
vertical; f40 (x) = − 12 (10x − x2 )− 2 (10 − 2x) = √10x−x
x−5
2 . At the point x = 1,
0 1 1
f = − 2 · 3 ·8 = − 3 . The equation of the tangent there is y−(−3) = − 43 (x−1),
4

i.e. 4x + 3y + 5 = 0.
3
(e) f5 (x) = |3 − 2x|; tangent at x = .
2
Solution: The function is defined for all real x. As
½
3 − 2x 3 − 2x ≥ 0
f5 (x) = ,
2x − 3 3 − 2x < 0
½
−2 x < 32
f5 (x) = .
2 x > 32
The derivative does not exist at the point x = 23 , since the difference quotient
approaches +2 on one side of this point, and −2 on the other, so there is no
2-sided limit to the difference quotient.
 3 
 x 
when x 6= 0
(f) f6 (x) = |x| ; tangent and normal at (0, 0).
 
0 when x = 0
Solution: The function has all real numbers in its domain; the only point that
might have caused difficulty would have been 0, as the given quotient is not
defined there; however, the value at x = 0 is given separately. Differentiating
f within each of the defining half-lines gives
½
0 2x when x > 0
f6 (x) =
−2x when x < 0
To determine the value of the derivative at x = 0 we have to appeal to first
h3 h3
−0 −0 h2 −0
principles. f60 (0) = limh→0 |h|
h
. Since lim+ |h|
h
= lim+ h
= lim+ h = 0,
h→0 h→0 h→0
and the limit from the left is also equal to 0, we can assert that the 2-sided
limit exists, so f60 (0) = 0. The line through the origin with slope 0 is the
x-axis, which is tangent to the curve at that point.
½ 2
x − 6x when x < 1
(g) f7 (x) = ; tangent at (1, −5).
−1 − 4x when x ≥ 1
Solution: f7 is defined for all x. The function was created by piecing together
a branch of a parabola for x < 1, and a ray of a straight line for x ≥ 1.

UPDATED TO October 25, 2004


Information for Students in MATH 140A 2003 09 3015

Strictly within each of the constituent half-lines the function is obviously


differentiable: for x < 1, f70 (x) = 2x − 6; while, for x > 1, f70 (x) = −4; note
that we were unable to make any assertion about x = 1. Indeed, at x = 1
the limit from the right of the difference quotient is again seen to be −4. The
limit from the left is the value 2(1) − 6 = −4, which can be seen from first
principles. As here again the limits from left and right are equal, the limit
exists, and the function is differentiable at x = 1. The line through (1, −5)
with slope 4 has equation y + 5 = 4(x − 1), or 4x − y = 9.
 2
 x − 6x when x < 1
(h) f8 (x) = 0 when x = 1 ; normal at (1, −5).

−1 − 4x when x > 1
Solution: f8 is similar to f7 , except for the definition of the function at x = 1.
This time, when we attempt to determine the derivative at x = 1 by first
principles, we find that the quotient f (1+h)−f
h
(1)
= f (1+h)
h
has infinite limits —
i.e. does not have a (real) limit from the two sides. As the limit does not
exist, the function is not differentiable at x = 1, and there can be no normal.

3. (The functions referred to below were defined in Problem 2 above.)


(a) Determine the value of the function f7 ◦ f7 at any point x in its domain. [This
problem is technically difficult. Try to understand what makes the problem
difficult. Your solution to this problem will not be graded.]
Solution:


 (x2 − 6x)2 − (x2 − 6x) if x2 − 6x < 1 and x<1

(−1 − 4x)2 − (−1 − 4x) if x≥1 and −1 − 4x < 1
f7 ◦f7 (x) =

 −1 − 4(x2 − 6x) if x<1 and x2 − 6x ≥ 1

−1 − 4(−1 − 4x) if x≥1 and −1 − 4x ≥ 1
To study the four combinations of inequalities we observe that x2 − 6x =
(x − 3)2 − 9. The inequalities simplify to
 2 2 2


 (x − 6x) − (x − 6x) if 3 − 10 < x < 1

(−1 − 4x)2 − (−1 − 4x) if x ≥ 1√
f7 ◦ f7 (x) =

 −1 − 4(x2 − 6x) if x ≤ 3 − 10

−1 − 4(−1 − 4x) never.
i.e.
 2 √
 (x − 6x)(x2 − 6x − 1) if 3− 10 < x < 1
f7 ◦ f7 (x) = 2(8x2 + 6x + 1) if x ≥ 1√

−4x2 + 24x − 1 if x ≤ 3 − 10

UPDATED TO October 25, 2004


Information for Students in MATH 140A 2003 09 3016

(b) Determine the value of the functions f1 ◦ f4 and f4 ◦ f1 at all points in their
domains. Find the derivatives of these functions in two ways:
i. by differentiation of the formulæ you have determined
ii. by applying the chain rule
and show that the results are the same.
Solution:

− 10x − x2
(f1 ◦ f4 )(x) = √
5 − 8(− 10x − x2 )
1
= − 1
8 + 5(10x − x2 )− 2
s µ ¶ µ ¶2
x x
(f4 ◦ f1 )(x) = − 10 −
5 − 8x 5 − 8x
s
x(50 − 81x)
= −
(5 − 8x)2
Both methods of differentiation should lead to the same results:
5(x − 5)
(f1 ◦ f4 )0 (x) = √
5 + 8 10x − x2
205x − 125
(f4 ◦ f1 )0 (x) = p
(5 − 8x)2 x(50 − 81x)
Don’t panic! These computations are much to difficult for an examination!
The purpose was to test your perseverence, under conditions where two meth-
ods had to lead to the same answer, so you could verify your work.
1+s 1 √ dy
(c) Suppose that y = , s = t − , t = x. Determine the value of at
1−s t dx
x = 2.
Solution:
dy dy ds dt
= · ·
dx ds dt dx µ ¶
2 1 1
= 2
· 1− 2 · √
(1 − s) t 2 x

When x = 2, t = 2, s = √1 .
Hence
2
µ ¶ √
dy 2 1 1 3 2+4
=³ ´2 · 1 − · √ = .
dx 1 2 2 2 2
1 − √2

UPDATED TO October 25, 2004


Information for Students in MATH 140A 2003 09 3017

4. For each of the following functions, and for the given closed interval,
• find all critical points;
• find all local maxima and local minima;64
• find all global maxima and global minima, or explain why none of either exists.
All claims should be supported by careful reasoning; show all your work.
(a) g1 (x) = (x − 3)5 ; interval [2, 4]
Solution: g10 (x) = 5(x − 3)4 for 2 < x < 4. The derivative is thus defined
at all points in the interior of the interval; we cannot speak of a derivative
at the end points, since the behavior of the function must be known on both
sides of a point where a derivative is to be determined; (we could speak of
1-sided derivatives, but have not done so in this course). The critical points
will be those points where the derivative is zero, i.e. only the point x = 3. To
determine local and global extrema we must consider the value of the function
at the critical point and also at the end-points of the interval. Since g1 (3) = 0,
g1 (2) = −1, and g1 (4) = 1, we see that the global maximum, of value 1, occurs
at x = 4, and that the global minimum, of value −1, occurs at x = 2. The
point x = 3 is neither a local minimum nor a local maximum: for x < 3 the
function value is less than g1 (3), while, for x > 3, the function value is greater
than g1 (3).
· ¸
2 3 5
(b) g2 (x) = 6 − 36x + 15x − 2x ; interval ,5
2
Solution: The function g2 , being a polynomial,
£5 ¤ is defined at all points. As the
definition of g2 has been specified
¡ ¢ as 2 , 5 , the function will have a derivative
at all points in the interval 25 , 5 ; we have no information about g2 for x < 52 ,
nor for x > 5, so we do not have a derivative at either of those end-points.
Where it is defined, g20 (x) = −36 + 30x − 6x2 = −6(x − 2)(x − 3), which
vanishes at x = 2 and x = 3. But x = 2 is not in the domain of definition
of the function, so this point is irrelevant. We thus have just one critical
point, x = 3; there g2 (3) = 6 − 108 + 135 − 54 = −21. At the end-points,
g2 ( 52 ) = − 33
2
, g2 (5) = −49. As g2 ( 52 ) > g2 (3) > g2 (5), the global maximum is
5
at 2 , and the global minimum at 5.

 −1

when −1 ≤ x < 0
(c) Solution: g30 (x) = x2 . For h 6= 0, the value of
 1
 − when 0 < x ≤ 1
x2
64
THIS PROBLEM WAS NOT TO BE GRADED: It was announced in the lectures that this topic
would not be discussed until [21, Chapter 4]; accordingly this part of the question was to be omitted.

UPDATED TO October 25, 2004


Information for Students in MATH 140A 2003 09 3018

the difference quotient g3 (h)−0


h
is h12 , which has no finite limit as h → 0; hence
the function is not differentiable at x = 0. The derivative is not zero anywhere
in the domain; thus the only critical point is x = 0. Evaluating the function
at this critical point, and at the two end-points, we obtain g3 (−1) = −1,
g3 (0) = 0, g(1) = 1. We may not apply [23, Theorem 3, p. 144] here, because
this function is not continuous throughout the domain of definition. In fact,
it is discontinuous at x = 0. As x → 0− , g3 (x) → −∞; and, as x → 0+ ,
g3 (x) → ∞; these one-sided limits are not equal real numbers, so no limit
exists; for continuity they must be equal, and must equal the function value,
here 0. Because the function becomes positively and negatively infinite inside
the domain of definition, it can have neither a global maximum nor a global
minimum. There is no need to evaluate it at the critical point, nor at the
end-points of the interval of definition.
(d) Solution: This problem differs from the preceding problem in that the limit
as x → 0, while still not existing as a real number, is ∞. In this case there
can be no global maximum. The cited theorem still does not apply, as there is
still a discontinuity at x = 0. The function values at x = ±1 are both 1, and,
from examination of the behavior of the function near these points, might be
suspected of being global minima. However, the value at x = 0 is lower. The
global minimum is thus 0, assumed only at the point x = 0; everywhere else
the function value is strictly larger.
(e) Solution:
√ The domain of definition of g5 is the whole√real line. The derivative
of x for x > 0 is 2√1 x ; similarly, the derivative of −x for x < 0 is − 2√1−x .
Investigation of the difference quotient g5 (h)−0
h
as h → 0, shows that the limit
does not exist as a finite number; in fact is it ∞, and the function may be
said to have a vertical tangent. Thus x = 0 is a critical point, because the
function lacks a derivative there. The function is, however, continuous, so we
may determine the global extrema by studying its behavior at the end-points
and the critical point: g5 (−4) = 2, g5 (0) = 0, g5 (9) = 3. The global maximum
is thus at x = 9, and the global minimum is at x = 0.

5. [25, Examples XLVI.17] (Cambridge Math. Tripos 1930) The graph of the function
ax + b
h(x) = has (2, −1) as a critical point. Determine a and b, and show
(x − 1)(x − 4)
that the critical point is a local maximum. (Note: It is intended that this problem
be solved without using concepts from Chapter 4 of your textbook (involving higher
derivatives.))
2
Solution: For x different from 1 and 4, h0 (x) = a(x −5x+4)−(ax+b)(2x−5)
(x−1)2 (x−4)2
. As the
derivative exists at x = 2, it can be a critical point only because the derivative is

UPDATED TO October 25, 2004


Information for Students in MATH 140A 2003 09 3019

zero; this implies that a(22 −5·2+4)−(a·2+b)(2·2−5) = 0, i.e. b = 0. We are also


2a+0 x
told that h(2) = −1, hence (2−1)(2−4) = −1, so a = 1. Thus h(x) = (x−1)(x−4) . As
(x−2) 2
h(2) − h(x) = − (x−1)(x−4) , this difference is non-negative in an interval surrounding
x = 2 (since the numerator is non-negative, and the denominator is negative). Thus
h(x) ≤ h(2), so 2 is a local maximum.

6. Determine values of the constants a, b, c which will cause the curve y = ax3 + bx2 +
cx + d to pass through the points (2, 6) and (−1, 6) and to be tangent to the line
y = 3x + 1 at the point (1, 4).
Solution: We are given four kinds of information: three points through which the
curve passes, and the equation of the tangent at one of these points. Imposing the
condition that the curve pass through the points (2, 6), (−1, 6), and (1, 4) yields
three linear equations:

8a + 4b + 2c + d = 6
−a + b − c + d = 6
a+b+c+d = 4

Finally, as the tangent at x = 1 has slope 3, we know that 3ax2 + 2bx + c|x=1 = 3,
i.e. 3a+2b+c = 3. Solving the 4 linear equations simultaneously yields (a, b, c, d) =
(−1, 3, 0, 2).

7. Find all lines with slope −3 which are normal to the curve 64y = x3 .
1 3 3 2
Solution: At the point (x, 64 x ) on the curve the slope of the tangent is 64 x , so
64
the slope of the normal is − 3x2 . Imposing ¡ the condition
¢ that this equal −3, we
obtain that x = ± 83 . Through
¡ ¢ the point ± 8
3
, ± 8
27
the equation of the line with
8 8 8·28
slope 3 is y ∓ 27 = −3 x ∓ 3 , i.e. 3x + y = ± 27 .

8. Find the volume of the uncovered box of greatest volume that can be made by
cutting equal squares out of the corners of a piece of sheet metal which is 21 cm.
× 5 cm., and turning up the sides.
Solution: If the side of the square cut from each corner is of lengh x, where 0 ≤ x ≤
5
2
, then the volume obtained after the sides are folded up is x · (21 − 2x)(5 − 2x) =
4x3 − 52x2 + 105x. Setting the derivative, 12x2 − 104x + 105 (i.e. (2x − 15)(6x − 7))
equal to zero, we find that x = 15 2
or x = 67 . The second of these is the only critical
point; the first is not in the interval of definition of the function. To determine the
maximum we compare the value of the volume at x = 76 with the volume at the
end-points, both of which give volume 0. At x = 67 the volume is 76 · 56 3
· 83 = 1568
27
3
cm. , which exceeds the value of 0 at the end-points. This is the largest volume.

UPDATED TO October 25, 2004


Information for Students in MATH 140A 2003 09 3020

9. Show that, among all right-angled triangles whose hypotenuse is 10 units long, the
triangle whose area is maximum is isosceles.
Solution: Denote the lengths of the non-hypotenuse sides by a and b. As √ these are
2 2 2 2
constrained by the equation
√ x + b = 10 = 100, we know that b = 100 − a .
1
The area is therefore 2 a 100 − a2 . We may take the domain to be 0 ≤ a ≤ 10.
2
The derivative of the area function is, after reduction, √50−a 2 , which is zero when
√ 100−a
a = ±5 2. Of these two values, only the positive one is in the domain of definition
of the function. At the end-points of the domain the function is zero; while, at
the critical point we have found, the function value is positive; this, then, is the
maximum point. For this value of a, b = a, which was to be proved.

E.2.4 Fourth Fall 1999 Problem Assignment


dy
1. Find if
dx
¡ ¢
(a) y = ln 1 + x2
2
(b) y = e−x

x
(c) y = x

2. Find an equation for the tangent line at x=0 to the graph of y = ex − e−x .
1
3. Find the greatest value of f (x) = ln x .
x2
4. If f (x) = e−x sin x , find the values of x where

(a) f 0 (x) = 0 ;
(b) f 00 (x) = 0 .
dy sin−1 2x
5. (a) Find if y= .
dx sin−1 x
µ ¶
−1 x+1 dz
(b) Show that if z = tan + tan−1 x , then =0.
x−1 dx
6. A picture a metres high is placed on a wall with its base b metres above
an observer’s eye. If the observer stands x metres away from the wall, find

(a) the angle α subtended by the picture at the observer’s eye; and
(b) the distance x which will give the maximum value for α.

UPDATED TO October 25, 2004


Information for Students in MATH 140A 2003 09 3021

7. Use the tangent line approximation to find an approximate value for

(a) ln(0.94) ;
(b) cos−1 (0.47) .

8. Find an equation for the line tangent to the curve x4 + x2 y 2 + y 4 = 21 at the


point (1, 2) .
d2 y
9. If x3 + y 3 + 6xy = −5 , find at the point (−1, 2) .
dx2
ln x
10. Evaluate lim . [Hint: Write down the limit definition of f 0 (1) , where
x→1 x − 1
f (x) = ln x .]

E.2.5 Fifth Fall 1999 Problem Assignment


1. For each of the functions,

(x2 − 1)2 1 − sin x


f (x) = g(x) =
4x2 √ cos x
−x2
h(x) = xe k(x) = ln x

determine all of the information requested below, and sketch a graph of the func-
tion. Show all your work.

(a) For any discontinuities, determine whether they are removable.


(b) Where is the function increasing? Where is it decreasing?
(c) Where is the graph concave upwards? Where is it concave downwards?
(d) Where, if any, are the local extrema? In each case you should indicate whether
the extremum is a local maximum or a local minimum, and provide justifica-
tion for your choice.
(e) Where, if any, are the global extrema? Again, justification is expected.
(f) Where, if any, are the intercepts (with the coordinate axes)?
(g) Where, if any, are the inflection points?
(h) Is the graph symmetric about any vertical line x = a? (The graph of F is
symmetric about x = a if the equation remains unchanged under the trans-
formation x − a → a − x; i.e. if F (x) = F (2a − x). In particular, the graph
is symmetric about the y-axis if F (x) = F (−x). Such a function is said to be
even.)

UPDATED TO October 25, 2004


Information for Students in MATH 140A 2003 09 3022

(i) Is the graph symmetric under rotation about the origin? (The graph of F is
symmetric under rotation about the origin if the equation remains unchanged
under the transformation (x, y) → (−x, −y); i.e. if F (−x) = −F (x). Such a
function is said to be odd .)
(j) Determine all horizontal and all vertical asymptotes to the graph.
2. (a) Show that at least one of the hypotheses of the Mean Value Theorem fails to
hold for the following functions; show also that the conclusion of the theorem
fails to hold for each function, where the interval is [a, b] = [−1, 1].
½ 1
when x 6= 0
i. f1 (x) = x .
0 when x = 0
2
ii. f2 (x) = x 3 .
(b) [21, Problem 4.3.48] [23, Problem 4.3.48] Show that the function f2 (x) de-
fined in Problem 2(a)ii above does satisfy the conclusion of the Mean Value
Theorem on the interval [−1, 27].
(c) Using the Mean Value Theorem, show carefully that the equation x7 + x − 12
has exactly one real solution. Then show — without using a calculator —
that this solution lies between x = 1 and x = 2.
3. Consider the function f3 (x) = x cos x.
(a) Determine the intercepts of the graph of f3 on the coordinate axes.
(b) Show that the critical points of f3 occur at points of intersection of y = tan x
1
with the curve y = .
x
1
(c) Show that, wherever it is defined, the function tan x − is increasing. Use
x
this
µ fact to show that there
¶ is exactly one critical point of f3 in each interval
(2n − 1)π (2n + 1)π
, , where n is any integer. [Hint: Use a Corollary to
2 2
the Mean Value Theorem.]
(d) Sketch the graph of f3 .
1
4. Use the Second Derivative Test in finding all points on the curve y = 2 which
2x
are closest to the origin.
65
5. In each of the following cases, evaluate the limit, or show that it does not exist.
65
This problem and the next were originally included in Assignment 2, but were subsequently omitted,
because they are based upon [21, §§4.5,4.7], which material had not been discussed in the lectures before
the due date of Assignment 2.

UPDATED TO October 25, 2004


Information for Students in MATH 140A 2003 09 3023


3x + 2 x
(a) lim
x→∞ 1−x
|2x − 3|
(b) lim
x→−∞ x + 2

6. Find a value for a so that


µ ¶
x2 + 1
lim 3x + 1 − a
x→∞ x+1

exists as a finite limit, and evaluate that limit.

E.3 2000/2001 Problem Assignments, with Solutions


E.3.1 First 2000/2001 Problem Assignment, with Solutions
Instructions

• The basic principle you should follow in any mathematics course is that every statement
should be justified. While it may be that in some cases you are not able to provide a
flawless logical argument, that should always be your goal. Getting the right answer is
always desirable, but will usually not be enough.

• Notwithstanding the preceding comment, you should not spend disproportionately long
amounts of time on any one question. Solutions will be posted on the Web.

• While the textbook in [10, 1.1] describes four different ways in which to represent a func-
tion, we usually regard the representation by an explicit formula as the most desirable.
In particular, graphical representation should be used only to assist you in visualizing a
solution, not as the final solution; it will normally not be acceptable to explain a step in
a proof by reference to a graph — but it is a good policy to make a quick sketch of the
graph of any function you have to work with.

• You should always attempt, in an algebraic representation, to make your functions as


“simple” as possible; there may be different “simple” ways of representing the function.
Use your good judgment: the simplification is intended to help you!

• Do not approximate numbers unless you are asked to do so. Thus, π should not be
replaced by 3.1415926...; of course, you should know the values of the standard trigono-
metric functions at familiar multiples and submultiples of π, and use this information√to
simplify your answers, where applicable. For example, you should replace sin π3 by 23 ,
but not by 0.8660....
It is usually preferable to leave fractions in the form m
n , rather than rewriting as decimal
fractions. This preference applies even when the decimal expansion is finite, as in, for

UPDATED TO October 25, 2004


Information for Students in MATH 140A 2003 09 3024

example, 34 = 0.75. One reason for this preference is that decimal fractions are often
interpreted as approximations, rather than as the exact value; so by writing 0.75 instead
of 34 you could be obscuring the fact that your datum is exact.

• You may assume that, over an interval [c, d], the maximum and minimum values of a
linear function — that is a function of the form f (x) = ax+b, where a and b are constants
— are attained at the end points of the interval.

1. Let a function f be defined by f (x) = |2x| + |2x + 3|.

(a) Find algebraic formulæ which give the value of f (x) without using the absolute
value function. For this purpose it will be necessary to break the domain up
into several parts, as the formula will be different in different subintervals.
(b) Showing all your work, determine the domain and the range66 of f .
(c) Determine whether or not f has an inverse function. If it does, determine a
formula for f −1 (x).

Solution:

(a)
½
−2x when x<0
Since |2x| =
2x when x≥0
½
−2x − 3 when 2x + 3 < 0
and |2x + 3| =
2x + 3 when 2x + 3 ≥ 0
(
−2x − 3 when x < − 23
or, equivalently |2x + 3| = ,
2x + 3 when x ≥ − 23

we may add the inequalities to obtain



 −2x − 2x − 3 when x < − 32
|2x| + |2x + 3| = −2x + 2x + 3 when − 32 ≤ x < 0

2x + 2x + 3 when x ≥ 0

 −4x − 3 when x < − 23
= 3 when − 32 ≤ x < 0

4x + 3 when x ≥ 0
66
Many mathematicians prefer to use the term image instead of range.

UPDATED TO October 25, 2004


Information for Students in MATH 140A 2003 09 3025

(b) While the description of the function changes as one moves from one interval
to the next, the function is defined for all x — i.e. the domain is the whole
real line.
For the interval −∞ < x ≤ − 32 , the function value ranges between +∞ and
3; it remains at 3 through the middle interval; and, for − 32 < x < ∞, again
takes on all values which are ≥ 3. Thus the range is [3, ∞).
[How could one prove, rigorously, that the stated intervals do indeed constitute the
range of the function? For example, when x < − 23 , we reverse the inequality when
we multiply by −4, to obtain −4x > 6; adding −3 to both sides of the inequality
yields −4x − 3 > 3, showing that the portion of the range of the function for this
part of the domain is contained in the interval (3, ∞). And, if y is any real number
such that 3 < y, then 6 < y + 3; so, multiplying both sides of the inequality by − 14 ,
y+3 3
and
³ thereby
´ reversing
³ ´the inequality, we obtain − 4 < − 2 . But this tells us that
f − y+34 = −4 y+3 4 − 3 = y. We have thus shown that every number y in the
interval (3, ∞) is in the range: thus the range of the function corresponding to this
portion of the domain is precisely the interval (3, ∞).]
(c) This function is not invertible, since there are values which are attained at
more than one point in the domain. For example, the function takes on the
same value for all x such that − 32 ≤ x ≤ 0.
2. Let a function g be defined by

g(x) = 2x − 5 . (71)
Showing your work, answer each of the following questions for g.
(a) Determine the domain.
(b) Determine the range.
(c) Determine whether the function has an inverse function. If it does, find a
formula for g −1 (x), and determine the domain and range.
Solution:
(a) g is the composition of two functions. In the first phase x is mapped on
to 2x − 5. The second function applied is the square root. The mapping
x 7→ 2x − 5 is defined for all x; that is, its domain is the whole real line.
However, the mapping to the square root is not defined for negative numbers.
Thus we cannot proceed unless 2x − 5 ≥ 0, i.e. unless
5
x≥ . (72)
2
Inequality (72) gives the domain of g.

UPDATED TO October 25, 2004


Information for Students in MATH 140A 2003 09 3026

(b) Any non-negativeµ 2 real¶ number r is attained as a value of the function g;


r +5
specifically, g = r. Thus the range of g is the interval [0, ∞).
2
(c) We may square both sides67 of (71) to obtain (g(x))2 = 2x − 5, so x =
(g(x))2 + 5
, showing that x is determined as soon as g(x) is known. Thus g
2
x2 + 5 68
is invertible, and its inverse is given by x 7→ .
2
3. A curve is symmetric about the y-axis if the presence of any point (x0 , y0 ) on the
curve implies the presence of (−x0 , y0 ); a curve is symmetric about the x-axis if
the presence of any point (x0 , y0 ) on the curve implies the presence of (x0 , −y0 );
a curve is symmetric about the origin if the presence of any point (x0 , y0 ) on the
curve implies the presence of (−x0 , −y0 ). A function F is odd if its graph is
symmetric about the origin, i.e. for any x in the domain, F (−x) = −F (x) (which
is equivalent to saying that if (x, F (x)) is on the graph, then so is (−x, −F (x))).
F is even if its graph is symmetric about the y-axis; i.e., for any x in the domain,
F (−x) = F (x) (which is equivalent to saying that if (x, F (x)) is on the graph, then
so is (−x, F (x))).
For each of the following functions F , determine whether

• F is odd.
• F is even.
• the graph of F is symmetric about the x-axis.
• the graph of F is symmetric about the y-axis.

(a) F (x) = | sin x|


(b) F (x) = cos x
³ π´
(c) F (x) = cos x −
2
(d) F (x) = 0
1
(e) F (x) = x3 +
x
67
√ √
More precisely, we can multiply both sides of the equation g(x) − 2x − 5 = 0 by g(x) + 2x − 5.
68
Note that, while this formula is meaningful even when x < 0, we are specifically restricting the
domain of this inverse function to be the range of the function g. With that proviso we find that the
pair of functions g and g −1 have the property that the domain ¡ of each
¢ is the range of the other, and
vice-versa. Thus the domain of g −1 is [0, ∞), and the range is 52 , ∞

UPDATED TO October 25, 2004


Information for Students in MATH 140A 2003 09 3027

1
(f) F (x) = x3 −
x
2
(g) F (x) = G(x ), where G is any function.

Solution:
Before beginning our solution, we make the following observations:

• The graph of F is symmetric about the y-axis ⇔ F is even.


• If the graph of F is symmetric about the x-axis, then, for any x0 in the
domain, both points (x0 , F (x0 )) and (x0 , −F (x0 )) will lie on the graph. But
the graph of a function can never meet two distinct points on the same vertical
line x = x0 . It follows that these points cannot be distinct, i.e. that F (x0 ) =
−F (x0 ), i.e. that F (x0 ) = 0 for all points x0 in the domain. Thus only the
0 function, or restrictions of this function to subsets of the real line can have
this symmetry property.
• The only function which is both even and odd is the 0 function (or restrictions
of this function to subsets of the real line). For both conditions F (−x) = F (x)
and F (−x) = −F (x) must hold for all x, implying that F (x) = 0 for all x.

(a) | sin(−x)| = |−sin x| = | sin x|, so this function is even; (hence, it not being the
0 function, it is not odd). By the preceding comments, its graph is symmetric
about the y-axis, but not about the x-axis.
(b) cos(−x)
¡ π ¢= cos x for all x, so this function is even. It is not odd; for example,
cos − 4 = 2 6= − √12 = − cos π4 .
√1

¡ ¢
(c) By a well-known property of the cosine function, cos x − π2 = cos ¡ x cos ¢π2 +
sin x sin π2 = (cos x) · 0 + (sin
¡ x) π· ¢1 = sin x for all x. Hence cos −x − 2 =
π

sin(−x) = − sin x = − cos x − 2 , so this function is odd. As the function


is not identically 0, it is not even; its graph is not symmetric about either
coordinate axis.
(d) The 0 function is both even and odd, and its graph is symmetric about both
coordinate axes.
µ ¶
3 1 3 1
(e) (−x) + =− x + , so this function is odd. It is not even, and its
−x x
graph is symmetric about neither coordinate axis.
1
(f) The function x3 − has the same symmetry properties as the preceding
x
function.

UPDATED TO October 25, 2004


Information for Students in MATH 140A 2003 09 3028

(g) G((−x)2 ) = G(x2 ), so this function is even, and its graph is symmetric about
the y-axis. The function is not odd, and the graph is not symmetric about
the x-axis.

4. Let
x2 − 4
f (x) =
x−1
1
g(x) =
f (x − 1)
³ π´
h(x) = sin x −
4
1
k(x) = ³ π´
h x−
4
(a) Determine an explicit formula for (f ◦ g)(x).
(b) Describe the domains of the functions f , g, f ◦ g, h and k.
(c) [BONUS QUESTION] Describe the domain of g ◦ f .

Wherever possible in this problem, algebraic expressions should be simplified.


Solution:

(b) f . Since f is defined as a ratio of polynomials, its domain is the set if all real
numbers where the denominator is not zero; as the denominator is x − 1,
the domain of f is all real numbers except 1.
g.
1 1 1 x−2
g(x) = = 2 = 2 = 2
f (x − 1) (x − 1) − 4 x − 2x − 3 x − 2x − 3
(x − 1) − 1 x−2
The preceding statement is valid wherever g is defined. That will be the
intersection of the set of x such that f (x − 1) is defined, and the set
where the fraction 1f (x − 1) is defined; i.e., it will be the set of x where
f (x − 1) is defined, from which we must remove all points x such that
f (x − 1) = 0. Since f (x) is defined for x 6= 1, f (x − 1) is defined for all
x such that x − 1 6= 1, i.e. such that x 6= 2. And f (x − 1) = 0 precisely
when (x − 1)2 − 4 = 0, equivalently when x − 1 = ±2, i.e. when x = −1
or x = 3. Therefore the domain of g is the set R − {−1, 2, 3}.
f ◦ g. The first function to be applied is g. Its domain consists of R −
{−1, 2, 3}. But the application of f requires, further, that we exclude

UPDATED TO October 25, 2004


Information for Students in MATH 140A 2003 09 3029

points x where g(x) = 1, since 1 is not in the domain√ of f . Thus we


3 ± 13
exclude points x such that x2 − 3x − 1 = 0, i.e. . The domain is,
2
therefore, ( √ √ )
3 + 13 3 − 13
R − −1, 2, 3, ,
2 2
(We could also have determined this by excluding from R the roots of the
denominator of the ratio of polynomials that we have determined above
for f ◦ g.)
h. h(x) is defined for all real numbers x.
¡ ¢
g. k(x) is not defined at points where sin x − π2 is zero, i.e. where x − π2 is
an integer multiple of π. Thus the points that have to be excluded from
2n + 1
R are all numbers of the form π, where n is any integer.
2
(a)
µ ¶2
x−2
−4
x2 − 2x − 3
(f ◦ g)(x) =
x−2
2
−1
x − 2x − 3
(x − 2)2 − 4(x2 − 2x − 3) 1
= 2 2
· 2
(x − 2) − (x − 2x − 3) x − 2x − 3
4 3 2
4x − 16x − 9x + 52x + 32
=
(x2 − 3x − 1)(x2 − 2x − 3)

(c) We can also express (g ◦ f )(x) as a ratio of polynomials in x:

f (x) − 2
(g ◦ f )(x) =
f (x)2 − 2f (x) − 3
x2 − 4
−2
= µ 2 x−1
¶2
x −4 x2 − 4
−2 −3
x−1 x−1
((x2 − 4) − 2(x − 1)) (x − 1)
=
(x2 − 4)2 − 2(x2 − 4)(x − 1) − 3(x − 1)2
(x2 − 2x − 2)(x − 1)
= 4
x − 2x3 − 9x2 + 14x + 5

UPDATED TO October 25, 2004


Information for Students in MATH 140A 2003 09 3030

This ratio is not defined at points where the denominator is zero. However,
that denominator is a quartic function of x, and its roots are not obvious.69
Instead, let us attack this problem the same way we dealt with f ◦ g above.
First we have to exclude the points where f is not defined, i.e., the point x = 1.
Next we have to exclude points x such that f (x) is not in the domain of g:
x2 − 4 x2 − 4
these are the solutions to each of the equations = 3, = 2, and
x−1 x−1
x2 − 4
= −1, i.e., the roots of the three quadratic polynomials, x2 − 3x − 1,
x−1
x2 − 2x − 2, and x2 + x − 5. It follows that the domain of g ◦ f is
( √ √ √ √ )
3 + 13 3 − 13 −1 + 21 −1 − 21 √ √
R − 1, , , , , 1 + 3, 1 − 3
2 2 2 2

5. (a) Showing all your work, and without using tables, computers, slide rules, or a
calculator , determine the value of log16 2.
(b) Showing all your work, determine all values of x for which
µ ¶x
x 1
2 = . (73)
2

(c) Showing all your work, determine all values of x for which log2 (ln x) = 1.
(d) Showing all your work, simplify cos(x + y) cos(x − y) − cos2 x − cos2 y, where
x and y are any real numbers.
Solution:
(a) By [10, p. 70]
ln 2 1 1 1 1
log16 2 = = = = =
ln 16 ln 16 log2 16 log2 24 4
ln 2
(b) By the Laws of Exponents [10, p. 58]
µ ¶x µ ¶x
x 1 x 1
1=1 = 2· =2 · ,
2 2
hence µ ¶x
1 1 20
= x = x = 2−x .
2 2 2
69
Although, in fact, this quartic polynomial does factorize into the product of two quadratic polyno-
mials, x2 − 3x − 1 and x2 + x − 5, whose roots can be found in the usual way.

UPDATED TO October 25, 2004


Information for Students in MATH 140A 2003 09 3031

Thus (73)⇔ 2x = 2−x . Applying the inverse function log2 to both sides of this
equation yields the equivalent statement x = −x, which is, in turn, equivalent
to x = 0.
(c) Applying the exponential function (to base 2) to both sides of the hypothesis
log2 (ln x) = 1 yields ln x = 21 = 2. Now apply the exponential function, this
time to base e, to obtain the equivalent statement x = e2 .
6. (a) [10, Exercise 14, p. A34] A circle has radius 10 cm. Showing all your work,
determine the length of the arc subtended by a central angle of 72◦ .
(b) [10, Exercise 34, p. A35] Showing all your work, find the remaining 5 trigono-
4 3π
metric ratios for the angle θ, if it is known that csc θ = − , and < θ < 2π.
3 2
(c) (Adapted from [10, Exercise 70, p. A35]) Find all values of x that satisfy the
equation cos x + sin 2x = 0.
(d) Determine the value of cos(x + y) cos(x − y) − cos2 x − cos2 y, as x and y range
over the real numbers.
Solution:
72
(a) The entire circle has circumference 2π · 10 cm. The angle of 72◦ is 360 = 15
of the central angle subtending the entire circle. Hence the arc has length
1
5
× 20π = 4π cm.
1 3
(b) First observe that sin θ = =− .
csc θ 4
Now, since θ is in the 4th quadrant, the cosine is positive. This resolves the
2 2
sign choice when
r we solve for
√ cos θ in the equation sin θ + cos θ = 1, so we
9 7
have cos θ = + 1 − = . The remaining ratios can now be computed
16 4
without ambiguity:
−3
sin θ 4 3
tan θ = = √ = −√
cos θ 7 7
4
√ √
7 −4 7
cot θ = cos θ · csc θ = · =−
4 3 3
1 4
sec θ = =√
cos θ 7
(c) (There are many ways of approaching problems of this type, so the following
is only one possibility. Of course, all methods lead to the same answers.)
cos x + sin 2x = 0 ⇔ cos x + 2 sin x cos x = 0 (double-angle formula)

UPDATED TO October 25, 2004


Information for Students in MATH 140A 2003 09 3032

⇔ cos x(1 + 2 sin x) = 0


1
⇔ cos x = 0 or sin x = −
2
The solution set will, therefore, be the union of the solution sets for the two
equations shown.
π
cos x = 0. The solutions to this equation are all x of the form x = + nπ,
2
where n is any integer.
sin x = − 12 . One solution is x = − π6 ; another solution is x = − 5π 6
; remember
that the graph of the sine function is symmetric about the line x = 3π 2
(as it is about all odd integer multiples of π2 ): any value which is realized
between 3π 2
and 2π will also be attained at a mirror-image point between

π and 2 . These two solutions each give rise to an infinite set of solutions,
2π units¡ apart. The
¢ full
¡ 5set of solutions
¢ is all real numbers of either of the
1
forms
¡ 11 − + 2m
¢6 ¡ 7 π, −
¢ 6 + 2m π or, equivalently of either of the forms
6
+ 2m π, 6 + 2m π where m is any integer.
The set of solutions to the original equation is, therefore, the union of the two
lists above:
½µ ¶ µ ¶ µ ¶ ¯ ¾
1 7 11 ¯
+ m π, + 2m π, ¯
+ 2m π ¯ m any integer
2 6 6

(d) (There are several possible approaches to this problem. The following may
not be the simplest.)
cos(x + y) cos(x − y) − cos2 x − cos2 y
= (cos x cos y − sin x sin y)(cos x cos y + sin x sin y) − cos2 x − cos2 y
= cos2 x cos2 y − sin2 x sin2 y − cos2 x − cos2 y
= cos2 x cos2 y − (1 − cos2 x)(1 − cos2 y) − cos2 x − cos2 y
= cos2 x cos2 y − (1 − cos2 x − cos2 y + cos2 x cos2 y) − cos2 x − cos2 y = −1
for all x and for all y.
7. (a) [10, Exercise 8, p. A-23] Show that the following equation represents a circle,
and determine its centre and radius:
16x2 + 16y 2 + 8x + 32y + 1 = 0

(b) [10,
¡ 1 Exercise
¢ 36, p. A-15] Find an equation of the line through the point
2
2
, − 3
which is perpendicular to the line 4x − 8y = 1.
Solution:

UPDATED TO October 25, 2004


Information for Students in MATH 140A 2003 09 3033

(a) We first group the terms in x and y separately, as quadratic functions. Then
we scale to arrange for the coefficients of x2 and of y 2 to be 1, and complete
the square in each of these:

16x2 + 16y 2 + 8x + 32y + 1 = 0


⇔ (16x2 + 8x) + (16y 2 + 32y) = −1
µ ¶
1 1
⇔ x + x + (y 2 + 2y) = −
2
2 16
à µ ¶2 ! à µ ¶2 !
1 1 1 1
⇔ x2 + x + · + y 2 + 2y + ·2
2 2 2 2
µ ¶2 µ ¶2
1 1 1 1
=− + · + ·2
16 2 2 2
µ ¶2
1 1 1
⇔ x+ + (y + 1)2 = − + +1=1
4 16 16
¡ ¢
which is the equation of the circle with centre − 14 , −1 and radius 1.
(b) The slope of the line 4x − 8y = 1 is 21 ; so the line we seek, being perpendicular
to that line, must have slope − 11 = −2. The line through the given point,
2
with slope −2, has equation
µ ¶ µ ¶
2 1
y− − = −2 x −
3 2

which simplifies to 2x + y = 31 .

8. (a) [10, Exercise 84(a), p. A36] Showing all your work, determine the exact value
of arctan(−1).
(b) [10, Exercise 86(b), p. A36] Showing all your work, determine the exact value
of arcsin 1.
(c) [10,¡Exercise 92, p. A36]
¢ Showing all your work, determine the exact value of
sin arcsin 31 + sin−1 32 .
(d) (Adapted from [10, Exercise 96, p. A36]) Simplify sin (−2 cos−1 x).
Solution:

(a) The inverse tangent


¡ π πfunction
¢ inverts the restriction of the tangent function to
the interval − 2 , + 2 . In that interval the only x such that tan x = −1 is
− π4 . Hence arctan −1 = − π4 .

UPDATED TO October 25, 2004


Information for Students in MATH 140A 2003 09 3034

(b) The inverse


£ π sineπ ¤function inverts the restriction of the sine function to the
π
interval − 2 , + 2 . In that interval the only x such that sin x = 1 is 2 . Hence
arcsin 1 = π2 .
(c) Let’s apply the formula for the sine of a sum:
µ ¶
1 −1 2
sin arcsin + sin
3 3
µ ¶ µ ¶ µ ¶ µ ¶
1 −1 2 1 −1 2
= sin arcsin · cos sin + cos arcsin · sin sin
3 3 3 3
µ ¶ µ ¶
1 2 1 2
= · cos sin−1 + cos arcsin ·
3 3 3 3

Now sin−1 23 is in the first quadrant, so its cosine is positive, and equal to
q √
+ 1 − 49 = 35 ; arcsin 13 is also in the first quadrant, so its cosine is also

2 2
positive, and is equal to 3
. Substituting these values gives
µ ¶ µ ¶ √ √
1 −1 2 1 2 5+4 2
· cos sin + cos arcsin · = .
3 3 3 3 9

(d)
¡ ¢ ¡ ¢
sin −2 cos−1 x = − sin 2 cos−1 x
¡ ¢ ¡ ¢
= −2 sin cos−1 x · cos cos−1 x

We know that, for all x, cos cos−1£x =¤x. Also, since the inverse cosine function
takes its values in the interval 0, π2 , and the sine function is non-negative

throughout that interval, we also know that sin cos−1 x = + 1 − x2 . It follows
that ¡ ¢ √
sin −2 cos−1 x = −2x 1 − x2 .

E.3.2 Second 2000/2001 Problem Assignment, with Solutions


In all of these problems you were expected to justify every statement you made, and to
show all your work.

1. For the curve y = 2x3 , find the slope MP Q of the secant line through the points
P = (1, 2) and Q = (2, 16), i.e. the points with x = 1 and x = 2.
2·23 −2·13
Solution: The slope of the line joining P and Q is 2−1
= 14.

UPDATED TO October 25, 2004


Information for Students in MATH 140A 2003 09 3035

2. Given 
 x3 + 2 if x ≤ −1
2
f (x) = x + x + 1 if −1 < x < 1

x4 + 2 if x≥1
find the following limits, if they exist; or explain why the limit does not exist.
Justify your answers.

(a) lim f (x)


x→−1+

(b) lim f (x)


x→−1−

(c) lim f (x)


x→−1

(d) lim+ f (x)


x→1

(e) lim− f (x)


x→1

(f) lim f (x)


x→1

Solution:

(a) For x to the right of −1 (but to the left of +1) f (x) = x2 + x + 1; hence
lim + f (x) = (−1)2 + (−1) + 1 = 1.
x→−1

(b) For x to the left of −1, f (x) = x3 + 2; hence lim − f (x) = −13 + 2 = 1
x→−1

(c) Since the one-sided limits from both left and right exist at −1 and have the
same value, lim f (x) exists, and its value is the common value of the one-
x→−1
sided limits, i.e. 1.
(d) For x to the right of 1, f (x) = x4 + 2; hence lim+ f (x) = 14 + 2 = 3.
x→1

(e) For x to the left of 1 (but to the right of −1) f (x) = x2 + x + 1; hence
lim− f (x) = 12 + 1 + 1 = 3.
x→1

(f) Since the one-sided limits from both left and right exist at 1 and have the
same value, lim f (x) exists, and its value is the common value of the one-
x→1
sided limits, i.e. 3.

3. Given that lim f (x) = 5, lim g(x) = 0, and lim h(x) = −8, find the following
x→3 x→3 x→3
limits, if they exist. If a limit does not exist, explain why.

(a) lim (f (x) + h(x))


x→3

UPDATED TO October 25, 2004


Information for Students in MATH 140A 2003 09 3036

(b) lim (x2 f (x))


x→3

(c) lim (f (x))2


x→3
f (x)
(d) lim
x→3 2h(x)
g(x)
(e) lim
x→3 f (x)
f (x)
(f) lim
x→3 g(x)
2h(x)
(g) lim
x→3 f (x)−h(x)
p
(h) lim 3
h(x)
x→3

Solution:

(a) lim (f (x) + h(x)) = lim f (x) + lim h(x) = 5 + (−8) = −3. (We have used the
x→3 x→3 x→3
Sum Law.)
(b) Since we know that the limit of x as x → 3 is 3, we can use the Product Law:
³ ´2
2
lim (x f (x)) = lim x · lim f (x) = 32 · 5 = 45.
x→3 x→3 x→3

(c) Again by the Product Law, the limit of a square is the square of the limit:
³ ´2
lim (f (x))2 = lim f (x) = 52 = 25.
x→3 x→3

(d) First we use the Constant Multiple Law to determine the limit of the de-
nominator. Then we use the Quotient Law, since the limit of the denom-
inator exists, and is not zero, and the limit of the numerator also exists:
f (x) 5 5
lim = =− .
x→3 2h(x) 2(−8) 16
g(x) 0
(e) Again by the Quotient Law, lim = = 0.
x→3 f (x) 5

(f) But, in this case, the limit of the denominator is 0. Since the limit of the
numerator exists and is non-zero, the limit of the quotient does not exist.
(g) The Difference Law is used for the denominator; then the Constant Multiple
2h(x) 2(−8) 16
Law and the Quotient Law: lim f (x)−h(x) = 5−(−8) = − 13 .
x→3
p √
(h) By the Root Law, lim 3 h(x) = 3 −8 = −2.
x→3

UPDATED TO October 25, 2004


Information for Students in MATH 140A 2003 09 3037

4. Use the Intermediate Value Theorem to show that there is a solution to the equation
x3 + 2x2 − 42 = 0 in the interval (0, 3).
Solution: Let f (x) = x3 + 2x2 − 42. This function is continuous everywhere on
R. Since f (0) = −42, and f (3) = 33 + 2(9) − 42 = 3, and since −42 < 0 < +3,
the function must assume the intermediate value 0 at a point in the open interval
0 < x < 3.

5. Given 
 2x3 + 16 if x ≤ −2
2
f (x) = x + bx + c if −2 < x < 2

3x4 − 48 if x≥2
determine values for b and c so that f is continuous everywhere. Justify your
answer.
Solution: In each of the intervals (−∞, −2], (−2, 2), [2, ∞) the function is a poly-
nomial, and is therefore continuous at all points. The only possibly problematic
points are x = −2 and x = +2. We will determine limits from the left and right at
each of these two points; then we equate the two one-sided values, and obtain con-
ditions on b and c which we attempt to satisfy. lim − 2x3 + 16 = 2(−2)3 + 16 = 0,
x→−2
lim + x2 + bx + c = (−2)2 + b(−2) + c = 4 − 2b + c; lim− x2 + bx + c = 22 + b(2) + c =
x→−2 x→2
4 4
4 + 2b + c, lim+ 3x − 48 = 3(2) − 48 = 0. We solve the equations 0 = 4 − 2b + c
x→2
and 4 + 2b + c = 0, to obtain b = 0, c = −4. With this pair of values — and only
these values — f is continuous everywhere.

6. The displacement in meters of a particle moving in a straight line is given by


s = t3 + sin tπ, where t is measured in seconds. Find the average velocity in meters
per second over the time period [1, 5].
53 + 0 − 13 − 0
Solution: Average velocity = = 31 meters per second.
5−1
x−5
7. Find the value of lim+ .
x→5 |x − 5|
x−5
Solution: For all x > 5, |x−5|
= 1. Hence the limit, as x approaches 5 from the
right is 1.
3x
8. Find the value of x at which the curve y = x+7
has a vertical asymptote.
Solution: The domain of the function is R − {−7}: at all points except for −7
the function is a ratio of polynomials, each having a limit, where the limit of the
denominator is non-zero. Hence, by the Quotient Law, the limit of the function

UPDATED TO October 25, 2004


Information for Students in MATH 140A 2003 09 3038

exists (as a real number). However, as x → −7− , the denominator approaches 0


from the left, while the numerator approaches −21; so the ratio becomes infinitely
3x
large positively; this is what we mean when we write lim − x+7 = +∞. Similarly,
x→−7
as x → −7+ , the denominator approaches 0 from the right, while the numerator
approaches −21; so the ratio becomes infinitely large negatively; this is what we
3x
mean when we write lim + x+7 = −∞. It is in cases like these, where the one-sided
x→−7
limits are infinite positively or negatively, that we say that the curve has a vertical
asymptote.
x3 +8
9. Find the value of the limit lim .
x→2 x+2

x3 + 8 lim (x3 + 8) 23 + 8
x→2
Solution: lim = = = 16. (Problem: Find the limit
x→2 x + 2 lim (x + 2) 2+2
x→2
3
of the given quotient as x → −2. Solution: lim x +8 = lim x2 − 2x + 4 =
x→−2 x+2 x→−2
22 + 2 · 2 + 4 = 12.)
¡ 1 ¢
10. Find the value of lim x+2 + x24−4 .
x→−2
¡ 1 ¢
Solution: lim x+2 + x24−4 = lim (x+2)(x−2)
x+2
= lim 1
= 1
= − 14 .
x→−2 x→−2 x→−2 x−2 −2−2

11. Find the value of lim √x−4 .


x→4 x−2
√ √
( x−2)( x+2) √
Solution: For positive x, different from 4, √x−4 = √ = x + 2. Hence
√ x−2 x−2
the limit, as x → 4 is 4 + 2 = 4.
1
12. How close to 4 do we have to take x so that 4x + 10 is within of 26?
100
Solution:
1
|4x + 10 − 26| < 0.01 ⇔ |4(x − 4)| <
100
1
⇔ |x − 4| <
400

(x+1)2
13. At what value of x does the function x2 −1
have a removable discontinuity?
(x+1)2
Solution: x2 −1 = x+1
x−1
for all x except x = −1, where the ratio is not defined
because both numerator and denominator of the fraction become 0 there. Every-
where except at x = 1 this is a ratio of continuous functions, and the denominator

UPDATED TO October 25, 2004


Information for Students in MATH 140A 2003 09 3039

is non-zero. Only at x = 1 or at x = −1 can this function have any type of


discontinuity.
As x → 1− , the function approaches −∞; and, as x → 1+ , the function approaches
+∞. Thus x = 1 is an infinite discontinuity: it cannot be “removed” by the device
of defining the function at the point, since no matter what value we would choose
to give the function there, it could still not be continuous.
But a different situation holds at x = −1. Since the function is equal to x+1 x−1
near (but not at) x = −1, its one-sided limit from either side of −1 is 0, so the
limit exists at the point. However, the function is not defined at the point, since
our definition was expressed as a ratio that becomes meaningless there. We can
“remove” the discontinuity by extending the function to the domain R − {1} —
we simply define the value of the newly extended function to be 0 there. Now the
new function is continuous at every point except −1.
(x+1)2
14. At what value of x does the function x2 −1
have an infinite discontinuity?
Solution: See the solution to Problem 13.
|x−2|
15. At what value of x does the function x−2
have a jump discontinuity?
Solution: This function is equal to +1 when x > 2, and to −1 when x < 2; it is
not defined at x = 2. The limits from the left and right are different at x = 2,
so the function has no limit there, and is said to be discontinuous. But, as both
one-sided limits exist, the discontinuity is a jump discontinuity.
1
16. Find the distance between the two values of x at which the function x2 −3x+2
is
discontinuous.
1 1
Solution: x2 −3x+2 = (x−1)(x−2) . Expressed as a ratio of two polynomials — which
are continuous everywhere — this function is also continuous everywhere, except
possibly at points where the denominator is zero. That happens precisely at x = 1
and x = 2. At either of these points the function is not even defined — hence it is
discontinuous at both of these points. The distance between the two discontinuities
is, therefore, |2 − 1| = 1.
(x2 −1)(x2 +3x+2)
17. Find the values of x where the function f (x) = (x2 −1)2 (x+2)(x−3)
has a removable
discontinuity.
Solution: This function is a ratio of polynomials, and polynomials are continuous
everywhere. This function will also be continuous everywhere, except where it is not
defined. That can happen only when the denominator is zero. So the discontinuities
of the function are at x = −1, 1, −2, 3. We arrived at this information without even
considering the factorization of the numerator.

UPDATED TO October 25, 2004


Information for Students in MATH 140A 2003 09 3040

Let us now factorize both numerator and denominator:


(x2 − 1)(x2 + 3x + 2) (x − 1)(x + 1)2
= .
(x2 − 1)2 (x + 2)(x − 3) (x − 1)2 (x + 1)2 (x − 3)

We can cancel equal non-zero factors in numerator and denominator. Thus


1
f (x) =
(x − 1)(x + 3)

provided x is distinct from 1, −1, and −2. As x → 1, and also as x → −3, this
ratio becomes infinite; so the limits as we approach these two points are infinite;
in fact the function approaches +∞ from one side of each of these points, and −∞
from the other side. So 1 and −3 are infinite discontinuities. But, as x → −2, the
ratio does not become infinite: the limit is 13 from either side. Nor, as x → −1,
does the ratio become infinite: the limit is 14 from either side; but the function is
not defined at either x = 2 or x = −1. We can “remove” these discontinuities by
defining a new function which takes on the values 13 at x = −2, 14 at x = −1, and,
elsewhere, behaves like f (x).
½
−3x if x ≤ 1
18. Find the real number(s) c for which the function f (x) =
(x − c)(x + c) if x > 1
is continuous on (−∞, +∞).
Solution: For x < 1 the function is defined by the upper line of the array; as
x → 1− , f (x) approaches the value (−3) · 1 = −3. For x > 1 it is the second
line of the array which defines the function. Here, as x → 1+ , (x − c)(x + c) →
(1 − c)(1 + c). To make the function continuous at x = 1 it is necessary and
sufficient that −3 = (1 − c)(1 + c), i.e. that c = 2 or c = −2.
r
8x + 3x2
19. Evaluate lim .
x→∞ 13x2 − 9
Solution:
r s
2 1
8x + 3x x2 8x + 3x2
lim = lim 1 ·
x→∞ 13x2 − 9 x→∞
x2
13x2 − 9
s
1
x2
(8x + 3x2 )
= lim 1
x→∞
x2
(13x2 − 9)
s
8
x
+3
= lim
x→∞ 13 − x92

UPDATED TO October 25, 2004


Information for Students in MATH 140A 2003 09 3041

q
8
x
+3
= lim q
x→∞
13 − x92
q
lim x8 + 3
x→∞
= q (Quotient Law)
lim 13 − x92
x→∞
q ¡ ¢
lim x8 + 3
x→∞
= q ¡ ¢ (Root Law)
lim 13 − x92
x→∞

0+3
= √ (Sum Law)
13 − 0
√ r
3 3
= √ =
13 13

20. Determine whether there exists a real number x which is exactly 10 more than its
5th power.
Solution: (cf. [12, Solution to Exercise 2.6.59]). The problem may be paraphrased
as asking whether there exists a solution x to the equation x5 + 10 = x; or, equiv-
alently, whether the function f (x) = x5 − x + 10 is ever zero. Now f is continuous
everywhere; f (0) = 10 > 0, and f (−2) = −32 + 2 + 10 < 0. By the Intermediate
Value Theorem there will exist a point in the interval (−2, 0) at which f (x) = 0.

E.3.3 Third 2000/2001 Problem Assignment, with Solutions


1. Let f and g be functions whose domain is R, and which possess derivatives at every
point in R. Suppose also that the following data are given about f , g and their
derivatives.
x f (x) g(x) f 0 (x) g 0 (x)
1 1 −1 4 −2
2 −2 3 0 2
3 5 0 5 6
Showing all your work, determine each of the following, or explain why either
• it does not exist; or
• you do not have enough information to find the value.
µ ¶¯
d f (x) ¯¯
(a)
dx g(x) ¯x=1

UPDATED TO October 25, 2004


Information for Students in MATH 140A 2003 09 3042

¯
d ¯
(b) (f (t) + g(t))¯¯
dt t=2
µ ¶¯
d 1 ¯
(c) ¯
dx g(x) ¯x=3
µ ¶
d 0
(d) (f (x) + g (x)) (2)
dx
µ x ¶¯
d e ¯
(e) ¯ (Do not attempt to approximate e.)
dx f (x) ¯x=1
µ ¶¯
d f (x) ¯¯
(f)
dx f (x) ¯ x=2

Solution:

(a) By the Quotient Rule,


µ ¶¯
d f (x) ¯¯ f 0 (1) · g(1) − f (1) · g 0 (1) 4 · (−1) − 1 · (−2)
¯ = = = −2 .
dx g(x) x=1 g(1)2 (−1)2

(b) By the Sum Rule,

d
(f (x) + g(x)) (2) = f 0 (2) + g 0 (2) = 0 + 2 = 2 .
dx
1
(c) Since g(3) = 0, the function is not defined at x = 3. The derivative of
g(x)
g at x = 3 is defined in terms of g(3); so, if g is not defined there, it cannot
have a derivative there either.
(d) We do not know whether the function g 0 has a derivative at x = 2 — equiv-
alently, whether the function g has a second derivative at x = 2. Without
this information, and the actual value of that derivative if it exists, we cannot
evaluate the derivative of the sum.
(e) By the Quotient Rule,
µ ¶¯ d x¯
¯
d ex ¯¯ dx
e x=1· f (1) − e1 · f 0 (1)
=
dx f (x) ¯x=1 f (1)2
e1 (f (1) − f 0 (1))
= = e(1 − 4) = −3e
f (1)2

UPDATED TO October 25, 2004


Information for Students in MATH 140A 2003 09 3043

(f) This case can also be attacked using the Quotient Rule, and we find that
µ ¶
d f (x) f 0 (2) · f (2) − f (2) · f 0 (2) 4·1−1·4
(2) = 2
= = 0.
dx f (x) f (2) 12

f (x)
Alternatively, one can observe that the function is defined wherever
f (x)
f (x) 6= 0; in particular, it is defined at x = 2; wherever it is defined, its value
is 1. Thus this is a constant function! The derivative of any constant function
is zero.

2. Suppose that a particle moves in a straight line with its position at time t given
by the formula f (t) = et (sin t + cos t).

(a) Find the velocity, the speed, and the acceleration of the particle at time t.
(b) Determine the average velocity during the time interval from t = 0 to t = 2π.
(c) Determine the smallest positive value of t — if any — when the particle
returns to the origin.
(d) Determine the smallest positive value of t — if any — when the particle is
stationary — i.e. the velocity is 0.
(e) Determine whether there is a maximum distance that the particle attains
away from the origin. If there is a maximum distance, determine what it is.

[The velocity is given by the derivative of f (t) (which is called the displacement;
the speed is the magnitude of the velocity. The acceleration is the derivative of the
velocity with respect to time.]
Hints: You may wish to use the identities:
³ π´ 1
sin x + = √ (sin x + cos x)
4 2
³ π´ 1
cos x + = √ (− sin x + cos x)
4 2
which are consequences of the addition formulæ and the known values for the sine
and cosine of π4 . Do not attempt to simplify complicated formulæ involving π.
Solution:

(a) The velocity at time t is f 0 (t) = et (cos t − sin t) + et (sin t + cos t) = 2et cos t.
The speed is the magnitude of the velocity, i.e., 2et | cos t|. (We may remove
the exponential factor outside the absolute signs since exponentials are always

UPDATED TO October 25, 2004


Information for Students in MATH 140A 2003 09 3044

d 0
positive.) The acceleration is the derivative of the velocity, i.e. f (t) =
¡ t ¢ dt
2 e (− sin t) + et cos t = 2et (− sin t + cos t).
√ ¡ ¢
(b) Using one of the given identities, we see that f (t) = 2 · et · sin x + π4 . The
average velocity from t = 0 to t = 2π is

f (2π) − f (0)
2π − 0
1 ³√ 2π ³ π´ √ ³ π ´´
= 2e · sin 2π + − 2 · e0 · sin
2π 4 4
1 √ ¡ ¢ π 2π
e −1
= 2 e2π − 1 sin =
2π 4 2π
√ ¡ ¢
(c) We seek the smallest positive value t when f (t) = 0, i.e. when
¡ et 2¢sin t + π4 =
0. Since et is never zero, this equation is equivalent to sin t + π4 = 0, whose
solutions are t + π4 = nπ, where n is any integer. The smallest positive value
will then be 34 π.
(d) 2et cos t will be zero precisely when the cosine is zero, i.e. when t is an odd
integer multiple of π2 .
t
(e) We have
√ seen above that f (t) ¡ is πa¢product of e (which is never 0), the con-
stant 2, and the factor sin t + 4 , which oscillates in value between +1 and
−1. Because of the exponential factor et the amplitude of the oscillations is
increasing as t becomes large. The limit lim f (t) does not exist. We can see
t→∞
this by exhibiting values of t where the function is arbitrarily large positively,
and others where it is arbitrarily large negatively. For example, if we take
π
t = 2nπ + , we find the values oscillating between et and −et , and we know
4
that the exponential function approaches infinity.

3. Determine the derivative of each of the following functions. You may not use the
Chain Rule, but you may use any of the Rules and Theorems in [10, §§3.1–3.4],
including the General Power Rule.
µ ¶
1
(a) 21 ex + x
e
1
(b) 2
(ex − e−x )
x4 + 2x2 − x − 5
(c) √
x
(d) xe − ex

UPDATED TO October 25, 2004


Information for Students in MATH 140A 2003 09 3045

Solution:
(a)
µ µ ¶¶ µµ ¶¶
d 1 x 1 1 d x 1
e + x = e + x Constant Multiple Rule
dx 2 e 2 dx e
µ ¶
1 dex 0 · ex − 1 · ex
= + Quotient Rule
2 dx e2x
ex − e−x
=
2
ex + e−x
(b) Analogously to the preceding, .
2
(c)
x4 + 2x2 − x − 5 7 3 1 1
√ = x 2 + 2x 2 − x 2 − 5x− 2
x
µ 4 ¶
2
d x + 2x − x − 5 d ³ 7 3 1 1
´
⇒ √ = x 2 + 2x 2 − x 2 − 5x− 2
dx x dx
µ ¶
7 5 3 1 1 −1 1 3
= x 2 + · 2x 2 − x 2 − − 5x− 2
2 2 2 2
7 5 1 1 1 5 3
= x 2 + 3x 2 − x− 2 + x− 2
2 2 2
d e
(d) (x − ex ) = e · xe−1 − ex , by the Generalized Power Rule and properties of
dx
the exponential.
4. Showing all your work, determine the derivative of each of the following functions
from first principles: that is, you are to evaluate a limit in each case, and are not
to use any of the “Rules” for evaluating derivatives.
(a) f (x) = 4x − 6

(b) g(x) = 2x + 3
1
(c) h(x) = 3 . [Hint: Remember the factorization a3 − b3 = (a − b)(a2 + ab + b2 ).]
x
Solution:
(a) For all x,
(4(x + h) − 6) − (4x − 6) 4h
f 0 (x) = lim = lim =4
h→0 h h→0 h

UPDATED TO October 25, 2004


Information for Students in MATH 140A 2003 09 3046

(b) For all x,


p √
2(x + h) + 3 − 2x + 3
g 0 (x) = lim
h→0
p h p
√ √
2(x + h) + 3 − 2x + 3 2(x + h) + 3 + 2x + 3
= lim ·p √
h→0 h 2(x + h) + 3 − 2x + 3
(2(x + h) + 3) − (2x + 3)
= lim p √
h→0 h( 2(x + h) + 3 + 2x + 3)
2
= lim p √
h→0 2(x + h) + 3 + 2x + 3
1
= √
2x + 3

(c) For all x,


1 1
3
− 3
(x + h) x
h0 (x) = lim
h→0 h
x3 − (x + h)3 −3x2 h − 3xh2 − h3
= lim = lim
h→0 hx3 (x + h)3 h→0 hx3 (x + h)3
−3x2 − 3xh − h2
= lim
h→0 x3 (x + h)3
lim (−3x2 − 3xh − h2 ) −3x2
h→0
= = = −3x−4
lim (x3 (x + h)3 ) x6
h→0

5. Determine precisely where the function f , defined below, is differentiable.




 −x − 2 if x ≤ −1


 −x2 if −1 < x < 0
f (x) = 1 if x=0



 −x2 if 0 < x ≤ +2

4 − 4x if x > +2

Solution: The function is a polynomial in each of the intervals (−∞, −1], (−1, 0),
(0, 2), and (2, ∞). As polynomials are differentiable [10, §3.1], the only places
where differentiability can fail is at the points x = −1, 0, 2. We check each of them
separately.

UPDATED TO October 25, 2004


Information for Students in MATH 140A 2003 09 3047

f (x) − f (−1)
x = −1: We have to determine whether the limit lim exists. By
x→−1 x − (−1)
[10, Theorem 2.3.1, p. 107], the two one-sided limits must exist and be equal.
But
f (x) − f (−1) (−x − 2) − (1 − 2)
lim − = lim −
x→−1 x − (−1) x→−1 x − (−1)
−x − 1
= lim − = −1 while
x→−1 x+1
f (x) − f (−1) −x2 − (1 − 2)
lim + = lim +
x→−1 x − (−1) x→−1 x − (−1)
−x2 + 1
= lim + = lim + (−x + 1) = 2 6= −1
x→−1 x+1 x→−1

As the one-sided limits are not equal, the function is not differentiable at this
point.
2
x = 0: Here we have to consider the existence of lim f (x)−fx
(0)
, that is lim −xx−1 .
x→0 x→0
But this limit cannot exist, since the ratio becomes infinitely large close to
0, as the numerator approaches −1, but is divided by an arbitrarily small
denominator. Thus the limit does not exist, and the function fails to be
differentiable at x = 0.
We could have argued this alternatively by appealing to [10, Theorem 2.9.4,
p. 169], since the function is discontinuous at x = 0, so it cannot possibly be
differentiable there.
f (x)−f (2)
x = 2: This time we have to consider the existence of lim x−2
. We observe
x→2
2
from the definition that f (2) is defined to be −2 = −4. Then

f (x) − f (2) −x2 − (−4)


lim− = lim−
x→2 x−2 x→2 x−2
= lim− (−x − 2) = −4 ; and
x→2
f (x) − f (2) (4 − 4x) − (−4)
lim+ = lim+
x→2 x−2 x→2 x−2
= lim+ −4 = −4
x→2

As the limits from the left and right are equal, the function is, indeed, differ-
entiable at the point x = 2.

6. The curve y = 3x5 − 20x3 − 675x + 12 has some points with horizontal tangents.
Showing all your work, find all such points.

UPDATED TO October 25, 2004


Information for Students in MATH 140A 2003 09 3048

d ¡ 5 ¢
Solution: Setting 3x − 20x3 − 675x + 12 equal to zero, we find that 15x4 −
dx
60x2 − 675 = 0, or, equivalently, (x2 )2 − 4x2 − 45 = 0. This is a quadratic equation
(in x2 ), and its solutions are x2 = −5 and x2 = 9. There are no real solutions to
x2 = −5, so we may confine ourselves to the second equation, whose solutions are
x = ±3. Thus there are exactly 2 points with horizontal tangents: (3, −1824) and
(−3, 1848).
7. Determine the value of each of the following limits, or explain why they do not
exist. Do not use l’Hôpital’s Rule.
tan3 πx
(a) lim
x→0 x3
(b) lim(cot 5θ)2 · (sin 3θ) · tan(−2θ)
θ→0

Solution:
(a)
õ ¶3 ³ !
tan3 πx sin πx π ´3
lim = lim ·
x→0 x3 x→0 πx cos πx
µ ¶3
sin πx ³ π ´3
= lim lim (Product Rule)
x→0 πx x→0 cos πx
µ ¶3 µ ¶3
sin πx limx→0 π
= lim (Quotient Rule)
x→0 πx limx→0 cos πx
µ ¶3 µ ¶3
sin y limx→0 π
= lim (where y = πx)
y→0 y limx→0 cos πx
³ π ´3
3
= 1 · = π3
1
(b)
¡ ¢
lim (cot 5θ)2 · (sin 3θ) · tan(−2θ)
θ→0
à µ ¶2 µ ¶ µ ¶ !
1 5θ sin 3θ tan(−2θ)
= lim cos2 5θ · · · · 3θ · · 2θ
θ→0 25θ2 sin 5θ 3θ −2θ
õ ¶2 µ ¶ µ ¶!
6 5θ sin 3θ sin(−2θ) 1
= − lim cos2 5θ · lim · · · lim
25 θ→0 θ→0 sin 5θ 3θ −2θ θ→0 cos(−2θ)

6 6
= − · 1 · 12 · 1 · 1 · 1 = −
25 25

UPDATED TO October 25, 2004


Information for Students in MATH 140A 2003 09 3049

8. Determine all points (x, cos2 x) on the graph of the function cos2 x (i.e. cos x · cos x)
at which
(a) the tangent to the curve is parallel to the line y = x + 1.
(b) the tangent to the curve is horizontal
(c) the tangent to the curve is vertical
(d) the tangent to the curve is parallel to the line y = 2x − 3
(e) the normal to the curve passes through the origin, and |x| > 1.
[Hints: The first and last parts of the problem are more difficult than the others.
Remember the identities involving sin 2x and cos 2x. The normal to the curve at
a point is the line through the point which is perpendicular to the tangent.]
Solution:
(a) The derivative of (cos x)2 is cos x·(− sin x)+(− sin x)·cos x = −2 sin x·cos x.70
We impose the condition that this be equal to the slope of the line y = x + 1,
i.e., that
−2 sin x · cos x = 1 (74)
The points we seek will be the solutions to equation (74). A simple attack is
to observe that this equation is equivalent to
sin 2x = −1 . (75)
¡3 ¢
The general solution ¢ last equation is 2x = 2 + 2n π, where n is any
¡ to the
integer; hence x = 43 + n π, where n is any integer. Thus there are infinitely
many points where the tangent¡ has¢ slope11: the x-coordinates are as indicated;
the y-coordinates are cos 4 + n π = 2 . But note that the line y = 21 cuts
2 3

the curve also in infinitely many points where the tangent has slope −1.
(b) We have to solve the equation −2 sin x·cos x = 0, or, equivalently, −2 sin 2x =
0. The solutions to this equation are 2x = nπ, i.e. x = nπ 2
, where n is any
integer.
(c) The function is differentiable everywhere; that means that the derivative has
a (finite) value at every point, and so the tangent cannot be vertical. There
are no such points on the curve.
(d) The value of the derivative is −2 sin x · cos x, or − sin 2x. As a sine cannot
have magnitude exceeding 1, the slopes of tangents to this curve can never
equal 2. There are no points of this type!
70
We are avoiding using the Chain Rule, as the assignment was due before the Rule had been fully
discussed in the lectures.

UPDATED TO October 25, 2004


Information for Students in MATH 140A 2003 09 3050

1
(e) The slope of the normal at the point (t, cos2 t) is − . The equation of
− sin 2t
the normal is
1
y − cos2 t = (x − t)
sin 2t
The normal will pass through the origin if the coordinates of the origin satisfy
the equation, that is if t = cos2 t sin 2t. But this implies that t is the product
of two factors, neither of which can exceed 1 in magnitude, so there are no
points with this property outside the interval −1 ≤ t ≤ 1.

E.3.4 Fourth 2000/2001 Problem Assignment, with Solutions


Solutions to most of the problems can be found in the Student Solutions Manual [12].
1. (a) [10, Exercise 3.5.3, p. 221] Write the composite function cos(tan x) in the form
f (g(x)). [Identify the “inner” function u = g(x), and the “outer” function
dy
y = f (u).] Then find the derivative .
dx
(b) Find an equation for the tangent to the curve y = cos(tan x) at the point
π
x= .
4
(c) Find an equation for the normal to the curve y = cos(tan x) at the point

x= .
4
Solution:

(a) In [12, Exercise 3.5.3, p. 82] the derivative is shown to have value − sin tan x ·
sec2 x. ³π ´ √ 2
(b) The slope of the tangent at , cos 1 is − sin 1 · 2 . Hence an equation of
4
the tangent is ³ π´
y − cos 1 = −2 sin 1 · x − .
4
µ ¶
5π √
(c) The slope of the tangent at , cos(1) is − sin(1) · (− 2)2 = −2 sin 1;
4
1
hence the slope of the normal is . An equation of the tangent is
2 sin 1
µ ¶
1 5π
y − cos 1 = · x− .
2 sin 1 4

or x − 2 sin 1 · y = − sin 2 + .
4

UPDATED TO October 25, 2004


Information for Students in MATH 140A 2003 09 3051

2. Find the derivative of each of the following functions:

(a) [10, Exercise 3.5.19, p. 222] (2x − 5)4 (8x2 − 5)−3


2
(b) [10, Exercise 3.5.21, p. 222] xe−x
1
(c) [10, Exercise 3.5.29, p. 222] 5− x

Solution:

(a) [12, Exercise 3.5.19, p. 82]


(b) [12, Exercise 3.5.21, p. 82] Note that the original function is odd ; and the
derivative is even. This is an instance of [10, Exercise 3.5.73, p. 224].
(c) [12, Exercise 3.5.29, p. 82]. (This problem could be solved from first princi-
ples. Simply observe that 5 = eln 5 , apply the usual rules for exponents, and
differentiate an exponential [now to base e] using the Chain Rule.)

3. [10, Exercise 3.5.56, p. 222] A table of values for f , g, f 0 , g 0 is given:

x f (x) g(x) f 0 (x) g 0 (x)


1 3 2 4 6
2 1 8 5 7
3 7 2 7 9

(a) If F (x) = (f ◦ f )(x), determine F 0 (2).


(b) If G(x) = (g ◦ g)(x), determine G0 (3).

Solution:

(a) F 0 (2) = f 0 (f (2)) · f 0 (2) = f 0 (1) · f 0 (2) = 4 · 5 = 20.


(b) G0 (3) = g 0 (g(3)) · g 0 (3) = g 0 (2) · g 0 (3) = 7 · 9 = 63.
dy
4. In each of the following cases, find by implicit differentiation.
dx
y
(a) [10, Exercise 3.6.11, p. 230] = x2 + 1
x−y

(b) [10, Exercise 3.6.13, p. 230] xy = 1 + x2 y
(c) [10, Exercise 3.6.17, p. 230] cos x − y = xex

Solution:

(a) Two different approaches are given in [12, Exercise 3.6.11, p. 86].

UPDATED TO October 25, 2004


Information for Students in MATH 140A 2003 09 3052

(b) [12, Exercise 3.6.13, p. 86]


(c) [12, Exercise 3.6.18, p. 86]
5. (a) [10, Exercise 3.6.23, p. 230] Regarding y as the “independent” variable, and x
as the “dependent” variable (i.e. regarding x as a function of y) apply implicit
differentiation to the equation
y 4 + x2 y 2 + yx4 = y + 1
dx
to determine .
dy
dy dy dx
(b) Use the same equation to determine , and verify that · = 1 (a fact
dx dx dy
which follows from the Chain Rule by interpreting y as a function of x which,
in turn, is a function of y, i.e. taking the point of view y = y(x(y))).
Solution:
(a) [12, Exercise 3.6.23, p. 86]
(b) Differentiating both sides of the given equation with respect to x yields
¡ ¢ ¡ ¢
4y 3 · y 0 + 2xy 2 + x2 · 2yy 0 + y 0 x4 + y · 4x3 = y 0 + 0
¡ ¢
⇒ 4y 3 + 2x2 y + x4 − 1 y 0 = −2xy 2 − 4yx3
2xy 2 + 4yx3
⇒ y0 =
1 − 4y 3 − 2x2 y − x4
³ √ ´
6. (a) Determine the derivative of cot−1 x − 1 + x2
(b) [10, Exercise 3.6.41, p. 231] Determine the derivative of arcsin(x2 ).
Solution:
(a)
d ³ −1 ³ √ ´´
cot x − 1 + x2
dx
1 d ³ √
2
´
= − ¡ √ ¢2 · x − 1 + x
1 + x − 1 + x2 dx
µ ¶
1 x
= − ¡ √ ¢ · 1− √
1 2 + 2x2 − 2x 1 + x2 1 + x2

1 1 + x2 − x 1
= − √ ¡√ ¢· √ =−
2 1+x 2 2
1+x −x 1+x 2 2(1 + x2 )

UPDATED TO October 25, 2004


Information for Students in MATH 140A 2003 09 3053

(b) [12, Exercise 3.6.41, p. 88]


7. (a) [10, Exercise 3.6.35] Find all points on the curve 2(x2 + y 2 )2 = 25(x2 − y 2 )
where the tangent is horizontal. (You may assume that the origin is not one
of these points.)
dx
(b) For the same curve, determine all points where = 0, i.e. where the tangent
dy
is vertical. (You may assume that the origin is not one of these points.)
Solution:
(a) [12, Exercise 3.6.35, p. 88] (It is shown that the four points with horizontal
tangents lie on a certain circle centred at the origin. To determine the points
one must solve the equation of this circle with the equation of the given curve.)
(b) One finds by implicit differentiation that
dx 25y + 4y(x2 + y 2 )
=
dy 25x − 4x(x2 + y 2 )
This derivative vanishes when y = 0 or 4(x2 + y 2 ) = −25. The latter situation
cannot occur, as a sum of squares cannot be negative; hence y = 0. That
is, the points with vertical tangents are at points where the curve meets the
5
x-axis: these are the points where x = 0 or x = ± √ . But being on the x-axis
2
is a necessary condition for having a vertical tangent, and is not sufficient. At
the origin there are two tangents,³ and neither
´ of them is vertical. The only
5
points with vertical tangents are ± √2 , 0 .

8. (cf. [10, Exercise 3.7.53, p. 239]) Find constants ci (i = 0, 1, 2) such that the
function f ( t) = c2 t2 + c1 t1 + c0 has the properties that f (−1) = 0, f 0 (−1) = 7,
f 00 (−1) = 10.
Solution: Imposing the given conditions yields the three equations
c2 − c1 + c0 = 0
−2c2 + c1 = 7
2c2 = 10
from which we conclude that c0 = 22, c1 = 27, c2 = 5.
9. [10, Exercise 3.8.27, p. 246] Find the domain of the function f , and determine its
derivative, where f (x) = x2 ln(1 − x2 ).
Solution: [12, Exercise 3.8.27, p. 94]

UPDATED TO October 25, 2004


Information for Students in MATH 140A 2003 09 3054

10. Determine the first and second derivatives of the function f (x) = | ln(sec x+tan x)|.
Solution:
d 1 d
ln(sec x + tan x) = · (sec x + tan x)
dx sec x + tan x dx
1
= · (sec x · tan x + sec2 x) = sec x
sec x + tan x
d2 d
2
ln(sec x + tan x) = sec x = sec x · tan x
dx dx
The problem referred to the derivatives of f (x) = | ln(sec x + tan x)|; the logarithm
will be negative precisely when sec x+tan x < 1; this last inequality may be shown,
using trigonometric identities, to be equivalent to
³x π ´ ³x´
cos + · sin < 0.
2 4 2
For the range 0 < x < 4π, we find, examining the signs of the two factors as
0 < x2 < π4 , ... 7π
4
< x2 < 2π, that the product is negative except when ¡0 < x2 < π
¢ 4
x 5π 1
or π < 2 < 4 . Thus, if we define s(x) to be +1 when 2nπ < x < 2n + 2 π,
where n is any integer, and −1 everywhere else, f 0 (x) = s(x) · sec x, and f 00 (x) =
s(x) · sec x tan x; neither function is defined at odd integer multiples of π2 .
11. Using “logarithmic differentiation”, or otherwise, determine the derivative of
sin2 x · tan4 x
(a) [10, Exercise 3.8.37, p. 246] y = .
(x2 + 1)2
(b) y = xln x
Solution:
(a) [12, Exercise 3.8.37, p. 95]
1 1
(b) Since ln y = (ln x)2 , · y 0 = 2 ln x · . Hence y 0 = 2x(ln x)−1 ln x.
y x
12. (a) [10, Exercise 3.7.19, p. 238] Find the first, second, and third derivatives of
g(t) = t3 e5t .
(b) [10, Exercise 3.7.13, p. 238] Find the second, third, and fourth derivatives of
x
.
1−x
(c) [10, Exercise 3.7.29, p. 238] Find y 00 where y is defined implicitly by the
equation x3 + y 3 = 1. Simplify your answer as much as possible.
Solution:

UPDATED TO October 25, 2004


Information for Students in MATH 140A 2003 09 3055

(a) [12, Exercise 3.7.19, p. 90]


d ¡ ¢
g 000 (t) = (25t3 + 30t2 + 6t)e5t
¡dt 2 ¢ ¡ ¢
= 75t + 60t1 + 6t0 e5t + 5 25t3 + 30t2 + 6t e5t
¡ ¢
= 125t3 + 225t2 + 90t1 + 6t0 e5t

(b) [12, Exercise 3.7.13, p. 90] This problem can be attacked naı̈vely, with re-
peated applications of the Quotient Rule. Another approach is to observe
x 1
that = −1 + . Hence
1−x 1−x
d x d ¡ ¢
= 0+ (1 − x)−1
dx 1 − x dx
d
= (−1) · (1 − x)−2 · (1 − x) = (1 − x)−2
dx
d2
x d ¡ ¢
2
= (1 − x)−2 = (−2) · (1 − x)−3 · (−1) = 2 · (1 − x)−3
dx 1 − x dx
d3 x d ¡ −3
¢
= 2 (1 − x) = 2(−3) · (1 − x)−4 · (−1) = 3 · 2 · (1 − x)−4
dx3 1 − x dx
d4 x d ¡ ¢
4
= 3·2 (1 − x)−4 = 3 · 2 · (−4) · (1 − x)−5 · (−1)
dx 1 − x dx
= 4 · 3 · 2 · (1 − x)−5

(c) [12, Exercise 3.7.29, p. 91]


³ x ´n
13. Evaluate the following limits, if possible; you may assume that lim 1 + = ex .
n→∞ n
In each case it is suggested that you make a substitution of the form m = kn, where
k is a constant that you choose, and then replace lim by lim when k > 0, or by
n→∞ m→∞
lim when k < 0.
m→−∞
µ ¶n
2
(a) lim 1 + .
n→∞ n
³ x ´n
(b) lim 1 − .
n→∞ 3n
Solution:
n
(a) Define m = , so n = 2m. Then
2
µ ¶n µ ¶2m
2 1
lim 1 + = lim 1 +
n→∞ n m→∞ m

UPDATED TO October 25, 2004


Information for Students in MATH 140A 2003 09 3056

õµ ¶m ¶2 !
1
= lim 1+
m→∞ m
µ µ ¶m ¶2
1
= lim 1 + Product Law
m→∞ m
= e2
m
(b) Define m = −3n, so n = − . Then
3
µ ¶n r³
1 x ´m
lim 1− = lim 3 1 −
n→∞ 3n m→−∞ m
r ³ x ´m
= 3
lim 1 − Root Law
m→−∞ m
r ³
3 x ´−t
= lim 1 + (t = −m)
t→∞ t
s
1
= 3 lim
¡ ¢
t→∞ 1 + x t
t
1 1 x
= r ³ ´ = √3 x
= e− 3
3 x t e
lim 1 +
t→∞ t

E.3.5 Fifth 2000/2001 Problem Assignment, with Solutions


1. [10, Exercise 3.9.12, p. 251] Prove the following identity about hyperbolic functions,
using only the definitions of these functions in terms of exponentials:

cosh(x + y) = cosh x · cosh y + sinh x · sinh y

Solution:

cosh x · cosh y + sinh x · sinh y


ex + e−x ey + e−y ex − e−x ey − e−y
= · + ·
2 2 2 2
(ex+y + ex−y + e−x+y + e−x−y ) + (ex+y − ex−y − e−x+y + e−x−y )
=
4
2ex+y + 2e−x−y
= = cosh(x + y)
4

UPDATED TO October 25, 2004


Information for Students in MATH 140A 2003 09 3057

4
2. [10, Exercise 3.9.21, p. 251] If tanh x = , find the values of the other 5 hyperbolic
5
functions at x.
Solution: [12, Exercise 3.9.21, p. 96]. (The problem could also be solved by using
the given information to show that e2x = 9, hence ex = +3, and thence determining
the values of the other functions.)
3. Use the definitions of the hyperbolic functions to find the following limits:
(a) [10, Exercise 3.9.23(b), p. 252] lim tanh x
x→−∞

(b) [10, Exercise 3.9.23(c), p. 252] lim sinh x


x→∞
(c) [10, Exercise 3.9.23(e), p. 252] lim sechx
x→∞
sinh x
(d) [10, Exercise 3.9.52, p. 252] lim
x→∞ ex
Solution:
(a) [12, Exercise 3.9.23(b), p. 96]
(b) [12, Exercise 3.9.23(c), p. 96]
(c) [12, Exercise 3.9.23(e), p. 96]
sinh x 1¡ −2x
¢ 1
(d) Since x
= 1 − e , and lim e−x = 0, the limit as x → ∞ is .
e 2 x→∞ 2
4. Determine the derivative of each of the following functions:
(a) [10, Exercise 3.9.32, p. 252] g(x) = sinh2 x
(b) [10, Exercise 3.9.38, p. 252] f (t) = ln(sinh t)
x √
(c) [10, Exercise 3.9.45, p. 252] x sinh−1 − 9 + x2
3
Solution:
(a) g 0 (x) = 2 sinh x·cosh x, which could be expressed more compactly as sinh(2x).
d 1
(b) f (t) = · cosh t = coth t
dt sinh t
(c) [12, Exercise 3.9.45, p. 98]
5. (a) [10, Exercise 3.10.7, p. 257] A street light is mounted at the top of a 15-
foot-tall pole. A man 6 feet tall walks away from the pole with a speed of 5
feet/second along a straight path. How fast is the tip of his shadow moving
when he is 40 feet from the base of the pole?

UPDATED TO October 25, 2004


Information for Students in MATH 140A 2003 09 3058

(b) The man extends his arms horizontally so that the distance between his fin-
gertips is 6 feet. What is the rate of increase of the shadow of his extended
arms on the ground when his feet are 40 feet from the base of the pole? You
may assume that his shoulders are 5 feet above the ground.
Solution:
(a) [12, Exercise 3.10.7, p. 99]
(b) We can consider two sets of similar triangles similar to the triangles considered
in part (a). Let A denote the top of the lamppost, and B its foot; let C be
a point on the man, at shoulder-height, and let D be a point on the man’s
feet; let G be the point where the line through A and C meets the ground.
15 |AB| |AG| |BG|
Then triangles ABG and CDG are similar, so = = = ,
5 |CD| |CG| |DG|
so |BG| = 3|DG|, |AG| = 3|CG|, and |BD| = 2|DG|.
Now let E be the end of the man’s horizontally-extended left arm, and let F
be the end of the shadow cast by that arm on the ground. Here triangles AGF
|AG| |GF | |GF | 3
and ACE are similar, so = , so, since |AG| = 3|CG|, = ;
|AC| |CE| |CE| 2
9
since |CE| = 3, |GF | = . The shadow of the extended arms is 2 × 29 = 9
2
feet long. As this shadow is of constant length, its rate of change is 0.
6. [10, Exercise 3.10.25, p. 258] Boyle’s Law states that, when a sample of gas is
compressed at a constant temperature, the pressure P , and volume V satisfy the
equation P V = C, where C is a constant depending on the sample. Suppose that,
at a certain instant, the volume is 600 cm3 , the pressure is 150kPa, and the pressure
is increasing at a rate of 20 kPa/minute. At what rate is the volume decreasing at
this instant?
Solution: [12, Exercise 3.10.25, p. 100]
7. Use differentials (or, equivalently, a linear approximation) to estimate the following
numbers:
(a) [10, Exercise 3.11.34, p. 265]
(b) [10, Exercise 3.11.35, p. 265]
(c) [10, Exericse 3.11.36, p. 265]
Solution:
(a) Let y = f (x) = x6 , so dy = 6x5 dx. (1.97)6 = (2 − 0.03)6 = f (2 − 0.03) ≈
f (2) + f 0 (2) · (−0.03) = 26 + 6 · 25 · (−0.03) = 58.24.

UPDATED TO October 25, 2004


Information for Students in MATH 140A 2003 09 3059

(b) [12, Exercise 3.11.35, p. 104]


dx
(c) Let y = g(x) = ln x. Then dy = g 0 (x) dx = . Then ln(1.07) = g(1+0.07) ≈
x
0.07
g(1) + g 0 (1) · 0.07 = ln 1 + = 0.07.
1
8. [10, Exercise 4.2.32, p. 294] Prove the identity

2 arcsin x = arccos(1 − 2x2 ) (x ≥ 0)

by using the method of [10, Example 6, p. 2902].


1
Solution: Define f (x) = 2 arcsin x − arccos(1 − 2x2 ). Then f 0 (x) = √ −
1 − x2
−1 1 4x 1 4x
p ·(0−4x) = 2 √ −p = 2√ − √ =
1 − (1 − 2x2 )2 1−x 2 22 |x|2 (1 − x2 ) 1 − x 2x 1 − x2
2

0; note that we have used the hypothesis that x > 0. Hence, by [10, Theorem 4.2.5,
p. 291], f (x) =constant. We can determine the value of the constant by select-
π
ing a “convenient” value; for example, take x = 1: then, since arcsin 1 = , and
2
arccos(−1) = π, we find that the constant is equal to 0.
Note that this argument does not hold for x = 0. There, however, we can simply
observe that f (0) = 2 arcsin 0 − arccos 1 = 2 · 0 − 0 = 0.

9. [10, Exercise 4.2.6] Let f (x) = (x − 1)−2 . Show that f (0) = f (2), but there is no
number c such that 0 < c < 2 and f 0 (c) = 0. Why does this not contradict Rolle’s
Theorem?
Solution: f (0) = (−1)−2 = 1; f (2) = 1−2 = 1. Since f 0 (x) = −2(x − 1)−3 ,
wherever it is defined, the value of the derivative ranges, as 0 ≤ x < 1, from 2 to
∞; similarly, as 1 < x ≤ 2, the derivative ranges between −∞ and −2. Thus the
derivative never takes values between −2 and 2.
This is not a counterexample to Rolle’s Theorem, since that theorem requires
that the function be differentiable throughout the open interval, and the present
function fails to be differentiable at one point in the interval — namely the point
x = 1. That failure to be differentiable, even though it is only at a single point,
renders Rolle’s Theorem inapplicable.

10. [10, Exercise 4.2.17, p. 293] Show that the polynomial x5 + 10x + 3 has exactly one
real root.
Solution: f (0) = 3 > 0, but f (−1) = −12 < 0; by the Intermediate Value Theorem
this function — which, being a polynomial, is continuous everywhere, in particular

UPDATED TO October 25, 2004


Information for Students in MATH 140A 2003 09 3060

in the interval −1 < x < 0 — must take on the value 0 at least once between −1
and 0. However, the Intermediate Value Theorem does not exclude the possibility
that there is more than one place where f = 0. If, however, f were to vanish at
distinct points x1 and x2 , then, by Rolle’s theorem, f 0 would be zero somewhere
between x1 and x2 . But f 0 (x) = 5x4 + 10; being the sum of a square and a positive
number, this cannot be zero.

11. For each of the following functions

• find the “critical numbers”;


• find the vertical and horizontal asymptotes;
• find the intervals of increase or decrease;
• find the local maximum and minimum values;
• find the intervals of concavity and the inflection points;
• use the information you have determined above to sketh the graph.

(A “critical number” of a function is a point in the domain of the function where


either the derivative is not defined, or the derivative is defined and equal to zero.
This term is sometimes used (by other authors) with a slightly different definition.)

(a) (cf. [10, Exercise 4.3.39, p. 304]) f1 (x) = x2 + 1 − x
1 + x2
(b) [10, Exercise 4.3.43, p. 304] f2 (x) =
1 − x2

(c) [10, Exercise 4.3.45, p. 304] f3 (x) = x2 + 1 − x
(d) [10, Exercise 4.3.47, p. 304] f4 (x) = ln(1 − ln x)
1
(e) [10, Exercise 4.3.49, p. 304] f5 (x) = e− x+1

Solution:
µ ¶ µ ¶
3 3
(a) (cf. [12, Exercise 4.3.39, p. 132]) Since f1 (x) = x 1 + , and since 1 + →
x x
1 as either x → ∞ or x → −∞, lim f1 (x) = ±∞, so there are no horizontal
x→±∞
asymptotes. All limits of f1 (x) as x approaches any (finite) point are finite,
so there are no vertical asymptotes either. (However, the graph of f1 does
have a vertical tangent at x = −3.)
(b) [12, Exercise 4.3.43, p. 133]. The derivative is defined everywhere in the
domain of the function, and vanishes when x = 0, so x = 0 is the unique
“critical number”.

UPDATED TO October 25, 2004


Information for Students in MATH 140A 2003 09 3061

(c) [12, Exercise 4.3.45, p. 134]. The derivative is defined everywhere, and never
vanishes, so there are no “critical numbers”.
(d) [12, Exercise 4.3.47, p. 134]. The derivative is defined everywhere in the
domain of the function, and never vanishes, so there are no “critical numbers”.
(e) [12, Exercise 4.3.49, p. 134]. The derivative is defined everywhere in the
domain of the function, and never vanishes, so there are no “critical numbers”.

E.3.6 Sixth 2000/2001 Problem Assignment, with Solutions


1. In each of the following cases you are to find the limit, if it exists. Since the subject
of [10, §4.4] is l’Hôpital’s Rule, you should try to use that rule, if it is possible.
However, if l’Hôpital’s Rule cannot be applied, explain why. Mathematicians usu-
ally regard the avoidance of l’Hôpital’s Rule as a challenge; try to find another way
to evaluate those limits where you use the Rule.
ex − 1
(a) [10, Exercise 4.4.9, p. 312] lim
x→0 sin x
ln x
(b) [10, Exercise 4.4.15, p. 312] lim
x→∞ x
ln x
(c) [10, Exercise 4.4.17, p. 312] lim+
x→0 x
1 − cos x
(d) [10, Exercise 4.4.27, p. 312] lim
x→0 x2
2x − arcsin x
(e) [10, Exercise 4.4.37, p. 312] lim
x→0 2x + arccos x
2x − arcsin x
(f) [10, Exercise 4.4.38, p. 312] lim
x→0 2x + arctan x
³ √ ´
(g) [10, Exercise 4.4.51, p. 312] lim x − x2 − 1
x→∞
³ 1
´
(h) [10, Exercise 4.4.57, p. 312] lim (1 − 2x) x
x→0

Solution: We shall follow the notation of the Student Solution Manual [12], and
H
write = when we are applying l’Hôpital’s Rule.

(a) [12, Exercise 4.4.9, p. 138]


ex − 1 H ex
lim = lim
x→0 sin x x→0 cos x

UPDATED TO October 25, 2004


Information for Students in MATH 140A 2003 09 3062

lim ex e0
x→0
= = =1
lim cos x cos 0
x→0
ex − 1 ex − 1 x
lim = lim ·
x→0 sin x x→0 x sin x
ex − e0 x
= lim · lim (Product Law)
x→0 x x→0 sin x
¯
d x ¯¯
= e ·1
dx ¯x=0

= e0 = 1

(b) [12, Exercise 4.4.15, p. 138].


(c) L’Hôpital’s rule cannot be used, since the limit of the numerator is (negatively)
infinite, while the limit of the denominator is 0. As x → 0+ , ln x → −∞,
1 1
while → +∞, so the product ln x · approaches −∞.
x x
(d) [12, Exercise 4.4.27, p. 138]. Here is one way to avoid using l’Hôpital’s Rule:
µ ¶
1 − cos x 1 − cos x 1 + cos x
lim = lim ·
x→0 x2 x→0 x2 1 + cos x
µ ¶ õ ¶2 !
1 − cos2 x 1 sin x 1
= lim · = lim ·
x→0 x2 1 + cos x x→0 x 1 + cos x
µ µ ¶¶2
sin x 1
= lim ·
x→0 x lim (1 + cos x)
x→0
1 1
= 12 · =
1+1 2
(e) [12, Exercise 4.4.37, p. 138].
(f)
1
2− √
2x − arcsin x H 1 − x2
lim = lim
x→0 2x + arctan x x→0 1
2+
1 + x2

2 − lim 1 1 − x2
x→0
=
1
2 + lim
x→0 1 + x2

UPDATED TO October 25, 2004


Information for Students in MATH 140A 2003 09 3063

2−1 1
= =
2+1 3
It is possible to avoid the Rule, but the result looks very much like what we
have proved above:
arcsin x − arcsin 0
2x − arcsin x 2−
lim = lim x−0
x→0 2x + arctan x x→0 arctan x − arctan 0
2+
x−0
arcsin x − arcsin 0
2 − lim
=
x→0 x−0
arctan x − arctan 0
2 + lim
x→0 x−0
¯
d ¯
2− arcsin x¯¯
dx
= ¯ x=0
d ¯
2+ arctan¯¯
dx x=0

1
2− √
1 − 02 1
= =
1 3
2+ 2
1+1

(g) [12, Exercise 4.4.51, p. 139].


(h) [12, Exercise 4.4.57, p. 139].

2. Discuss each of these functions under the following headings, using the guidelines
of the same names in [10, pp. 315–316]

A. Domain
B. Intercepts
C. Symmetry
D. Horizontal or Vertical Asymptotes. (Do not attempt to investigate slant as-
ymptotes.)
E. Intervals of Increase or Decrease
F. Local Maximum and Minimum Values
G. Concavity and Points of Inflection

UPDATED TO October 25, 2004


Information for Students in MATH 140A 2003 09 3064

Then sketch the graph of the function.

(a) [10, Exercise 4.5.5, p. 321] f1 (x) = x4 + 4x3


1
(b) [10, Exercise 4.5.17, p. 321] f2 (x) = 3
x −x
p
(c) [10, Exercise 4.5.29, p. 321] f3 (x) = x + |x|
(d) [10, Exercise 4.5.47, p. 322] f4 (x) = ln(x2 − x)
(e) [10, Exercise 4.5.39, p. 322] f5 (x) = sin(2x) − 2 sin x

Solution:

(a) [12, Exercise 4.5.5, p. 142]


(b) [12, Exercise 4.5.17, p. 145]
(c) [12, Exercise 4.5.29, p. 147]
(d) [12, Exercise 4.5.47, p. 150]
(e) [12, Exercise 4.5.39, p. 148]

3. Before attempting these problems, try [10, Exercise 4.7.7, p. 335]; then compare
your solution with that in [12, Exercise 4.7.7, p. 162].

(a) [10, Exercise 4.7.11, p. 335] If 1200 cm2 of material is available to make a box
with a square base and an open top, find the largest possible volume of the
box.
(b) [10, Exercise 4.7.15, p. 335] Showing all your work, use the calculus to find
the point on the line y = 4x + 7 that is closest to the origin. (Use the calculus,
even though you may know methods for solving this problem that require no
calculus.)
(c) [12, Exercises 4.7.25 and 4.7.27, p. 162] A right circular cylinder is inscribed in
a sphere of radius r. Showing all your work, find the largest possible volume
and the largest possible surface area of such a cylinder. (You may assume
that the cylinder has been inscribed so that its axis passes through the centre
of the sphere.)

Solution:

(a) [12, Exercise 4.7.11, p. 162]


(b) [12, Exercise 4.7.15, p. 163]

UPDATED TO October 25, 2004


Information for Students in MATH 140A 2003 09 3065

(c) The solutions in [12, Exercises 4.7.25 and 4.7.27, p. 164] use, as the variable,
either the half-height or the radius of the base of the inscribed cylinder. You
might wish to try to solve the problem another way, using, as your variable,
the angle subtended at the centre of the sphere by the radius of the base.

4. Showing all your work, find the most general antiderivative of the following func-
tions. Check your answer by differentiation.

(a) [10, Exercise 4.10.3, p. 356] g1 (x) = 1 − x3 + 5x5 − 3x7


√ √
(b) [10, Exercise 4.10.7, p. 356] g2 (x) = x + 3 x
− 12
(c) [10, Exercise 4.10.15, p. 356] g3 (x) = 2x + 5 (1 − x2 )

Solution:

(a) [12, Exercise 4.10.3, p. 175]


(b) [12, Exercise 4.10.7, p. 175]
(c) [12, Exercise 4.10.15, p. 175]

5. Showing all your work, find the functions that have the listed properties. Check
your answers by differentiation and substitution in the differential equation.

(a) [10, Exercise 4.10.19, p. 356] h001 (x) = 6x + 12x2


√ 1
(b) [10, Exercise 4.10.27, p. 356] h02 (x) = 3 x − √ , h2 (1) = 2.
x
1
(c) [10, Exercise 4.10.39, p. 357] h003 (x) = 3 , x > 0, h3 (1) = 0 = h3 (2).
x
Solution:

(a) [12, Exercise 4.10.19, p. 175]


(b) [12, Exercise 4.10.27, p. 176]
(c) [12, Exercise 4.10.39, p. 176]

F Some Tests and Quizzes from Previous Years


F.1 Fall 1998 Class Quiz, with Solutions
(This quiz was given to both sections of 189-140A in the middle of October, 1998. No
grades were recorded. The solutions were posted on the Web.)

UPDATED TO October 25, 2004


Information for Students in MATH 140A 2003 09 3066

1. Determine the global maxima and global minima of the function


f (x) = 5x2/3 − x5/3 on the interval −2 ≤ x ≤ 4. (You may take 1.6 as an ap-
1
proximation to 4 3 . You may find it helpful to make a rough sketch, but your
solutions must not depend on that sketch.)
Solution:
2 −1 5 2
f 0 (x) = 5 · · x 3 − · x3
3 3
5(2 − x)
= 1 ,
3x 3
provided x 6= 0. When x = 0, the function lacks a derivative. The derivative
vanishes (i.e. is equal to zero) if and only if x = 2. Thus the list of critical points71
for this function, whose domain is the closed interval [−2, 4], is
Points where there is no derivative:
(0, f (0)) = (0, 0)

Points where the derivative vanishes:


2
(2, f (2)) = (2, 3 · 2 3 )

End-points of the domain of definition:


1
(−2, f (−2)) = (−2, 7 · 4 3 )
2
(4, f (4)) = (4, 1 · 4 3 )
Both the global maximum and the global minimum must be attained at points
among these 4. Comparing the function values — if necessary students could have
1
used the approximation of 1.6 for 4 3 — shows that the maximum occurs at the
1
point (−2, 7 · 4 3 ), and the minimum at the point (0, 0).
(A graph of this function may be found in [21, Example 6 of §3.5].)
2. A rectangle, with its base on the x-axis, is constructed so that its upper two vertices
are on the semi-circle x2 + y 2 = 4, y ≥ 0. Determine the maximum possible area
for such a rectangle, and determine where the maximum value is attained.
Solution: There are several ways of approaching this problem; we present two.
71
The definition [21, p. 144] of critical point in the text-book is unclear as to whether or not the
end-points of the interval of definition are to be considered critical . If you choose to consider end-points
as not being included in this definition, then you must include them with the (other) critical points in
your list of candidates for extrema.

UPDATED TO October 25, 2004


Information for Students in MATH 140A 2003 09 3067

(a) A rectangle inscribed in the semicircle must have its upper side parallel to the
x-axis, so the coordinates of the upper vertices
√ may be taken to be (±x, y);
as x2 + y 2 = 4, and y √
is non-negative, y = 4 − x2 . Thus the vertices of the
rectangle will be (±x, 4 − x2 ) and (±x, 0). The area, which we may denote
by A(x) is then given by

A(x) = |(2x) · 4 − x2 | .

(The absolute signs are needed since the intention of the problem is that
the area should be non-negative.) We can now approach the problem in two
equivalent ways.
i. We may√ take the domain of A(x) to be −2 ≤ x ≤ 2, since the circle has
radius 4 = 2. Differentiating yields
√ 1 1
A0 (x) = 2 4 − x2 + 2x · √ · (−2x)
2 4 − x2
4 − 2x2
= √
4 − x2
However, this is valid only for x > 0, because of the absolute signs; for
x<0
2x2 − 4
A0 (x) = √ .
4 − x2
Note that there is no derivative at x = 0.
Thus the critical points are

(0, A(0)) = (0, 0)


√ √ √
(− 2, A(− 2)) = (− 2, 4)
√ √ √
( 2, A( 2)) = ( 2, 4)
and the end points,
(−2, f (−2)) = (−2, 0)
(2, f (2)) = (2, 0)

Comparing the values, we√find the maximum of A = 2 to be attained at


both of the points x = ± 2. The minimum value of 0 is attained at 3
points: x = ±2 and x = 0.
ii. The preceding approach can be modified, in that we can restrict the
domain to be 0 ≤ x ≤ 2, by the symmetry of the function. In this

UPDATED TO October 25, 2004


Information for Students in MATH 140A 2003 09 3068

approach the only critical points are


√ √ √
( 2, A( 2)) = ( 2, 4)
and the end points,
(0, A(0)) = (0, 0)
(2, A(2)) = (2, 0)

and we obtain the same extreme values.


(b) A more elegant approach would observe that all points on the semi-circle have
coordinates of the form (2 cos t, 2 sin t), where 0 ≤ t ≤ π. Here again we could
treat two cases, according as we permit the domain to be the full interval
stated, or only 0 ≤ t ≤ π2 . In either case the area is α(t) = |4 cos t · 2 sin t| =
|4 sin 2t|. For convenience we will take the domain to be 0 ≤ t ≤ π2 . Then,
observing that α(t) = 4 sin 2t, we have α0 (t) = 4(cos 2t) · 2, which vanishes
only where 2t = π2 , so t = π4 . Other than the end-points of the interval, this
is the only critical point, and the area there is α( π4 ) = 4 sin π2 = 4. At the
end-points we have α(0) = sin 0 = 0, and α( π2 ) = 0. Thus the maximum value
of 4 occurs only at t = π4 , and the minimum value of 0 occurs at t = 0 and
t = π2 .

F.2 Last Three Tutorial Quizzes in 2000/2001 (many versions)


F.2.1 Fourth 2000/2001 Tutorial Quizzes
T01

Distribution Date: Monday, October 23rd, 2000 — 13:30 to 14:30 h.

1. Evaluate the following limit, but do not use L’Hôpital’s rule:

sin(x2 − 1)
lim .
x→1 (x − 1)

dy 2
2. Find if y = 3x .
dx
1
3. 72
Find an equation for the tangent line to the graph of exy = x − +e at the
¡ 1¢ y
point e, e .
72
corrected, 7 November 2000

UPDATED TO October 25, 2004


Information for Students in MATH 140A 2003 09 3069

T02, T03, T05


Distribution Date: Monday, October 23rd, 2000 — 14:30 to 15:30 h.

1. Evaluate the following limit, but do not use L’Hôpital’s rule:


x + tan x
lim .
x→0 sin x
dy
2. Find if y = tan(3x ).
dx
3. Find an ³equation ´ for the tangent line to the graph of x2 (x2 + y 2 ) = y 2 at the
1 1
point √ , √
2 2
.

T04, T06
Distribution Date: Monday, October 23rd, 2000 — 15:30 to 16:30 h.

1. Use the limit definition of the derivative to find f 0 (0) for


½ 2 ¡ ¢
x sin x1 for x 6= 0
f (x) = .
0 for x = 0

dy 2
2. Find if y = sec(5x ) .
dx
3. If the line y = 3x − 1 is tangent to the graph of y = f (x) at the point
with x = 1 , find the equation of the tangent line to the graph of y = [f (x)]2
at the point with x = 1 .

T07
Distribution Date: Wednesday, October 25th, 2000 — 13:30 to 14:30 h.

1. Use the limit definition of the derivative to find f 0 (0) for


( 1 − cos x
when x = 6 0
f (x) = x .
0 when x = 0

dy 1
2. Find if y = 2 2x .
dx
3. If g(x) = 2x · f (x2 − 2x + 2) and f (1) = 3 , find g 0 (1) .

UPDATED TO October 25, 2004


Information for Students in MATH 140A 2003 09 3070

T08, T09, T11


Distribution Date: Wednesday, October 25th, 2000 — 14:30 to 15:30 h.
1. Use the limit definition of the derivative to find f 0 (8) for f (x) = x1/3 .
µ ¶
dy 1
2. Find if y = x tan .
dx 3x

3. If f (x) = ex·g(x) and g(0) = 5 , find f 0 (0) .

T10, T12, T15


Distribution Date: Wednesday, October 25th, 2000 — 15:30 to 16:30 h.
1. Use the limit definition of the derivative to find f 0 (1) for f (x) = x−1/3 .
dy ¡1¢
2. Find if y = x sec−1 2x
.
dx
3. Find the equation(s) of the line(s) through the origin, tangent to the graph of
y = ex .

T16
Distribution Date: Wednesday, October 25th, 2000 — 16:30 to 17:30 h.
1. Use the limit definition of the derivative to find f 0 (0) for f (x) = sec x .
µ ¶
dy −1 2x
2. Find if y = tan .
dx 1 − x2
3. Find equations(s) of the line(s) through the origin, tangent to the graph of
y = x2 + 9 .

T13
Distribution Date: Thursday, October 26th, 2000 — 16:00 to 17:00 h.
1. Use the limit definition of the derivative to find f 0 (0) for f (x) = ex sin x .
µ x ¶
dy −1 e − e−x
2. Find if y = tan .
dx 2
3. Find the equations(s) of the line(s) through the origin, tangent to the graph of
2
y = ex .

UPDATED TO October 25, 2004


Information for Students in MATH 140A 2003 09 3071

T14

Distribution Date: Thursday, October 26th, 2000 — 17:00 to 18:00 h.

1. Use the limit definition of the derivative to find f 0 (0) for

f (x) = e3x (1 − cos x) .


µ ¶
dy −1 x
2. Find if y = sin √ .
dx 1 + x2
3. Find equations(s) of the line(s) through the origin, tangent to the graph of

y = e2x .

F.2.2 Fifth 2000/2001 Tutorial Quizzes


T01

Distribution Date: Monday, November 13th, 2000 — 13:30 to 14:30 h.

dy x
1. Find dx if y = x x−1 .
2. Water is pouring into a leaky tank at the rate of 10m3 /h. The tank
is a cone with the vertex down, 9m deep and 6m in diameter at the
top. The surface of the water is rising at a rate of 0.2m/h when the
depth is 6m. How fast is the water leaking out at that time?

T02, T03, T05

Distribution Date: Monday, November 13th, 2000 — 14:30 to 15:30 h.



x
dy
1. Find dx if y = x x−1 .
2. How fast must you let out the line if the kite you are flying is 30m
high, 40m horizontally away from you, and moving horizontally away
from you at 10m/min?

UPDATED TO October 25, 2004


Information for Students in MATH 140A 2003 09 3072

T04, T06
Distribution Date: Monday, November 13th, 2000 — 15:30 to 16:30 h.
dy √
1. Find dx if y = ( x + 1)x .
2. A ferris wheel you are riding has diameter 20m and is rotating at 1
revolution per minute. How fast are you rising or falling when you
are 6m horizontally away from the vertical line through the centre of
the wheel?

T07
Distribution Date: Wednesday, November 13th, 2000 — 13:30 to 14:30 h.
dy
1. Find dx if y = (x2 + 1)1/x .
2. A policeman stationed at a fixed distance from a highway aims a
radar gun at a car. When the gun is pointing at an angle of 45◦ to
the highway, the radar gun records the rate at which the distance of
the car from the gun is increasing at 100km/h. How fast is the car
travelling?

T08, T09, T11


Distribution Date: Wednesday, November 13th, 2000 — 14:30 to 15:30 h.
dy
1. Find dx if y = (xx + 1)1/2 .
2. [CORRECTED] A balloon is released and rises vertically from point
A and is tracked from point B, 100m horizontally away. If the balloon
is rising at the rate of k m/sec., how fast is the angle of elevation of
the balloon, as observed at point B, increasing when it is 200m above
A?

T10, T12, T15


Distribution Date: Wednesday, November 13th, 2000 — 15:30 to 16:30 h.
dy
1. Find dx if y = xln x .
2. A man 6ft tall walks towards a building at the rate of 5ft/sec. If
there is a light on the ground 50ft away from the building, how fast
is the man’s shadow shrinking when he is 30ft from the building?

UPDATED TO October 25, 2004


Information for Students in MATH 140A 2003 09 3073

T16

Distribution Date: Wednesday, November 13th, 2000 — 16:30 to 17:30 h.

1
dy
1. Find dx if y = x 2x .
2. A spherical balloon is being inflated so that its volume is increasing
at the rate of 5m3 /min. At what rate is the surface area increasing
when the radius is 6m?
(Volume = 43 πr3 , Area = 4πr2 , where r is the radius.)

T13

Distribution Date: Thursday, November 14th, 2000 — 16:00 to 17:00 h.

dy
1. Find dx if y = (x2x + 1)1/3 .
2. A 10m long ladder has one end on the ground, and is supported part
way along its length by a fence 3m high, so that part of the ladder
projects past the fence. If the end on the ground is 4m from the base
of the fence and is being dragged away at 0.2m/sec, how fast is the
vertical height of the other end of the ladder changing?

T14

Distribution Date: Thursday, November 14th, 2000 — 17:00 to 18:00 h.

dy
¡ 1 ¢x
1. Find dx if y = 2x .
2. A light shines from a pole 50ft high. A ball is dropped from the same
height from a point 30ft away from the light. In t secs the ball falls
16t2 ft. How fast is the shadow of the ball moving along the ground
after 0.5 secs?

UPDATED TO October 25, 2004


Information for Students in MATH 140A 2003 09 3074

F.2.3 Sixth 2000/2001 Tutorial Quizzes


T01

Distribution Date: Monday, November 27th, 2000 — 13:30 to 14:30 h.

Let f (x) = (2x2 + x + 1)ex .

1. [5 MARKS] Determine the domain of f .

2. [5 MARKS] Determine all horizontal asymptotes and all vertical asymptotes of the
graph of f .

3. [5 MARKS] Determine the intervals where f increases, and the intervals where it
decreases.

4. [5 MARKS] Determine all local maxima and all local minima of f .

5. [5 MARKS] Determine the intervals where the graph is concave upwards, and the
intervals where it is concave downwards.

6. [5 MARKS] Determine all inflection points of the graph.

7. [5 MARKS] Sketch the graph.


xa
(You may use, without proof, the fact that lim = 0 for any real number a.)
x→∞ ex

T02, T03, T05

Distribution Date: Monday, November 27th, 2000 — 14:30 to 15:30 h.

Let f (x) = (2x2 − 3x + 2)ex .

1. [5 MARKS] Determine the domain of f .

2. [5 MARKS] Determine all horizontal asymptotes and all vertical asymptotes of the
graph of f .

3. [5 MARKS] Determine the intervals where f increases, and the intervals where it
decreases.

4. [5 MARKS] Determine all local maxima and all local minima of f .

5. [5 MARKS] Determine the intervals where the graph is concave upwards, and the
intervals where it is concave downwards.

UPDATED TO October 25, 2004


Information for Students in MATH 140A 2003 09 3075

6. [5 MARKS] Determine all inflection points of the graph.

7. [5 MARKS] Sketch the graph.


xa
(You may use, without proof, the fact that lim = 0 for any real number a.)
x→∞ ex

T04, T06

Distribution Date: Monday, November 27th, 2000 — 15:30 to 16:30 h.

Let f (x) = (6x2 + x + 5)ex .

1. [5 MARKS] Determine the domain of f .

2. [5 MARKS] Determine all horizontal asymptotes and all vertical asymptotes of the
graph of f .

3. [5 MARKS] Determine the intervals where f increases, and the intervals where it
decreases.

4. [5 MARKS] Determine all local maxima and all local minima of f .

5. [5 MARKS] Determine the intervals where the graph is concave upwards, and the
intervals where it is concave downwards.

6. [5 MARKS] Determine all inflection points of the graph.

7. [5 MARKS] Sketch the graph.


xa
(You may use, without proof, the fact that lim = 0 for any real number a.)
x→∞ ex

T07

Distribution Date: Wednesday, November 29th, 2000 — 13:30 to 14:30 h.

Let f (x) = (2x2 − 5x + 4)ex .

1. [5 MARKS] Determine the domain of f .

2. [5 MARKS] Determine all horizontal asymptotes and all vertical asymptotes of the
graph of f .

3. [5 MARKS] Determine the intervals where f increases, and the intervals where it
decreases.

UPDATED TO October 25, 2004


Information for Students in MATH 140A 2003 09 3076

4. [5 MARKS] Determine all local maxima and all local minima of f .

5. [5 MARKS] Determine the intervals where the graph is concave upwards, and the
intervals where it is concave downwards.

6. [5 MARKS] Determine all inflection points of the graph.

7. [5 MARKS] Sketch the graph.


xa
(You may use, without proof, the fact that lim = 0 for any real number a.)
x→∞ ex

T08, T09, T11


Distribution Date: Wednesday, November 29th, 2000 — 14:30 to 15:30 h.
1 64
Let f (x) = − .
x−1 x+1
1. [5 MARKS] Determine the domain of f .

2. [5 MARKS] Determine all horizontal asymptotes and all vertical asymptotes of the
graph of f .

3. [5 MARKS] Determine the intervals where f increases, and the intervals where it
decreases.

4. [5 MARKS] Determine all local maxima and all local minima of f .

5. [5 MARKS] Determine the intervals where the graph is concave upwards, and the
intervals where it is concave downwards.

6. [5 MARKS] Determine all inflection points of the graph.

7. [5 MARKS] Sketch the graph.

T10, T12, T15


Distribution Date: Wednesday, November 29th, 2000 — 15:30 to 16:30 h.
64 1
Let f (x) = − .
x−1 x+1
1. [5 MARKS] Determine the domain of f .

2. [5 MARKS] Determine all horizontal asymptotes and all vertical asymptotes of the
graph of f .

UPDATED TO October 25, 2004


Information for Students in MATH 140A 2003 09 3077

3. [5 MARKS] Determine the intervals where f increases, and the intervals where it
decreases.

4. [5 MARKS] Determine all local maxima and all local minima of f .

5. [5 MARKS] Determine the intervals where the graph is concave upwards, and the
intervals where it is concave downwards.

6. [5 MARKS] Determine all inflection points of the graph.

7. [5 MARKS] Sketch the graph.

T16

Distribution Date: Wednesday, November 29th, 2000 — 16:30 to 17:30 h.


1 64
Let f (x) = − .
x−1 x+8
1. [5 MARKS] Determine the domain of f .

2. [5 MARKS] Determine all horizontal asymptotes and all vertical asymptotes of the
graph of f .

3. [5 MARKS] Determine the intervals where f increases, and the intervals where it
decreases.

4. [5 MARKS] Determine all local maxima and all local minima of f .

5. [5 MARKS] Determine the intervals where the graph is concave upwards, and the
intervals where it is concave downwards.

6. [5 MARKS] Determine all inflection points of the graph.

7. [5 MARKS] Sketch the graph.

T13

Distribution Date: Thursday, November 30th, 2000 — 16:00 to 17:00 h.


1 16
Let f (x) = − .
4x + 1 x + 2
1. [5 MARKS] Determine the domain of f .

UPDATED TO October 25, 2004


Information for Students in MATH 140A 2003 09 3078

2. [5 MARKS] Determine all horizontal asymptotes and all vertical asymptotes of the
graph of f .

3. [5 MARKS] Determine the intervals where f increases, and the intervals where it
decreases.

4. [5 MARKS] Determine all local maxima and all local minima of f .

5. [5 MARKS] Determine the intervals where the graph is concave upwards, and the
intervals where it is concave downwards.

6. [5 MARKS] Determine all inflection points of the graph.

7. [5 MARKS] Sketch the graph.

T14

Distribution Date: Thursday, November 30th, 2000 — 17:00 to 18:00 h.

Let f (x) = ln |x + 2| − 4 ln |x − 1| .

1. [5 MARKS] Determine the domain of f .

2. [5 MARKS] Determine all horizontal asymptotes and all vertical asymptotes of the
graph of f .

3. [5 MARKS] Determine the intervals where f increases, and the intervals where it
decreases.

4. [5 MARKS] Determine all local maxima and all local minima of f .

5. [5 MARKS] Determine the intervals where the graph is concave upwards, and the
intervals where it is concave downwards.

6. [5 MARKS] Determine all inflection points of the graph.

7. [5 MARKS] Sketch the graph.

UPDATED TO October 25, 2004


Information for Students in MATH 140A 2003 09 3079

G Examinations from Previous Years


1. These examinations have been taken from the original versions in computer files.
Occasionally an examination has been changed or corrected in the examination
room: such changes may not have been incorporated into these versions.

2. Listed below are only the texts of the problems on most of the examinations from
1996-2003; the actual formats of most of the examinations can be seen at the
following URL:

http://www.math.mcgill.ca/brown/math140a.html

3. It is likely that the form and format of the examination in MATH 140 will undergo
changes in December 2003. Further information may be given at the lectures.

4. The syllabus on which the following examinations was based has undergone only
minor changes over the years. Some changes derive from the use of different text-
books. Notwithstanding such changes, the examinations can serve as a useful
indication of what a student in Math 140 should know by examination time.

G.1 December 1996 Final Examination in 189-122A


1
x
− 12
1. [4 MARKS] Evaluate lim .
x→2 x−2
x−9
2. [4 MARKS] Evaluate lim √ .
x→9 x−3
3. [4 MARKS] Evaluate lim x cot 3x .
x→0

4. [5 MARKS] Find an equation for the straight line which is normal to the graph of
f (x) = x2 at x = −3 .

5. [8 MARKS] The surface area of a sphere is increasing at a rate of 10 square metres


per hour. At what rate is the volume increasing, when the radius is 2 metres?
(Note: The volume of a sphere of radius r is 34 πr3 ; the surface area is 4πr2 .)

6. [8 MARKS] Show that the equation 6x4 − 7x + 1 = 0 does not have more than
two distinct real roots.

7. [10 MARKS] A right circular cylinder is inscribed in a right circular cone of height
H and radius R. Determine the dimensions of the cylinder with the largest possible
volume. What is that largest volume?

UPDATED TO October 25, 2004


Information for Students in MATH 140A 2003 09 3080

1
8. [8 MARKS] Show that the function f (x) = 2 + (1 − x3 ) 5 has an inverse. Deter-
mine f −1 (x) .

9. The function f (x) = x − cos x (−π ≤ x ≤ π) is differentiable.

(a) [2 MARKS] Show that this function has an inverse.


(b) [4 MARKS] Calculate the derivative of the inverse function.
(c) [4 MARKS] Evaluate the derivative of f −1 (x) at all points x such that f (x) =
−1 .

10. [8 MARKS] Use the mean value theorem to show that, when
x>1,
x−1
< ln x < x − 1 .
x
µ ¶
x4
11. Let f (x) = ln .
x−1
(a) [3 MARKS] Specify the domain of f .
(b) [3 MARKS] Determine the interval(s) where f increases, and the interval(s)
where f decreases.
(c) [3 MARKS] Determine the concavity of the graph of f , and find the points
of inflection.
(d) [3 MARKS] Sketch the graph, using the information determined above.

12. Let f (x) = xe−x .

(a) [3 MARKS] Specify the domain of f .


(b) [3 MARKS] Determine the interval(s) where f increases, and the interval(s)
where it decreases.
(c) [3 MARKS] Determine the concavity of the graph of f , and find the points
of inflection.
(d) [3 MARKS] Sketch the graph, using the information determined above.

13. [7 MARKS] Given the curve x3 + y 3 = 1 + 3xy 2 , verify that the point (x, y)
dy d2 y
= (0, 1) is on the curve. Find and 2 at (x, y) = (0, 1) .
dx dx

UPDATED TO October 25, 2004


Information for Students in MATH 140A 2003 09 3081

G.2 December 1997 Final Examination in 189-140A


x2 − 4
1. [8 MARKS] For the function f (x) =
|x − 2|
(a) Find the left-hand and right-hand limits at x = 2 .
(b) Determine whether the two-sided limit exists at x = 2 .
(c) Sketch the graph of y = f (x) .

2. [2 MARKS] Multiple Choice: Circle the correct answer (A, B, C, D, or E.)


√ √
9 + x − 9 − 3x
lim
x→0 x

A B C D E
2
=0 = =1 does not exist (or = is a real number r dif-
3 2
∞, or = −∞). ferent from 0, , 1.
3

3. [8 MARKS] Apply the intermediate value property of continuous functions to prove


that the equation x3 − 4x2 + 1 = 0 has at least three different solutions. (Hint:
Denoting f (x) = x3 − 4x2 + 1 , we have f (+1) = −2.)
60 − t
4. [8 MARKS] At time t, the radius of a leaking spherical balloon is r =
12
centimetres. Determine the rate (in cm.3 /second) at which the volume is decreasing
when t = 30 . (Hint: You may assume that the volume of a sphere of radius r is
4
3
πr3 .)

5. [8 MARKS] Determine an equation for the straight line that passes through the
point (1, 5) and is tangent to the curve y = x3 .

6. [2 MARKS] Multiple Choice: Circle the correct answer (A, B, C, D, or E.)

x − 4 tan x
lim
x→0 sin x

A B C D E
2
=0 = =1 does not exist (or = is a real number r dif-
3 2
∞, or = −∞). ferent from 0, , 1.
3

UPDATED TO October 25, 2004


Information for Students in MATH 140A 2003 09 3082

7. [8 MARKS] Showing all your work, determine the maximum area of a rectangle
with a base that lies on the x-axis, and with two upper vertices that lie on the
graph of the equation y = 4 − x2 .
8. [8 MARKS] A tank is in the shape of an inverted right circular cone of height 800
cm., whose top is a disk of radius 160 cm. Water is running out of a small hole
at the vertex (apex) of the cone, which is at the bottom. Showing all your work,
determine the rate of change of volume V with respect to height h, at a time when
the height is 600 cm. (Hint: You may assume that the volume of a right circular
cone of height h, whose base has radius r, is 31 πr2 h.)
9. [2 MARKS] Multiple Choice: Circle the correct answer (A, B, C, D, or E.)
dy π
If y = sin 2x cos 3x , the value of when x = is
dx 2
A B C D E
2 2
0 1 −1 different from 0, , 1, −1.
3 3

10. [8 MARKS] A covered rectangular box is to be constructed with volume 576 cubic
centimetres, with its bottom twice as long as it is wide. Determine the dimensions
of the box that will minimize its total surface area (including the cover).
11. [5 MARKS] Showing all your work, determine the maximum and minimum values
of f (x) = 3 − |x − 2| on the interval [1, 4] .
12. [5 MARKS] Multiple Choice: Circle the correct answer (A, B, C, D, or E.)
¡ ¢
The function f (x) = 2x2 − 3x e−x has a global maximum on the half-line x ≥ 0 .
The maximum value is
A B C D E

√ 1 9 9 −3
− e −√ e 4 none of the preceding
e e2 8
4 values.

13. [8 MARKS] Showing all your work, sketch the graph of the function
e2x
f (x) = ,
e2x + 3
identifying asymptotes, critical points, and inflection points. Show clearly where
the graph is concave upward and where it is concave downward.

UPDATED TO October 25, 2004


Information for Students in MATH 140A 2003 09 3083

14. [8 MARKS]

(a) State the Mean Value Theorem.


(b) Given that f (x) = |x| for −1 ≤ x ≤ 1 , does it follow from the Mean Value
Theorem that there exists a point c such that −1 ≤ c ≤ 1 and f 0 (c) = 0 ?
Explain your answer.

15. [2 MARKS] Multiple Choice: Circle the correct answer (A, B, C, D, or E.)
dy
If x5 − y 5 = 2x2 y 2 , the value of when (x, y) = (1, −1) is
dx

A B C D E
5 5 5 5
−1 1 − none of −1, 1, − , .
3 3 3 3

16. [10 MARKS] Showing all your work, sketch the graph of the function

x2 − x − 2
f (x) = − 1,
(1 − x)2
identifying asymptotes, critical points, and inflection points. Show clearly where
the graph is concave upward and where it is concave downward.

G.3 December 1998 Final Examination in 189-140A


x3 − 27
1. [4 MARKS] Showing your work, find lim .
x→3 x − 3

2. [5 MARKS] Given that a function f has the property that

|f (x) − 2| ≤ (x − 1)2 ,

determine lim f (x).


x→1

3. (a) [4 MARKS] Define v(x) = x3 − 4x2 + x + 3 . Use the Intermediate Value


Theorem to show that the equation v(x) = 0 has a solution between x = 1
and x = 2.
(b) [4 MARKS] By examining the behavior of v(x) as x → ∞ and as x → −∞,
or otherwise, discuss the existence of other solutions to v(x) = 0.

4. [8 MARKS]
√ Find equations for all straight lines which are both normal to the curve
y = x − 3 and parallel to the straight line y = −2x + 11.

UPDATED TO October 25, 2004


Information for Students in MATH 140A 2003 09 3084

5. For the function g(x) = |1 − x2 |,

(a) [3 MARKS] Sketch the graph of g.


(b) [3 MARKS] Show that g is continuous at x = 1.
(c) [2 MARKS] Determine whether g is differentiable at x = 1.
sin x
6. (a) [3 MARKS] Determine h0 (x) , if h(x) = .
1 − 2 cos x
(b) [3 MARKS] If u(x) = ln ((2x − 1)3 ), determine u0 (x).

7. [12 MARKS] Determine the slope of the curve

(x + y)2 − (x − y)2 = x4 + y 4

at all point(s), other than the origin, where the curve meets the line x = y.

8. [6 MARKS] A child is building a snowman by rolling a snowball on the ground; its


volume is increasing at the rate of 8 cubic centimetres per minute. Find the rate
at which the radius is increasing when the snowball is 75 centimetres in diameter.
(The volume of a sphere of radius r is 34 πr3 .)
£ ¤
9. [8 MARKS] The domain of the function F is to be taken to be the interval −2, 12 .
On that interval,
F (x) = x3 + x2 − x + 1 .
Determine the global maximum ( = absolute maximum), global minimum ( = ab-
solute minimum), local maxima ( = relative maxima), and local minima ( = relative
minima).

10. [10 MARKS] A closed box with a square base is to have a volume of 2, 000 cubic
centimetres. The material for the top and bottom of the box costs $3 per square
centimeter, while the material for the sides costs $1.50 per square centimetre.
Determine the dimensions of the least expensive box.

11. For the function f (x) = x2 ,


2
x −4
(a) [2 MARKS] Determine the (largest possible) domain.
(b) [4 MARKS] Determine all local extrema, and all points of inflection.
(c) [2 MARKS] Determine precisely where the function is increasing, decreasing,
concave upward, concave downward.
(d) [2 MARKS] Determine all horizontal and all vertical asymptotes, if any.

UPDATED TO October 25, 2004


Information for Students in MATH 140A 2003 09 3085

(e) [1 MARK] Sketch the graph.

12. It is given that the function f , defined by f (x) = x + x3 has an inverse, denoted
by f −1 .

(a) [3 MARKS] Determine the value of f −1 (2).


³ ´
(b) [3 MARKS] Determine the value of d (f −1 ) (2).
dx
13. [4 MARKS] Determine the derivative of the function

m(x) = xx .

14. [4 MARKS] Determine the value of the following limit, if it exists:


3
ex − 1
lim
x→0 x − sin x

G.4 May 1999 Supplemental Examination in 189-140A


x2 − 4
1. (a) Evaluate lim .
x→2 x − 2

(b) Let f be the function with domain ∞ < x < ∞ given by


½ 2
x − 2x if x ≤ 1
f (x) =
x−2 if x > 1

i. Is f continuous at x = 1? Explain (justify) your answer.


ii. Is f differentiable at x = 1? Explain (justify) your answer.

2. Find the derivative of each of the following functions. (You need not simplify your
answers.)

sin(2x2 − 1)
(a) F (x) =
(x2 + 1)3
µ ¶
−1 1
(b) G(x) = tan √
x3 + 1
(c) H(x) = x2 e−2x ln x

[a, b], and theorem. g 0 (x) > 0

3. It is given that the function f , defined by f (x) = x + x3 has an inverse, denoted


by f −1 .

UPDATED TO October 25, 2004


Information for Students in MATH 140A 2003 09 3086

(a) Determine the value of f −1 (2).


³ ´
(b) Determine the value of d −1
(f ) (2).
dx
4. (a) Find an equation for the line tangent at the point (1, 1) to the curve y 2 =
x3 (2 − x).
(b) A woman who is 1.75 metres tall walks at a rate of 1.5 metres per second
away from a lamp that is at the top of a 4-metre high lamp post. At what
rate is her shadow lengthening when she is 30 metres from the lamp post.

5. Find the area of the largest rectangle which can be inscribed in a semicircle of
radius 1, where one side of the rectangle lies on the diameter of the semicircle, and
the other two vertices lie on the semicircle.

6. Consider the function h(x) = 4x3 − 15x2 + 12x + 7, with domain −∞ < x < ∞.

(a) Find all points at which h has a local maximum, a local minimum, or a point
of inflection. Justify all of your answers.
(b) Find the global (absolute) maximum and the global (absolute) minimum of h
on the interval [0, 3].
2x2
7. Sketch the curve y = 2 , indicating any horizontal or vertical asymptotes.
x −1
8. Let the function u be defined by u(x) = ln(2x + 1).

(a) What is the (maximum possible) domain of u. (Explain.)


(b) Sketch the graph of u(x) = ln(2x + 1).

G.5 December 1999 Final Examination in 189-140A




 x + sin 3x
 x>0
tan 4x
1. Let f (x) = , where a is a constant, to be determined.

 (x − a)2
 x≤0
4
(a) [6 MARKS] Determine each of lim− f (x), lim+ f (x) or explain why either or
x→0 x→0
both do not exist.
(b) [3 MARKS] Use the information of part (a) to determine all values, if any, for
the constant a, which will make f continuous at x = 0.
2 −3
2. [8 MARKS] Let m(x) = xx − (x2 ) . Determine the value of m0 (1).

UPDATED TO October 25, 2004


Information for Students in MATH 140A 2003 09 3087

3. [8 MARKS] If y is defined implicitly as a function of x by 2x2 − 3xy + 5y 2 = 10,


d2 y
determine the value of 2 when (x, y) = (1, −1).
dx
2+x
4. For the function h(x) = arctan ,
1 − 2x
(a) [1 MARK] State the (largest possible) domain.
dh
(b) [5 MARKS] Determine for all points x where the derivative exists.
dx
(c) [2 MARKS] Give an example of a function different from h which has exactly
the same domain and exactly the same derivative as h.
x
5. The function u is defined by u(x) = , for −∞ < x < ∞.
1 + x2
(a) [3 MARKS] Determine the intervals where the function u is increasing, and
those where it is decreasing.
(b) [3 MARKS] Find all critical points. In each case determine whether the point
is a maximum or minimum point, or neither.
(c) [3 MARKS] Determine the intervals where the graph of u is concave upwards,
and those where it is concave downwards.
(d) [3 MARKS] Determine all inflection points of the graph.
(e) [2 MARKS] Determine all horizontal or vertical asymptotes of the graph.
(f) [4 MARKS] Sketch the graph.
(x − 1)(x + 1) 2x(x2 − 3)
You may assume that u0 (x) = − , and that u 00
(x) = .
(x2 + 1)2 (x2 + 1)3
(For each of parts (a) through (e) you are expected to show all your work and your
results, clearly marked by the question number, e.g., 5(c); it is not sufficient to
provide information only on your graph.)

6. [8 MARKS] Use the Intermediate Value Theorem and/or the Mean Value Theorem
1
and/or properties of G0 (x) to show that the function G(x) = x2 − e 1+x assumes
the value 0 for exactly one real number x such that 0 < x < 2. Show all your work.
1
[Hint: You may assume that e 3 < 2.]

7. [8 MARKS] Triangle OBC, in the first quadrant, has vertex O at the origin, vertex
B on the x-axis, and vertex C on the y-axis. If the vertices are constrained so that
the line joining B and C passes through the point (2, 3), determine the minimum
area for triangle OBC. Show all your work.

UPDATED TO October 25, 2004


Information for Students in MATH 140A 2003 09 3088

8. Showing all your work, evaluate the following limits, if they exist:
√ √
(a) [8 MARKS] lim ( x2 + x − x2 − x).
x→∞
tan x − sin x
(b) [8 MARKS] lim .
x→0 x3
9. [8 MARKS] Showing all your work, determine all lines with slope 3 which are
normal to the curve 64y + x3 = 0, (i.e. which are perpendicular to the tangent at
each point where they meet the curve).

10. [9 MARKS] Showing all your work, determine the (global) maxima and minima
of the function R(x) = 3x4 + 4x3 − 6x2 − 12x on the closed interval −2 ≤ x ≤ 2.
[Hint: x3 + x2 − x − 1 = (x2 − 1)(x + 1).]

G.6 December 1999 Special Final Examination in 189-140A


1 − x + ln x
1. [4 MARKS] Evaluate lim− .
x→1 1 + cos πx
2
2. [6 MARKS] When f (x) = x−2x , evaluate f 0 (1).
2x
3. [5 MARKS] Evaluate lim .
x→0 tan 3x
4. [5 MARKS] Find an equation for the straight line which is normal to the graph of
f (x) = x3 at x = −2 .

5. [10 MARKS] The volume of a sphere is increasing at a rate of 10 cubic metres per
hour. At what rate is the surface area increasing, when the radius is 2 metres?
(Note: The volume of a sphere of radius r is 34 πr3 ; the surface area is 4πr2 .)

6. [10 MARKS] Show that the equation 3x4 − 28x + 8 = 0 does not have more than
two distinct real roots.
dy d2 y
7. [10 MARKS] Determine the value of and 2 at the point (x, y) = (1, −1) when
dx dx
x and y are related by the equation x3 + xy + y 4 = 1.

8. [10 MARKS] A box with a square base and an open top is to have volume 62.5
cubic centimetres. Neglect the thickness of the material used to make the box, and
find the dimensions that will minimize the amount of material used.
µ ¶
x3
9. Let f (x) = ln .
x2 − 1

UPDATED TO October 25, 2004


Information for Students in MATH 140A 2003 09 3089

(a) [4 MARKS] Specify the domain of f , and any points of discontinuity.


(b) [5 MARKS] Determine all horizontal asymptotes and all vertical asymptotes.
(c) [5 MARKS] Determine the interval(s) where f increases, the interval(s) where
f decreases, and all local extrema.
(d) [3 MARKS] Determine the concavity of the graph of f , and find all points of
inflection.
(e) [3 MARKS] Sketch the graph, using the information determined above.
µ ¶
−1 8π
10. [5 MARKS] Showing all your work, determine sin sin .
3

G.7 December 2000 Final examination in 189-140A


1. (a) [5 MARKS] Find the derivative of ln cosh(x2 ) .
µ ¶
2t
(b) [5 MARKS] Find the derivative of arctan . You are expected to
1 + t2
simplify your answer.

2. [10 MARKS] Determine the equation of the tangent to the curve

x2 + y sin x + tan2 y = 1

at the point (1, 0) .

3. [10 MARKS] Use a linear approximation to estimate the value of


√3

5
0.97 + 0.97 .

4. [10 MARKS] Showing all your work, determine the domain of the following func-
tion, and all vertical or horizontal asymptotes to its graph.
2x + 5
f (x) = √ .
x2 − 36

5. [10 MARKS] Let f (x) = |2x + 5|. Show that there is no value of c such that

f (−4) = f (0) + f 0 (c) · (−4 − 0) .

Explain why this does not contradict the Mean Value Theorem.

UPDATED TO October 25, 2004


Information for Students in MATH 140A 2003 09 3090

6. [10 MARKS] A farmer wants to fence an area of 20,000 square metres in a rectan-
gular field, and then divide it into three parts by fences parallel to one of the sides
of the rectangle. What shape of field will minimize the cost of the fencing? Show
all your work.
7. For the function h(x) = x2 e2x , showing all your work
(a) [5 MARKS] Determine the intervals where h is increasing, and where it is
decreasing.
(b) [5 MARKS] Determine all local maxima, and all local minima.
(c) [5 MARKS] Determine intervals of concavity, and all inflection points.
(d) [5 MARKS] Sketch a graph of the function.
8. (a) [5 MARKS] Find the function f that satisfies all of the following conditions:
f 00 (x) = 3ex + 5 sin x , f (0) = 1 , f 0 (0) = 2 .

(b) [5 MARKS] Determine all values of the constant c that will make the
following function continuous everywhere:
½
cx + 1 if x < 5
f (x) =
cx2 − 1 if x ≥ 5

G.8 May 2001 Supplemental/Deferred Examination in 189-140A


√ √
1. (a) [5 MARKS] Showing all your work, evaluate lim ( x + 2 − x) .
x→∞
sin(x2 − 1)
(b) [5 MARKS] Showing all your work, evaluate lim .
x→−1 x+1
x2 + 1
2. (a) [5 MARKS] Find the derivative of .
x2 − 2
µ ¶
1
(b) [5 MARKS] Find the derivative of cos √ .
x+1
(c) [5 MARKS] Find the derivative of x sin−1 x .
3. [10 MARKS] Showing all your work, determine the greatest and least values at-
sin x
tained by the function f (x) = .
(7 − 2 cos x)2
dy d2 y
4. [10 MARKS] If x3 − xy + y 3 = 1 , determine and when x = 1
dx dx2
and y = 0.

UPDATED TO October 25, 2004


Information for Students in MATH 140A 2003 09 3091

5. [10 MARKS] The illumination I of an object by a light source is given by the


S
formula I = 2 units, where S is the strength of the light source, and d is the
d
distance of the object from the light source. If two light sources, one 8 times as
strong as the other, are placed 3 units apart, where should an object be placed on
the line between them so as to receive the least illumination?

6. [10 MARKS] Use a “linear” or “tangent-line” approximation at x = 4 to compute


√ 1
an approximate value for f (x) = x + √ at x = 3.97.
x
7. For the function f (x) = x5 + x6 , showing all your work

(a) [5 MARKS] Determine the intervals where f is increasing, and those where it
is decreasing.
(b) [5 MARKS] Determine all local maxima, and all local minima.
(c) [5 MARKS] Determine intervals of concavity, and all inflection points.
(d) [5 MARKS] Sketch a graph of the function.
1
8. (a) [5 MARKS] If f 0 (x) = x − 1 and f (1) = , find f (x).
5
1
(b) [5 MARKS] If f 0 (x) = and f (0) = 0, use the Mean Value Theorem
1 + x3
to show that
2
f (2) − f (0) =
1 + c3
2
for some c such that 0 < c < 2, and deduce that < f (2) < 2. (Use the
9
Mean Value Theorem — do not determine an exact formula for f (x).)

G.9 December 2001 Final Examination in 189-140A


A TOTAL OF 137 MARKS ARE AVAILABLE ON THIS EXAMINATION.

1. In each of the following cases evaluate the limit, or explain why the limit does not
exist. Show all your work. Do not use l’Hôpital’s Rule.
x−2
(a) [5 MARKS] lim−
x→2 |x − 2|

sin x − sin(3x)
(b) [5 MARKS] lim
x→0 x2 + 6x

UPDATED TO October 25, 2004


Information for Students in MATH 140A 2003 09 3092

x+4
(c) [5 MARKS] lim
x→−4 x2 + 5x + 4
2. In each of the following cases evaluate the limit, or explain why the limit does not
exist. Show all your work. If you wish, you may use l’Hôpital’s Rule.
2
ex − 1
(a) [10 MARKS] lim .
x→0 sec x − 1

sinh(x + 1)
(b) [5 MARKS] lim . (Hint: Express the functions in terms of
x→∞ cosh(x)
exponentials.)
3. Showing your work, find the derivative of each of the following functions at the
points indicated, or explain why the function fails to be differentiable there. In
all cases you are expected to simplify your answers as much as possible, but the
examiners are aware that you do not have a calculator.
π
(a) [5 MARKS] a(x) = ex sin x at x = − .
2

x+5
(b) [5 MARKS] b(x) = √ at x = 4 .
x−5
eln(2x)
(c) [5 MARKS] c(x) = at x = 4.
ln(e3x )
4. A function K(x) is defined as follows, where α and β are constants to be evaluated:
½
α + x − x2 if x < 2
K(x) = 2
x − β(x − 2) − 4 if x ≥ 2

(a) [8 MARKS] Showing all your work, determine all values of α and β — if any
— that will make K continuous at x = 2.
(b) [7 MARKS] Showing all your work, determine all values of α and β — if any
— that will make K differentiable at x = 2.

5. Suppose that y = 1 − xy 4 .
dy
(a) [10 MARKS] Showing all your work, determine the values of and
dx
2
dy
when x = 0 .
dx2
(b) [5 MARKS] Find an equation for the tangent to the curve y = 1 − xy 4
at the point where x = 0 .

UPDATED TO October 25, 2004


Information for Students in MATH 140A 2003 09 3093

1 1
6. Let f (x) = − − 1.
x x−1
(a) [3 MARKS] Determine the domain of f .
(b) [5 MARKS] Determine the intervals where f is increasing, and the intervals
where it is decreasing.
(c) [5 MARKS] Determine the intervals where the graph of f is concave upwards,
and the intervals where it is concave downwards.
(d) [5 MARKS] Determine the local extrema of f , or prove that there are no
local extrema.
(e) [5 MARKS] Determine the inflection points of the graph of f , or prove that
there are no inflection points.
(f) [4 MARKS] Determine the horizontal asymptotes (if any) and the vertical
asymptotes (if any) to the graph of f .
(g) [3 MARKS] Sketch the graph of f .

7. A line ` with positive slope m is drawn through the point (−4, 9) in the plane.

(a) [1 MARK] Find an equation for `.


(b) [2 MARKS] ` intersects the x-axis in a point A and the y-axis in a point B.
Find the coordinates of A and B.
(c) [12 MARKS] Showing all your work, determine which values of m minimize
the area of the triangle AOB (where O denotes the origin).

4
8. (a) [6 MARKS] Showing all your work, find a linear approximation to 10020 .
(b) [2 MARKS] Carefully state the Mean Value Theorem.
(c) [7 MARKS] Showing all your work, apply the Mean Value Theorem to show
that the function arctan x − x is equal to zero only at x = 0 .
(d) [2 MARKS] Prove that the curves y = x and y = arctan x intersect only
at the origin.

G.10 May 2002 Supplemental/Deferred Examination in 189-


140A
1. Compute the derivatives of the functions

(a) [4 MARKS] f (x) = 2x x3



(b) [4 MARKS] g(x) = sinh( x4 + 1)

UPDATED TO October 25, 2004


Information for Students in MATH 140A 2003 09 3094

sec x
(c) [4 MARKS] h(x) =
arctan x
2. Suppose that the function f satisfies

f (x) − 3
lim =4
x→1 x2 − 1

(a) [6 MARKS] Find lim f (x).


x→1

(b) [6 MARKS] Given that f is continuous at x = 1, use the definition of the


derivative to show that f is differentiable at x = 1 and find f 0 (1).

3. The equation
y 5 + xy 2 + x3 = 4x + 3
defines y implicitly as a function of x near the point (2, 1).

(a) [6 MARKS] Determine the values of y 0 and y 00 at this point.


(b) [4 MARKS] Use the tangent line approximation to estimate y when x = 1.98.
x3
4. Let f (x) = .
x2 − 4
(a) [3 MARKS] Compute and simplify f 0 (x) and f 00 (x).
(b) [3 MARKS] Determine, if any, all vertical and horizontal
asymptotes of f .
(c) [3 MARKS] Determine the intervals where f is increasing, and the intervals
where f is decreasing.
(d) [3 MARKS] Determine all local maxima and all local minima.
(e) [3 MARKS] Determine all intervals where (the graph of) f is concave upward
and all intervals where (the graph of) f is concave downward.
(f) [3 MARKS] Determine all inflection points.
(g) [3 MARKS] Sketch the graph of f .

5. Evaluate the following limits, using l’Hôpital’s rule or otherwise:


tan(3x)
(a) [5 MARKS] lim
x→0 sin(5x)
¡ ¢
(b) [6 MARKS] lim xe1/x − x
x→∞

UPDATED TO October 25, 2004


Information for Students in MATH 140A 2003 09 3095

6. A rectangle is inscribed with its base on the x-axis, its upper left vertex on the
y-axis and its upper right vertex on the graph of the function y = e−2x .

(a) [6 MARKS] Find the dimensions of the rectangle that maximize its area. Fully
justify your answer!
(b) [6 MARKS] Find the dimensions of the rectangle that minimize its circum-
ference. Fully justify your answer!

7. [10 MARKS] Use the mean value theorem, or properties


√ of the derivative, to show
that the graphs of y = arcsin x and y = 2x 2 intersect exactly once for x

2
between 2 and 1.

8. In each of the following problems, find the function that satisfies all the stated
conditions:

(a) [6 MARKS] f 0 (x) = e2x − sin x, f (0) = 0.


1
(b) [6 MARKS] g 00 (x) = 2 , g 0 (1) = 0, g(1) = 0.
x

G.11 December 2002 Final Examination in MATH 140 2002 09


A TOTAL OF 140 MARKS ARE AVAILABLE ON THIS EXAMINATION.

1. In each of the following cases evaluate the limit, or explain why the limit does not
exist. Show all your work. Do not use l’Hôpital’s Rule.
5+u
(a) [5 MARKS] lim .
u→−5 u2 − 25

x2
(b) [5 MARKS] lim
x→0 sin2 3x

(c) [10 MARKS] lim x(cosh ln x − sinh ln x).


x→∞

2. In each of the following cases evaluate the limit, or explain why the limit does not
exist. Show all your work. If you wish, you may use l’Hôpital’s Rule.
2 − x2 − 2 cos x
(a) [7 MARKS] lim .
x→0 x4
(b) [8 MARKS] lim x (2 arctan x − π) .
x→∞

3. In each of the following problems you are expected to show your work, and to
simplify your answer as much as possible. The examiners are aware that you do
not have the use of a calculator.

UPDATED TO October 25, 2004


Information for Students in MATH 140A 2003 09 3096

(a) [5 MARKS] Find the value of the derivative of a(x) = tan(1 + x2 ) at x = 0 .


(b) [8 MARKS] Find the value of the derivative of f (x) = 2 · xln x at x = e3 .
(c) [7 MARKS] Find a function f (x) such that f (−1) = 7 and, for x < 0 ,
2
f 0 (x) = .
x
4. Let A be the point (6, −3) on the curve C with equation x2 = y 2 (y + 7) .

(a) [12 MARKS] Showing all your work, determine an equation for the tangent
to the curve at A .
(b) [8 MARKS] A point P is moving along the curve C . Let t represent time.
dy
Determine the value of when P is at position A on the curve, if it is
dt
known that, at that moment,
dx
= 4 m/s.
dt
1
5. Let f (x) = x(4 − x2 ) 2 .

(a) [3 MARKS] Determine the (largest possible) domain of f .


(b) [5 MARKS] Determine the intervals where f is increasing, and the intervals
where it is decreasing.
(c) [5 MARKS] Determine the local extrema of f , or prove that there are no
local extrema.
(d) [5 MARKS] Prove that f 00 (x) = 2x(x2 − 6)(4 − x2 )−3/2 . Then determine the
intervals where the graph of f is concave upwards, and the intervals where
it is concave downwards.
(e) [5 MARKS] Determine the inflection points of the graph of f , or prove that
there are no inflection points.
(f) [4 MARKS] Determine every horizontal asymptote to the graph of f , or prove
that there is none; determine every vertical asymptote to the graph of f , or
prove that there is none.
(g) [3 MARKS] Sketch the graph of f .

6. (a) [8 MARKS] Determine the derivative of the function


1+x
h(x) = arctan x + arctan 1 − arctan .
1−x
for x 6= 1 . You are expected to simplify your answer.

UPDATED TO October 25, 2004


Information for Students in MATH 140A 2003 09 3097

(b) [7 MARKS] Use your solution to question 6.(a) to determine the value of
h(−5) . Only a solution using the previous result will be accepted. Reduce
your answer as much as possible; the examiners are aware that you do not
have the use of a calculator.

7. Let f (x) = sin3 x + 3 · cos3 x .

(a) [10 MARKS] Showing all your work, determine all local maxima (= relative
maxima) and all local minima (= relative minima) x of f such that
π π
− <x< .
2 2

(b) [5 MARKS] Determine the global maximum (= absolute maximum) and


global minimum (= absolute minimum) value of f on the interval
π π
− ≤x≤ .
2 2

(c) [5 MARKS] Showing all your work, use a linear approximation to estimate
the value of √
sin3 62◦ + 3 · cos3 62◦ .

◦ 1 ◦ 3
You may assume that the sine of 30 is , and that the sine of 60 is .
2 2

G.12 May 2003 Supplemental/Deferred Examination in MATH


140 2002 09
1. In each of the following cases, evaluate the limit or explain why the limit does not
exist. Do not use L’Hôpital’s Rule!
2x3 − 3x2 + 7
(a) [3 MARKS] lim
x→∞ −x3 + 5x − 1

x+4−3
(b) [3 MARKS] lim
x→5 x−5
(tan 2x) · (cos 2x)
(c) [3 MARKS] lim
x→0 x
½
2x − 4a2 if x ≤ 2
2. Let a be a real number, and f (x) = 2 .
a(x − 6x + 8) if x > 2

(a) [5 MARKS] Determine the value(s) of a — if any — for which f is continuous.

UPDATED TO October 25, 2004


Information for Students in MATH 140A 2003 09 3098

(b) [5 MARKS] Explain why f is not differentiable at x = 2 if a = −2.

3. Compute the derivatives of the following functions. Simplify your answers!


sin x
(a) [4 MARKS] f (x) = .
ex µ ¶
1
(b) [4 MARKS] g(x) = arctan .
x
³ √ ´
2
(c) [4 MARKS] h(x) = ln x + 1 + x .

4. (a) [5 MARKS] Determine the function f if it is known that f 00 (x) = 6x − 6,


f (0) = −2 and f (1) = 3.
3
(b) [5 MARKS] Let g(x) = √ . Find an antiderivative G of g such that
Ã√ ! 1 − x2
3
G = 0.
2

5. Consider the curve C with equation 2x3 + 2y 3 = 9xy .

(a) [2 MARKS] Verify that the point P = (1, 2) lies on C.


(b) [4 MARKS] Determine the equation of the tangent line to C at P .
d2 y
(c) [6 MARKS] Determine when x = 1. Is C concave upward or concave
dx2
downward at P ?

6. [10 MARKS] Use the Mean Value Theorem to prove that x = 0 is the only solution
of the equation sinh x = x. Carefully explain your reasoning!

7. [10 MARKS] A particle is moving along the parabola 4y = (x + 1)2 in such a


way that its x-coordinate is increasing at the constant rate of 5 units per second.
Determine how fast the distance from the particle to the origin is changing at the
instant the particle is at the point (3, 4).

8. [10 MARKS] A rectangle with sides parallel to the coordinate axes has one vertex
at the origin, one on the positive x-axis, one on the positive y-axis, and its fourth
1
vertex in the first quadrant on the curve y = . What is the maximum
1 + x2
possible area of such a rectangle? Fully justify your answer!

9. Let f (x) = (x2 + 4x + 4)e−x .

(a) [2 MARKS] Determine all x — if any — for which f (x) = 0 .

UPDATED TO October 25, 2004


Information for Students in MATH 140A 2003 09 3099

(b) [4 MARKS] Determine all horizontal asymptote(s) — if any — to the graph


of f .
(c) [4 MARKS] Find all local extrema — if any — of f ; determine which are
local maxima and which are local minima. Determine the intervals where f
is increasing, and the intervals where f is decreasing.
(d) [4 MARKS] Find all inflection points — if any — of f . Determine the intervals
where the graph of f is concave upward, and the intervals where the graph is
concave downward.
(e) [3 MARKS] Sketch the graph of f .

UPDATED TO October 25, 2004


Information for Students in MATH 140 2003 09 4001

H WeBWorK
H.1 Frequently Asked Questions (FAQ)
H.1.1 Where is WeBWorK?
WeBWorK is located on Web servers of the Department of Mathematics and Statistics,
and is accessible at the following URL’s:

http://msr01.math.mcgill.ca/webwork/m140f03
or
http://msr02.math.mcgill.ca/webwork/m140f03

If your student number ends with 1, 3, 5, 7, or 9, you should use the URL

http://msr01.math.mcgill.ca/webwork/m140f03;

if your student number ends with 0, 2, 4, 6, or 8, you should use

http://msr02.math.mcgill.ca/webwork/m140f03.

If you access WeBWorK through WebCT, the link on your page will have been pro-
grammed to take you to the correct WeBWorK server automatically.

H.1.2 Do I need a password to use WeBWorK?


You will need a user code and a password.

Your user code. Your user code will be your 9-digit student number.

Your password. Your initial password will be your 9-digit student ID number. You
will be able to change this password after you sign on to WeBWorK.73

Your e-mail address. The WeBWorK system requires each user to have an e-mail
address. After signing on to WeBWorK, you should verify that the e-mail address
shown is the one that you prefer. You should endeavour to keep your e-mail address up
to date, since the instructors may send messages to the entire class through this route.
We suggest that you use either your UEA74 or your po-box address. You may forward
your mail from these addresses to any other convenient address, (cf. §1.8.1.)
73
If you forget your password you will have to send a message to your instructor so that the system
administrator may be instructed to reset the password at its initial value.
74
Uniform E-mail Address

UPDATED TO October 25, 2004


Information for Students in MATH 140 2003 09 4002

H.1.3 Do I have to pay an additional fee to use WeBWorK?


WeBWorK is available to all students registered in the course at no additional charge.

H.1.4 When will assignments be available on WeBWorK?


Each assignment will have a begin date and a due date. The assignment is available to
you after the begin date; solutions will be made available soon after the due date.

H.1.5 Do WeBWorK assignments cover the full range of problems that I


should be able to solve in this course?
The “Precalculus” WeBWorK assignments (##P1 through P6) are based on precal-
culus material — on problems that you should have been able to solve before taking this
course.
WeBWorK problems are typically short, and only very brief answers are expected.
The questions on the “Regular” WeBWorK assignments (##R1 through R10) are a
sampling of some types of problem you should be able to solve after successfully com-
pleting this course. Some types of calculus problems do not lend themselves to this kind
of treatment, and may not appear on the WeBWorK assignments. Use of WeBWorK
does not replace studying the textbook — including the worked examples,
attending lectures and tutorials, and working exercises from the textbook
— using the Student Solutions Manual [3] to check your work. Students are
cautioned not to draw conclusions from the presence, absence, or relative frequencies of
problems of particular types, or from particular sections of the textbook. Certain sec-
tions of the textbook remain examination material even though no problems are included
in the WeBWorK assignments.

H.1.6 May I assume that the distribution of topics on quizzes and final
examinations will parallel the distribution of topics in the WeBWorK
assignments?
No!

H.1.7 WeBWorK provides for different kinds of “Display Mode”. Which


should I use?
“Display mode” is the mode that you enter when you first view a problem; and, later,
when you submit your answer. You may wish to experiment with the different formats.
The “best” is usually typeset2 mode, which should look similar to the version that
you print out (cf. next question); of intermediate quality is formatted text, but some

UPDATED TO October 25, 2004


Information for Students in MATH 140 2003 09 4003

characters may fail to print properly; the lowest quality is text mode, which is essentially
the way the author of the problem entered his data into the system. If your computer has
difficulty displaying in typeset2 75 mode, you may have to use one of the other modes.
Typeset mode is related to the TEX and LATEX systems that mathematicians use in
typesetting their documents; the notes that you are reading here were prepared using
LATEX; it contains formatting instructions in a “markup” language, and is difficult for
inexperienced readers.

H.1.8 WeBWorK provides for printing assignments in “Portable Document


Format” (.pdf ) or “PostScript” (.ps) form. Which should I use?
Most newer home computers have already been loaded with the Acrobat Reader for .pdf
files; if the Reader has not been installed on your computer76 , you will find instructions
for downloading this (free) software in §1.5.5 of these notes. If you are not happy with
.pdf files, and wish to print and view PostScript files, you may require such (free) software
as Ghostscript and Ghostview, available at
http://www.cs.wisc.edu/∼ghost/gsview/index.html
Most computers available to you on campus should be capable of printing in either
of .pdf and PostScript formats.

H.1.9 What is the relation between WeBWorK and WebCT?


WebCT is the proprietary system of Web Course Tools that has been implemented
by McGill University. You may access the web page for this course, and WeBWorK
through your WebCT account77 , and WebCT will link you to the appropriate server
for WeBWorK. If you follow this route to WeBWorK, you will still have to log in
when you reach the WeBWorK site. At the present time we will be using WebCT
primarily for the posting of grades, and as a convenient repository for links to notes and
announcements in the course. We are not planning to use the potential WebCT sites
that exist for the tutorial sections: use only the site for the lecture section in which you
are registered.

H.1.10 Which browser should I use for WeBWorK?


We recommend that you use Internet Explorer or Netscape. While other browsers may
give satisfactory results, your instructors and tutors do not have time to correct errors in
75
or typeset, which is another mode currently available
76
At the time these notes were written, the latest version of the Reader was 6.0, but recent, earlier
versions will also work properly.
77
http://webct.mcgill.ca

UPDATED TO October 25, 2004


Information for Students in MATH 140 2003 09 4004

your WeBWorK records that could be attributed to idiosyncracies in another browser.


Information about browsers supported by WebCT may be obtained at

http://ww2.mcgill.ca/icc/webct/browserCheck/browser.html#recommend

H.1.11 What do I have to do on WeBWorK?


After you sign on to WeBWorK, and click on “Begin Problem Sets”, you will see a list
of Assignments, each with a due date. You are required to obtain a grade of at least
75% on one of the “Precalculus” Assignments (##P1, P2, ..., P6) during the term; since
there is no limit to the number of attempts at problems on P1, you may play with this
assignment to learn how to use the WeBWorK software.
You may print out a copy of your assignment by clicking on “Get hard copy”. This is
your version of the assignment, and it will differ from the assignments of other students
in the course. You should spend some time working on the assignment away from the
computer. When you are ready to submit your solutions, sign on again, and select the
same assignment. This time click on “Do problem set”. You can expect to become more
comfortable with the system as you attempt several problems; but, in the beginning,
there are likely to be situations where you cannot understand what the system finds
wrong with some of your answers. It is useful to click on the Preview Answers button to
see how the system interprets an answer that you have typed in. As the problems may
become more difficult, you may have to refer to the “Help” page, and also to the “List
of functions” which appears on the page listing the problems. Don’t submit an answer
until you are happy with the interpretation that the Preview Answers button shows that
the system will be taking of your answer.

H.1.12 How can I learn how to use WeBWorK?


As soon as your instructor announces that the WeBWorK accounts are ready, sign
on and try assignments P1 or R1. Neither of these will have limits on the number of
attempts at problems. The system is self-instructive, so we will not burden you with a
long list of instructions.

You will need to learn how to enter algebraic expressions into WeBWorK as it is coded
to read what you type in a way that may different from what you expect. For exam-
ple, the symbol ^ is used for writing exponents (powers). If you type 2^3, WeBWorK
will interpret this as 23 = 8. However, if you type 2^3+x, WeBWorKwill interpret
it as 23 + x, i.e. as 8 + x; if you wish to write 23+x , you have to type 2^(3+x). You
may obtain more information from the List of Available Functions, available online, or at
http://msr01.math.mcgill.ca/webwork_system_html/docs/available_functions.html

UPDATED TO October 25, 2004


Information for Students in MATH 140 2003 09 4005

H.1.13 Where should I go if I have difficulties with WeBWorK ?


If you have difficulties signing on to WeBWorK, or with the viewing or printing func-
tions on WeBWorK, or with the specific problems on your version of an assignment,
you may send an e-mail distress message directly from WeBWorK by clicking on the
Feedback button. You may also report the problem to your instructor and/or your
tutor, but the fastest way of resolving your difficulty is usually the Feedback . Please
give as much information as you can. (All of the instructors and tutors are able to view
from within WeBWorK the answers that you have submitted to questions.)
If your problem is mathematical, and you need help in solving a problem, you should
consult one of the tutors at their office hours; you may go to any tutor’s office hours, not
only to the hours of the tutor of the section in which you are registered.

H.1.14 Can the WeBWorK system ever break down or degrade?


Like all computer systems, WeBWorK can experience technical problems. The systems
manager is continually monitoring its performance. If you experience a difficulty when
online, please click on the Feedback button and report it. If that option is not available
to you, please communicate with either instructor by e-mail.
If you leave your WeBWorK assignment until the hours close to the due time on
the due date, you should not be surprised if the system is slow to respond. This is
not a malfunction, but is simply a reflection of the fact that other students have also
been procrastinating! To benefit from the speed that the system can deliver under normal
conditions, do not delay your WeBWorK until the last possible day! If a systems failure
interferes with the due date of an assignment, arrangements will be made to change that
date, and an e-mail message will be broadcast to all users (to the e-mail addresses on
record).78

H.1.15 How many attempts may I make to solve a particular problem on


WeBWorK?
For Precalculus Assignment #P1 the number of attempts will not be limited. How-
ever, for later Precalculus Assignments ##P2–P6 there could be a maximum number of
attempts.
Regular Assignments ##R1, R3, R5, R7, R9 are intended to prepare you for the even
numbered regular assignments, and there will be no limit on the numbers of attempts
on the former. It is planned to limit the number of attempts at problems on the even
numbered assignments; information about the number of attempts permitted should
78
But slowness of the system just before the due time will not normally be considered a systems
failure.

UPDATED TO October 25, 2004


Information for Students in MATH 140 2003 09 4006

appear at the top of each assignment. The odd numbered assignment is intended to
prepare you for a painless completion of the subsequent even numbered assignment.

H.1.16 Will all Regular WeBWorK assignments have the same length? the
same value?
The numbers of problems on the various assignments may not be the same, and the
individual problems may vary in difficulty. But all 10 Regular assignments will count
equally in the 10% of the final grade allocated to them. TO REITERATE: You will
receive credit for both the grade you obtain on the odd-numbered assignment (with
unlimited attempts) and your grade on following even-numbered assignment (with re-
stricted attempts).

H.1.17 Is WeBWorK a good indicator of examination performance?


A low grade on WeBWorK has often been followed by a low grade on the examination.
A high grade on WeBWorK does not necessarily indicate a likely high grade on the
examination.
To summarize: WeBWorK alone is not enough to prepare this course; but students
who don’t do WeBWorK appear to have a poor likelihood of success in Math 140: that
is why we have made the WeBWorK assignments compulsory.

UPDATED TO October 25, 2004


Information for Students in MATH 140 2003 09 5001

I Contents of the DVD disks for


Larson/Hostetler/Edwards
These excellent disks were produced to accompany the textbook, Calculus of a Single
Variable: Early Transcendental Functions, 3rd Edition[18] (called LHE in the charts
below). The correspondence shown to sections of [10] are only approximate.

DVD LHE Stewart


# Section Subject Minutes Section
1 P Chapter P: Preparation for Calculus
1 P.1 Graphs and Models 45
1 P.2 Linear Models and Rates of Change 27 A10
1 P.3 Functions and Their Graphs 48 1.1
1 P.4 Fitting Models to Data 21 1.2
1 P.5 Inverse Functions 48 1.6
1 P.6 Exponential and Logarithmic Functions 30 1.5
DVD LHE Stewart
# Section Subject Minutes Section
1 1 Chapter 1: Limits and Their Properties
1 1.1 A Preview of Calculus 11 2.1
1 1.2 Finding Limits Graphically and Numeri- 25 2.2, 2.4
cally
1 1.3 Evaluating Limits Analytically 28 2.3
1 1.4 Continuity and One-Sided Limits 22 2.5
1 1.5 Infinite Limits 18 2.6
DVD LHE Stewart
# Section Subject Minutes Section
1 2 Chapter 2: Differentiation
1 2.1 The Derivative and the Tangent Line Prob- 68 2.1
lem
1 2.2 Basic Differentiation Rules and Rates of 34 2.3
Change
1 2.3 The Product and Quotient Rules and 25 3.2, 3.7
Higher Order Derivatives

UPDATED TO October 25, 2004


Information for Students in MATH 140 2003 09 5002

DVD LHE Stewart


# Section Subject Minutes Section
2 2 Chapter 2 (continued): Differentiation
2 2.4 The Chain Rule 44 3.5
2 2.5 Implicit Differentiation 50 3.6
2 2.6 Derivatives of Inverse Functions 17 3.5, 3.8, 3.9
2 2.7 Related Rates 34 3.10
2 2.8 Newton’s Method 26 4.9
DVD LHE Stewart
# Section Subject Minutes Section
2 3 Chapter 3: Applications of Differentiation
2 3.1 Extrema on an Interval 41 4.1
2 3.2 Rolle’s Theorem and the Mean Value The- 15 4.2
orem
2 3.3 Increasing and Decreasing Functions and 19 4.3
the First Derivative Test
2 3.4 Concavity and the Second Derivative Test 24 4.3
2 3.5 Limits at Infinity 23 2.6
2 3.6 A Summary of Curve Sketching 43 4.5
2 3.7 Optimization Problems 37 4.7
2 3.8 Differentials 51 3.11
DVD LHE Stewart
# Section Subject Minutes Section
3 4 Chapter 4: Integration
3 4.1 Antiderivatives and Indefinite Integration 40 4.10
DVD LHE Stewart
# Section Subject Minutes Section
4 7 Chapter 7: Integration by Parts Trigono-
metric Substitution Partial Fractions
L’Hôpital’s Rule
4 7.7 Indeterminate Forms and L’Hôpital’s Rule 22 4.4

(The coverage extends to part of the material for Math 141 as well.)

UPDATED TO October 25, 2004


Information for Students in MATH 140 2003 09 6001

J References
J.1 Stewart Calculus Series
[1] J. Stewart, Single Variable Calculus (Early Transcendentals), Fifth Edition. Thom-
son*Brooks/Cole (2003). ISBN 0-534-39330-6.
[2] J. Stewart, Calculus (Early Transcendentals), Fifth Edition. Thom-
son*Brooks/Cole (2003). ISBN 0-534-39321-7.
[3] D. Anderson, J. A. Cole, D. Drucker, Student Solutions Manual for Stewart’s Single
Variable Calculus (Early Transcendentals), Fifth Edition. Thomson*Brooks/Cole
(2003). ISBN 0-534-39333-0.
[4] J. Stewart, Single Variable Calculus (Early Transcendentals), Fifth Edition.
Thomson*Brooks/Cole (2003); bundled with Student Solutions Manual for Stew-
art’s Single Variable Calculus (Early Transcendentals), Fifth Edition. Thom-
son*Brooks/Cole (2003). ISBN 0-534-42976-9.
[5] J. Stewart, Calculus (Early Transcendentals), Fifth Edition. Thom-
son*Brooks/Cole (2003); bundled with Student Solutions Manual for Stew-
art’s Single Variable Calculus (Early Transcendentals), Fifth Edition. Thom-
son*Brooks/Cole (2003). ISBN 0-534-10307-3.
[6] R. St. Andre, Study Guide for Stewart’s Single Variable Calculus (Early Transcen-
dentals), Fifth Edition. Thomson*Brooks/Cole (2003). ISBN 0-534-39331-4.
[7] Video Outline for Stewart’s Calculus (Early Transcendentals), Fifth Edition.
Thomson*Brooks/Cole (2003). ISBN 0-534-39325-X. 17 VCR tapes.
[8] Interactive Video Skillbuilder CD for Stewart’s Calculus: Early Transcendentals,
5th Edition. Thomson*Brooks/Cole (2003). ISBN 0-534-39326-8.
[9] H. Keynes, J. Stewart, D. Clegg, Tools for Enriching Calculus, CD to accompany
[1] and [2]. Thomson*Brooks/Cole (2003). ISBN 0-534-39731-X.
[10] J. Stewart, Single Variable Calculus (Early Transcendentals), Fourth Edition.
Brooks/Cole (1999). ISBN 0-534-35563-3.
[11] J. Stewart, Calculus (Early Transcendentals), Fourth Edition. Brooks/Cole (1999).
ISBN 0-534-36298-2.
[12] D. Anderson, J. A. Cole, D. Drucker, Student Solutions Manual for Stewart’s
Single Variable Calculus (Early Transcendentals), Fourth Edition. Brooks/Cole
(1999). ISBN 0-534-36301-6.

UPDATED TO October 25, 2004


Information for Students in MATH 140 2003 09 6002

J.2 Other Calculus Textbooks


J.2.1 R. A. Adams

[13] R. A. Adams, Calculus, Single Variable, Fifth Edition. Addison, Wesley, Longman,
Toronto (2003). ISBN 0-201-79805-0.

[14] R. A. Adams, Calculus of Several Variables, Fifth Edition. Addison, Wesley, Long-
man, Toronto (2003). ISBN 0-201-79802-6.

[15] R. A. Adams, Calculus: A Complete Course, Fifth Edition. Addison, Wesley, Long-
man, Toronto (2003). ISBN 0-201-79131-5.

[16] R. A. Adams, Student Solution Manual for Adams’, Calculus: A Complete Course,
Fifth Edition. Addison, Wesley, Longman, Toronto (2003). ISBN 0-201-79803-4.

[17] R. A. Adams, Calculus: A Complete Course, with Solution Manual, Fifth Edition.
Addison, Wesley, Longman, Toronto (2003). ISBN 0-131-30565-4.

J.2.2 Larson, Hostetler, et al.

[18] Calculus Instructional DVD Program, for use with (inter alia) Lar-
son/Hostetler/Edwards, Calculus of a Single Variable: Early Transcendental Func-
tions, Third Edition [19]. Houghton Mifflin (2003). ISBN 0-618-25097-2.

[19] R. Larson, R. P. Hostetler, B. H. Edwards, D. E. Heyd, Calculus, Early Transcen-


dental Functions, Third Edition. Houghton Mifflin Company, Boston (2003). ISBN
0-618-22307-X.

J.2.3 Edwards and Penney

[20] C. H. Edwards, Jr., and D. E. Penney, Single Variable Calculus, Early Transcen-
dentals, Sixth Edition. Prentice Hall, Englewood Cliffs, NJ (2002). ISBN 0-13-
041407-7.

[21] C. H. Edwards, Jr., and D. E. Penney, Calculus with Analytic Geometry, Early
Transcendentals Version, Fifth Edition. Prentice Hall, Englewood Cliffs, NJ (1997).
ISBN 0-13-793076-3.

[22] C. H. Edwards, Jr., and D. E. Penney, Student Solutions Manual for Calculus with
Analytic Geometry, Early Transcendentals Version, Fifth Edition. Prentice Hall,
Englewood Cliffs, NJ (1997). ISBN 0-13-079875-4.

UPDATED TO October 25, 2004


Information for Students in MATH 140 2003 09 6003

[23] C. H. Edwards, Jr., and D. E. Penney, Single Variable Calculus with Analytic
Geometry, Early Transcendentals Version, Fifth Edition. Prentice Hall, Englewood
Cliffs, NJ (1997). ISBN 0-13-793092-5.

[24] C. H. Edwards, Jr., and D. E. Penney, Student Solutions Manual for Single Variable
Calculus with Analytic Geometry, Early Transcendentals Version, Fifth Edition.
Prentice Hall, Englewood Cliffs, NJ (1997). ISBN 0-13-095247-1.

J.2.4 Others, not “Early Transcendentals”

[25] G. H. Hardy, A Course of Pure Mathematics, 10th edition. Cambridge University


Press (1967).

J.3 Other References


[26] D. Ebersole, D. Schattschneider, A. Sevilla, K. Somers, A Companion to Calculus.
Brooks/Cole (1995). ISBN 0-534-26592-8.

[27] McGill Undergraduate Programs Calendar 2002/2003. Also accessible at


http://www.mcgill.ca/courses/#UGRAD.

UPDATED TO October 25, 2004

You might also like